Download as doc, pdf, or txt
Download as doc, pdf, or txt
You are on page 1of 158

STT

MỤC LỤC Trang


A. NĂM HỌC 2014 - 2015
1 Đề thi vào lớp 10 THPT Chuyên Nguyễn Trãi - Hải Dương 2
2 Đề thi vào lớp 10 THPT Chuyên Lê Hồng Phong - Nam Định 6
3 Đề thi vào lớp 10 THPT Chuyên Phan Bội Châu - Nghệ An 10
4 Đề thi vào lớp 10 THPT Chuyên Long An 15
B. ĐỀ THI NĂM 2015 - 2016
5 Đề thi vào lớp 10 THPT Chuyên Long An 19
6 Đề thi vào lớp 10 THPT Chuyên Đại học Sư Phạm Hà Nội 21
7 Đề thi vào lớp 10 THPT Chuyên Bắc Giang 27
8 Đề thi vào lớp 10 THPT Chuyên Ngoại ngữ Đại học Quốc gia Hà Nội 32
9 Đề thi vào lớp 10 THPT Chuyên TP. Hồ Chí Minh 36
10 Đề thi vào lớp 10 THPT Chuyên Hưng Yên 42
C. ĐỀ THI NĂM 2016 - 2017
11 Đề thi vào lớp 10 THPT Chuyên Nguyễn Trãi - Hải Dương 46
12 Đề thi vào lớp 10 THPT Chuyên Vĩnh Phúc 50
13 Đề thi vào lớp 10 THPT Chuyên Nguyễn Tất Thành - Kon Tum 55
14 Đề thi vào lớp 10 THPT Chuyên chung Hà Nam 61
15 Đề thi vào lớp 10 THPT Chuyên Lê Hồng Phong - Nam Định 64
D. ĐỀ THI NĂM 2017 - 2018
16 Đề thi vào lớp 10 THPT Chuyên Quốc Học Huế 71
17 Đề thi vào lớp 10 THPT Chuyên Vĩnh Phúc 75
18 Đề thi vào lớp 10 THPT Chuyên Lê Hồng Phong - Nam Định 81
19 Đề thi vào lớp 10 THPT Chuyên An Giang 84
20 Đề thi vào lớp 10 THPT Chuyên Bình Dương 89
21 Đề thi vào lớp 10 THPT Chuyên Đại Học Sư Phạm Hà Nội 92
ĐỀ THI NĂM 2018-2019
22 Đề thi vào lớp 10 THPT Chuyên TP. Hồ Chí Minh 98
23 Đề thi vào lớp 10 THPT Chuyên Hà Nam 104
24 Đề thi vào lớp 10 THPT Chuyên Bình Phước 109
25 Đề thi vào lớp 10 THPT Chuyên Bắc Giang 115
ĐÁP ÁN VÀ HƯỚNG DẢN GIẢI 123

1
Phần 1. ĐỀ THI VÀO LỚP 10 CHUYÊN
A. ĐỀ THI NĂM 2014 - 2015
ĐỀ SỐ 01
(Đề thi vào lớp 10 THPT Chuyên Nguyễn Trãi - Hải Dương)
Thời gian làm bài: 120 phút
A. PHONETICS
I. Choose the word whose underlined part is pronounced differently from that of the others (2.0
points):
1. A. hardware B. custard C. bargain D. chart
2. A. thanked B. blocked C. locked D. naked
II. Choose the word that has a different stress pattern from that of the others (3.0 points):
3. A. absolutely B. photography C. biology D. geography
4. A. purity B. accompany C. immense D. advantage
5. A. important B. disease C. organize D. community
B. GRAMMAR - VOCABULARY - LANGUAGE FUNCTIONS
I. Choose the word, phrase of expression which best completes each sentence (20 points):
6. I remember……………this film when I was a small child.
A. to watch B. watched C. watching D. to be watching
7. My new job……………working with old age pensioners.
A. requests B. consists C. involves D. contains
8. Jack: “Mary, remember to send my regards to your grandfather.”
Mary: “……………”
A. You’re welcome. B. Thanks, I will. C. That’s a good idea. D.It’s my pleasure.
9. The man……………two weeks ago has now been released.
A. arrested B. arresting C. was arrested D. who arrested
10. Did you……………well with your roommates when you were at university?
A. take on B. put on C. go on D. get on
11…………… to the beach before, little John was excited to see the huge waves crashing against the shore.
A. Having not been B. Not having been C. Not to have been D. Not being
12. Jenny bought a……………..when she was in Paris.
A. silk pretty red dress B. red pretty silk dress
C. dress pretty silk red D. pretty red silk dress
13. No doubt, sleeplessness can have a noticeable effect……………..our performance at work.
A. to B. with C. on D. for
14. I am sorry, but we can’t use the elevator at the moment because it is……………..
A. out of work B. out of sight C. out of order D. out of duty
15. If I……………..in your position, I would refuse to take part in the game.
A. am B. were C. have been D. be
16. Mr. Brown: “It’s very nice of you to meet me here.”
Tom: “……………..”
A. That’s right, sir. B. Yes, certainly, sir.
C. You’re right, sir. D. It’s my pleasure, sir.
17. “How long will you stay here, Susan?” the man asked.
A. The man wanted to know how long Susan will stay there.
B. The man asked Susan how long I would stay there
C. The man asked Susan how long would she stay there
D. The man asked Susan how long she would stay there

2
18. We must go now,……………..we will get stuck in the traffic.
A. or else B. so that C. although D. in order that
19. “Your aunt has rarely been abroad,……………..?”the policeman asked Tony.
A. has not she B. has she C. hasn’t she D. has your aunt
20. The notice “Keep out of reach of children!” can be seen……………..
A. at the gate of a supermarket B. on a box or bottle of medicine
C. on an electric post D. at a gas station
21. “There's someone behind us. Do you think we are……………..?” Jack asked worriedly.
A. following B. being following C. followed D. being followed
22. “How I hate to see the dentist tomorrow! I hope I don’t need to……………..,”
Bob cried out.
A. have anything done B. be done something
C. make something done D. let anything done
23. The party was awful. Jane wishes she……………..to it.
A. did not go B. had not gone
C. has not gone D. would not have gone
24. It was ……………..difficult a question that all of us could not answer it.
A. such B. so C. too D. very
25.……………..you give the answer,……………..marks you can get.
A. The quicker / the better B. The more quickly / better
C. The quickest / the best D. The more quickly / the better
II. Give the correct form of the words in brackets (5.0 points):
26. We regret to inform you that you are not (QUALIFICATION)……………..for the job.
27. How will you (BEAUTIFUL)……………..the Christmas free, Jane?
28. The man left his village on a (MISERY)……………..wet day.
29. It is education that helps with the (RICH) …………….. of knowledge for the young.
30. Henry drank a lot of coffee to keep him (WAKE)……………...
III. Choose the underlined word or phrase in each sentence that needs correcting (5.0 points):
31. Dictionaries frequently explain the origin of the defined word, state its part of speech, and indicating its
correct use. A B C D
32. John's parents are very glad that he has done lots of progress this school year.
A B C D
33. The Prime Minister, accompanied by his attendants, are present at the opening of the new sports center.
A B C D
34. The receptionist said Tom to sit down at the table and wait for the manager.
A B C D
35. There are many different ways of comparing the economy of one nation with those of another.
A B C D
C. READING
I. There are five blanks in the passage below. From the words given in the box, choose the most
suitable for each blank. There are TWO EXTRA WORDS that you do not need to use (5.0 points) :
A. taken C. made E. considerably G. experiment
B. total D. experience F. however

For many people, traveling by plane is an exciting (36)……………... Others, however, find the whole idea
quite terrifying, although flying is no more dangerous than any other forms of travel and some experts say it
is (37)…………….. safer. It is known, (38)…………….., that most accidents occur during take-off and
landing when the pilot’s decisions are very important.
The people whose job is to look after the passengers - the stewards and stewardesses - play an

3
important role in helping passengers to feel safe and comfortable. Indeed for many passengers, being (39)
……………..such care of is all part of the (40)……………..experience. No other form of travel is required
to wait on people in quite the same way, with food, drink, newspapers, magazines, music and even video
films.
II. Read the passage below and decide which option A, B, C or D best fits each space (10 points):
The media are the means or methods by which people learn what is (41)……………..in the city, in
the country and in the world. The news media can be classified (42)…………….. two general categories: the
categories of print media and electronic media. Print media use the written material to communicate news to
(43)……………... Electronic media use air waves to (44)……………..news to homes, offices and public
places. Print media are usually divided into magazines and newspapers. Most newspapers print news daily.
For example, the newspaper “The New York Time” is (45)……………..daily. However, Newsweek
and Time magazines are published once a week. The electronic media are (46)……………..divided into
radio and television. (47)……………..news is news that you listen to. In the United States many radio (48)
……………..broadcast five minutes of news every hour. Television news is news that you not only listen to
but also watch. In Canada and the United States, for example, many people watch an hour of news on TV at
six o’clock in the evening. In the future, new categories of news media will (49)…………….. even today
computers are beginning to influence the transmission and reception (50)……………..news.
41. A. showing B. happening C. making D. doing
42. A. into B. to C. for D. about
43. A. viewers B. listeners C. spectators D. readers
44. A. receive B.send C. offer D. get
45. A. published B. made C. taken D. brought
46. A. carefully B. especially C. generally D. completely
47. A. Print B. Newspaper C. Magazine D. Radio
48. A. stations B. offices C. buildings D. houses
49. A. discover B. develop C. invent D. disappear
50. A. for B. into C. of D. at
III. Read the following passage and choose the best answer to each question (10 points):
During the teenage years, many young people can at times be difficult to talk to. They often seem to
dislike being questioned. They may seem unwilling to talk about their work in school. This is a normal
development at this age, though it can be very hard for parents to understand. It is part of becoming
independent of teenagers trying to be adult while they are still growing up. Young people are usually more
willing to talk if they believe that questions are asked out of real interest and not because people are frying to
check up on them.
Parents should do their best to talk to their sons and daughters about school work and future plans.
However, it is not a good idea that parents push teenagers to talk if they do not want to. Parents should also
watch for the danger signs: some young people in trying to be adult may experiment with sex, drugs, alcohol,
or smoking. Parents need to watch for any signs of unusual behavior which may be connected with these and
get help if necessary.
51. This passage is taken from a……………...
A. handbook for parents B. school timetable
C. teenage magazine D. book for children
52. Sometimes it is difficult for adults to talk to teenagers because……………..
A. teenagers think adults are not honest
B. teenagers do not like the ways adults ask them
C. most teenagers are quiet
D. most teenagers hate adults
53. Which of the following is NOT true, according to the passage?
A. That teenagers do not want to talk to adults is unusual at their age.

4
B. Teenagers will be unwilling to talk if people try to check up on them.
C. It is advisable for parents to talk to their sons and daughters about school work and future plans.
D. Parents should not force teenagers to talk if they do not want to.
54. Some teenagers experiment with drugs, smoking or alcohol because……………..
A. cigarettes, drugs, or alcohol help them relax
B. cigarettes, drugs and alcohol are available everywhere
C. they feel proud to be respected by adults
D. they regard these as a mark of adulthood
55. The word behavior in the last sentence is closest in meaning to……………...
A. feeling B. reaction. C. activities D. manners
D. WRITING
I. Finish each of the following sentences in such a way that it means exactly the same as the sentence
printed before (5.0 points):
56. Nam finds it hard to drive on the left.
Nam is not ......................................................................................................................................
57. “You’ve damaged my car, Bob!” the mother said angrily.
The mother accused........................................................................................................................
58. Passengers always get something to eat on this airline even though the journey is short.
However.........................................................................................................................................
59. It was such a stale cake that the boy could not eat it.
The cake was not............................................................................................................................
60. Despite the heavy snow, the explorers managed to walk to the village.
Although........................................................................................................................................
II. Use the suggested words and phrases below to make complete sentences (10 points):
61. Everyone / become / aware / environment / be / serious issue.
......................................................................................................................................................................
62. However, / we / not / do / much / deal with / problem / because / we / seem / wait / government / take
actions.
......................................................................................................................................................................
63. In / opinion, / individuals / can / do / things / help / solve / problem.
......................................................................................................................................................................
64. begin with, / we / can / responsible / way / we / dispose / waste.
......................................................................................................................................................................
65. We / not / throw / rubbish / lakes / rivers.
......................................................................................................................................................................
66. Moreover, / we / also / need / save / water / we / use / because / fresh water / be / run out / many parts /
world.
......................................................................................................................................................................
67. Finally, / air / we / breathe / be / pollute / exhaust fumes / cars / cities.
......................................................................................................................................................................
68. I think / if / we / use / public transport / more, / we / reduce / air pollution.
......................................................................................................................................................................
III. It is said that traveling helps widen your knowledge. To what extent do you agree or disagree with
this statement? In about 180 words, write an essay to support your opinion (10 points).

5
ĐỀ SỐ 02
(Đề thi vào lớp 10 THPT Chuyên Lê Hồng Phong - Nam Định)
Thời gian làm bài: 120 phút
PART A. PHONETICS (1.0 point)
I. Choose the word whose underlined part is pronounced differently from that of the others. Circle the
correct answer A, B, C or D. There is an example at the beginning (0). (0.5p)
0. A. achieve B. children C.change D. chemistry
1. A. multiply B. simply C. imply D. purify
2. A. define B. decline C. determine D. mine
3. A. honest B. history C. hour D. honour
4. A. wood B. good C. food D. childhood
5. A. worked B. placed C. wicked D. missed
II. Choose the word whose stress pattern is different from that of the others. Circle the correct answer
A, B, C or D. There is an example at the beginning (0). (0.5p)
0. A. money B. army C afraid D. people
1. A. provide B. combine C. service D. account
2. A. electric B. dependent C. deposit D. different
3. A. imagine B. protection C. develop D. company
4. A. documentary B. occupation C. competitive D. individual
5. A. embarrassing B. experiment C. artificial D. experience
PART B. GRAMMAR AND VOCABULARY (3.0 points)
I. Choose the best word or phrase to complete the following sentences. Circle the correct answer A, B,
C or D. There is an example at the beginning (0).
0. Singapore is…………….than Vietnam.
A. much small B. less smaller C. more smaller D. much smaller
1. The girl…………….over there is my best friend.
A. stood B. standing C. stands D. who standing
2. My sister has a hat…………….yours. She bought it in Hanoi.
A. as B. like C.such as D. same as
3. The invention of labor-saving devices for the home made…………….for women to pursue a career and to
enjoy their social life.
A. possible B. it possible C. possible it D. it impossible
4.……………. you say, I won’t believe you anymore.
A. However B. How C. What D. Whatever
5. While some people find advertising very helpful,…………….think it is a waste of time and money.
A. some other B. others C. other D. others people
6. No longer…………….their parents allow them to go to the music club.
A. X (nothing) B. does C.do D. have
7. I regret…………….that the sight-seeing tour has been cancelled because of the bad weather.
A. to tell you B. telling you C. to say you D. saying you
8. The harder you try,…………….results you get.
A. the better B. better C. more better D. the best
9.…………….students study very hard before the examination.
A. Most of the B. Most of C. Most the D. Almost
10. If it…………….for her carelessness, she could have passed the driving test.
A. had been B. hadn’t been
C. were not D. would have been
II. Give the correct form of the words in capital letters to complete the sentence. There is an example

6
at the beginning (0). (l.0p)
0. She has been a stamp…collector…for ten years. (COLLECT)
1. There is a…………….of fruits in Vietnam. (VARY)
2. The company has had a…………….in sales. (REDUCE)
3. Her boss accused her of being over…………….. (AMBITION)
4. She was the first TV…………….that he had met in the flesh. (CELEBRATE)
5. His…………….knowledge is very poor. He thinks Paris is in Italy. (GEOGRAPHY)
6. Even if you’re good at a game, you shouldn’t be…………….. (CONFIDENT)
7. In some places the weather changes so quickly that it’s very……………. (PREDICT)
8. We always have a spare room in case visitors arrive…………….. (EXPECT)
9. Reducing levels of carbon dioxide in the atmosphere is of…………….importance. (CRITIC)
10. There are some …………….who want a new law forcing men to do their share. (POLITICS)
III. Choose one given phrasal verb to complete each sentence. You may have to change the verb form
but cannot use one phrasal verb twice. Write your answers in the spaces provided. There is an example
at the beginning (0).
look after slow down look down on
make out call in draw up
try on ring off put aside
sit up settle on
0. Can you…look after….our dog while we’re away on holiday?
1. His condition is worse than before. I think we should ……………. a doctor.
2. A red car has just…………….in front of our house. Are we expecting anyone?
3. Don’t…………….Peter just because he’s not as good at the job as you are.
4. Could you……………., please. I don’t like driving so fast on country roads.
5. The name on the cheque is wrong. The cheque is…………….to your brother, not to you.
6. I wonder if this dress fits. I’ll…………….it……………. and see.
7. I’m sorry, but I’ll have to…………….now. Someone has just come into the office.
8. Don’t spend the money you won. Why don’t you…………….it…………….to buy what you really want?
9. There was an excellent film on television late last night. We…………….to watch it.
10. We couldn’t decide whether to book a holiday in Spain or Italy. We finally…………….Spain.
PART C. READING (3.0 points)
I. Read the text below and answer the questions. Write your answers in the spaces provided (you can
give short but enough information answers). (1.2p)
I had been chosen by my teacher to attend a leadership training course which would be held at the Girls’
Brigade campsite. As I stepped into the campsite at Sembawang, I saw the ropes that were fastened from tree
to tree and some structures built for climbing. “I hope I don’t have to climb up all those trees,” I said to
myself. I suffered from acrophobia.
After Debbie and I had settled into our dormitory, we went for the briefing. The camp commandant told us
that we need not worry about climbing the awesome structures as she and the other officers were there to
help us. “To be a good leader, you must lead by example and be brave,” the commandant said.
The day came when all of us had to climb up the structures. “This is the moment of my life” I told myself. It
was a sunny afternoon as I stood under the tree and told myself I would be able to do it.
Debbie was among the brave ones who volunteered to climb up the man-made ladder to the top of the tree
where the ropes were. The task was to walk across to the middle of the ropes supported by the two trees and
jump down. Although we had support ropes around our bodies, I was still afraid and tried very hard to
overcome my fear of heights.
My heart pumped so hard while waiting for my turn. When it was finally my turn, I climbed up the ladder
with legs shaking. The higher I climbed, the faster my heart beat. Finally, I reached the top and took a deep
breath. “Phew, that was easy,” I told myself. Then I looked down. “I think I am going to faint,” I thought.

7
I could not think for the next few seconds. I shook my head and asked myseld, “How am I going to show the
people in the Girls’ Brigade that I am a good leader if I have no courage?”
From the top of the tree, I could see the entire campsite as well as the officiers and the other participants. I
hummed a tune. Then I held on top the ropes tightly and began to move one step at a time towards the centre.
I was enjoying the view around me. “This is fun after all! I thought.
Then I heard cheers and claps from below. I realized that I had made it to the centre. I closed my eyes and
jumped. I landed on the ground swiftly. It felt goot to be on the ground again although my legs were still
shaking. At the same time there was a feeling of triumph.
(Source : From SAM assessment paper specialist - Singapore)
1. Where specifically was the training camp held?
......................................................................................................................................................................
2. What does the word 'acrophobia’ in paragraph 1 mean?
......................................................................................................................................................................
3. Which word in the first paragraph has the same meaning as 'secured tightly'?
......................................................................................................................................................................
4. Why do you think the writer was chosen by the teacher to attend the camp?
......................................................................................................................................................................
5. What was it that the writer did not want the Girls’ Brigade to think other?
......................................................................................................................................................................
6. Write down the sentence which tells you that the writer’s fellow participants were very supportive.
......................................................................................................................................................................
II. Read an article about the Sahara Desert. Six sentences have been removed from the article. Choose
from the sentences A-G the one which fits each gap. There is one extra sentence which you do not need
to use. (0) is an example. (1.0p)
The Sahara’s meager resources are being plundered right to the present day. This is shown most
clearly in the dramatic fall in the region’s precious water reserves. The rock is of a type which hinders easy
underground storage so the water supply depends on rainfall. (0) …B… However, the seriousness of the
situation goes far beyond mere seasonal changes.
There’s one district where observations go back at least 125 years. Here it is known that wells of the
balance beam type were once used in cultivated areas. This way of drawing water is intended for the
irrigation of gardens and small plots and works only if there’s water not more than about five metres below
the surface. (1) ................. They have all been replaced by wells from which the water has to be raised by
draught animals, because it is now 25 metres down. (2)..................
In a classic example of a vicious circle, shortage of water has led to the digging of more wells to save
the flocks and birds. As these have been deepened the water supply has been further depleted. (3).................
The Air region was once regarded as the Switzerland of Africa because of its temperate climate.
(4).................. Nowadays, though, there are only a third of the palms which existed there at the turn of the
century. The amount of wildlife has shown a proportionate fall. (5) .................. This is highly dangerous in a
world where the overall population is growing rapidly. We cannot afford to lose fertile land.
A. So, too, has the number of human beings who can now survive in the area.
B. This being so, it is easy to understand why it varies.
C. Today, though, not one well of this type exists.
D. This has resulted in soil erosion which then dries out the land still further.
E. Here, then, the water level has evidently dropped by 20 metres in little over a century.
F. It was criss-crossed by valleys filled with palm trees and was thronged with wildlife.
G. Such are conditions in the Sahara Desert in recent times.
III. Read the passage and fill in each numbered blank with one suitable word? Write your answer in
the spaces provided. There is an example at the beginning (0). (0.8p)
Malaysia Airlines Flight 370 (MH370/MAS370) was (0)……..a…….

8
scheduled international passenger flight from Kuala Lumpur to Beijing that lost (1)………….....
with air traffic control on 8 March 2014 at 01:20 MYT. less than an hour after takeoff. At 07:24, Malaysia
Airlines (MAS) reported the flight missing. The aircraft, a Boeing 777-200ER. was carrying 12 Malaysian
crew members and 227 (2)………….....from 14 nations. There has been no confirmation of any flight debris
and no crash site has been found.
A multinational search and rescue effort, later reported as the largest and most expensive in history, began in
the Gulf of Thailand and the East Sea. Within a few days, the search was extended (3)………….....the Strait
of Malacca and Andaman Sea. On 15 March, (4)…………..... on military rada data and radio "pings"
between the aircraft and an Inmarsat satellite, investigators concluded that the aircraft had diverted from its
intended course and headed west across the Malay Peninsula, then continued on a northern or southern track
(5)………….....around seven hours. The search in the East Sea was abandoned. Three days later, the
Australian Maritime Safety Authority began searching the southern part of the Indian Ocean.
On 24 March, the Malaysian government confirmed independent analyses by the British Air
Accidents Investigation Branch (AAIB) and Inmarsat, and announced that search efforts would be (6).
………….....on the Australian-led area. In the first two weeks of April, aircrafts and ships deployed
equipment to listen for signals from the underwater locator beacons attached to the aircraft's "(7)…………..
box" flight recorders. Four unconfirmed signals (8)………….....detected between 6 and 8 April but after that
time the batteries of the beacons were believed to have become exhausted. The search continued in the area
of the signals, using a robotic submarine, , until 28 May, without evidence of debris being found.
(Source: From Wikipedia, the free encyclopedia)
PART D. WRITING (3.0 points)
I. Complete the second sentence so that it has a similar meaning to the first sentence, using the word
given in bold at the end. Do not change the word given. You must use between two and five words,
including the word given. There is an example at the beginning (0). (2.0p)
0. ‘Are you a member of the committee?’ The receptionist said to me. belonged
→ The receptionist asked….me if I belonged to……the committee.
1. This is the most amusing novel I’ve ever read. amusing
→ I’ve never read a………………………………………………………….one before.
2. As soon as I have any more news, I’ll phone you. moment
→ I’ll give…………………………………………………………I have any more news.
3. The previous manager wasn’t running the restaurant well, so he was replaced. badly
→ The restaurant…………………………………………the previous manager, so he was replaced.
4. You got lost because you didn’t do what I told you. instructions
→ If…………………………………………………………you wouldn’t have got lost.
5. I’d lost his phone number so I couldn’t contact him before. touch
→ I couldn’t……………………………………………before because I’d lost his phone number.
6. She really didn’t want to upset him so she kept quiet. avoid
→ She was anxious…………………………………………………………him so she kept quiet.
7. I’ve arranged a trip around Europe next year. arrangements
→ I’ve…………………………………………………………on a trip around Europe next year.
8. He seems not to care about anything. matters
→He acts…………………………………………………………to him.
9. This train should have left an hour ago. meant
→ This train…………………………………………………………off an hour ago.
10. We had a long discussion about the problem but we couldn’t solve it. length
→ We discussed…………………………………………………………but we couldn’t solve it.
II. Topic writing. (1.0 p)
Different people have different ways to express their love for their country. As a student, what are
you going to do to show your love for your country, especially when there are foreign invaders? Give reasons

9
for your actions. Your writing should be between 120 and 150 words.
ĐỀ SỐ 03
(Đề thi vào lớp 10 THPT Chuyên Phan Bội Châu - Nghệ An)
Thời gian làm bài: 120 phút
SECTION A: USE OF ENGLISH
I. Choose the best answer from A, B, C or D to complete following sentences.
1. As a citizen, it is normal to………….. to the laws and rules made by the society.
A. conform B. resist C. hinder D. obey
2. He attempted to…………..the painting from the gallery but he was caught.
A. rob B. steal C. thief D.
kidnap
3. Once the computer virus was removed, a lot of information…………...
A. is disappearing B. will have disappeared C. disappears D. disappeared
4. Telecommuter boosters will often mention…………..convenient it is for people wishing to cut down on
their work hours.
A. how B. therefore C. only D. most
5. A: “Will the Jackson be invited?”-B: “I…………..”
A. expect so not B. so expect C. expect so D. don’t expect
6. I read the contract again and again…………..avoiding making spelling mistakes.
A. with a view to B. on account of C. by means of D. in terms of
7. In a report submitted to the government yesterday, scientists…………..that the building of the bridge be
stopped.
A. banned B. complained C. said D. recommended
8. Anyone ordering a new MP3 player…………..the end of the month will receive a free extra set of
headphones.
A. at B. in C. upon D. before
9. I enjoyed reading that story. It was rather sad,…………...
A. also B. though C. but D. so
10. My job is so…………..that I don’t think I’ll be able to take a summer break
A. persisting B. hard C. demanding D. tough
1. 2. 3. 4. 5. 6. 7. 8. 9. 10.

II. Choose the best answer from A, B, C or D to complete the following passage.
SOUND ADVICE FOR LANGUAGE LEARNERS
A recent survey of a language learning magazine has consulted a number of experts in the (1)……….
of a second language acquisition. Their advice may prove invaluable for those (2) …………..a language
course. One suggestion is that you (3) …………..whether you are likely to be successful at learning a
language? Did you enjoy studying languages at school, for example? The major (4) …………..will be your
own time and effort. Therefore, you must be sure that the course on offer leads to a (5)…………..
qualification. Also, be realistic in your (6) …………... If you don’t set achievable aims, you are more likely
to give up. Do not be deceived (7) …………..thinking that the most expensive courses are the best (8) ……..
around to get the best possible value for money. You should also bear in mind that the faster you learn a
language, the more quickly you forget it. Sandra Miller, a French teacher, tried to teach herself German by
rolling on a (9) …………..course. Already fluent in four languages and with a sound knowledge of teaching
methodology her chances of (10)…………..progress were high. Three years on she remembers very little.
She feels her biggest mistake was not to follow up her first experience. “I should have consolidated what I’d
learnt by continuing to study, even if it were by myself.”
1. A. branch B. field C. area D. domain

10
2. A. considering B. wondering C. thinking D. looking
3. A. survey B. review C. balance D. assess
4. A. price B. charge C. cost D. valuation
5. A. valued B. regarded C. understood D. recognised
6. A.ends B. sights C. goals D. objects
7. A. into B. about C. by D. in
8. A. Nose B. Shop C. Push D. Run
9. A. rapid B. quick C. fast D. crash
10. A. achieving B. making C. doing D. gaining

1. 2. 3. 4. 5. 6. 7. 8. 9. 10.

III. Fill in each numbered space with a suitable word.


SAFE CAMPING
Camping in the country is usually great fun, but sometimes things can go wrong. Accidents can
happen, so it is essential to think about safety (1) …………..before you go and while you are there. This will
prevent your fun camping trip turning into (2) …………..less pleasant.
Firstly, you need to plan ahead. Check out the weather forecast a few days in (3) …………..and
watch out for any reports of fires in the area you are thinking of going to. (4) …………..an emergency kit in
case you or anyone with you has an accident or illness while you are there.
Choose your camp site carefully, avoiding any places (5) …………..there is risk of flooding before
you put up your tent, make (6) …………..there are no sharp objects on the ground, or ants’ or wasps’ nests
nearby.
In order to keep insects out of the tent, close it whenever you go in or out. If you need a camp fire for
cooking, be (7) …………..not to build it anywhere near your tent, and before you go to bed, remember to put
it out completely, preferably (8) …………..lots of water.
After meals, pick up any bits of food that (9) …………..be left on the ground, as these can attract
insects - or larger creatures. It also makes senses, for the same reason, to keep unused food in closed
containers away from the camp. You don’t want a hungry bear or another (10) …………..suddenly appearing
in your tent!
1.………….. 6.…………..
2.………….. 7.…………..
3.………….. 8.…………..
4.………….. 9.…………..
5.………….. 10.…………..

IV. Complete the passage below using the correct forms of the words in brackets.
SECURITY
People are becoming more security conscious these days. Crimes like burglary and (1 .THIEF).…….
are definitely on the increase. One of the most (2. PAIN).………….experiences a home (3. OWN).………….
can have is to arrive home and find that his or her (4. VALUE).………….. have disappeared, because a
window had (5. ACCIDENT).………….been left open. What can we do to protect ourselves?
The most important piece of (6. ADVISORY).………….is to make sure that your (7. INSURE) .
………….coverage is up to date. Another (8. SENSE).…………. thing to do is to go along to your local
police station, where they will be more than willing to make (9. SUGGEST) .…………. on (10. RELY)
.………….ways of safeguarding your property.
1..…………. 6..………….
2..…………. 7..………….
3..…………. 8..………….

11
4..…………. 9..………….
5..…………. 10..………….

SECTION C: READING
I. Read the passage below and choose the best answer from A, B, C or D.
A WALK IN THE MIDDAY SUN
When the heat is on, walkers need to be on their guard.
The heat can create serious heath problems for walkers.
Hot weather makes your heart pump harder, and if you’re not very fit, you start to understand why the
majority of mountain rescue statistics are made up from summer walkers suffering heat attacks. Heat
exhaustion is quite easy to get when you’re making great physical effort. It happens when your body can’t
get enough sweat to keep you cool.
Take enough water and drink it sensibly.
The answer is to keep up your water intake. It’s a good idea to drink a pint of water for every 10
degrees Fahrenheit every 24 hours. So, if the temperature is in the 70s, and you are doing a five- hour walk,
you’ll need a minimum of around one and a half pints of water. It’s vital that you don’t wait until you
develop a raging thirst before you stop for a drink- keep taking regular swigs from your water bottle.
There are several alternatives to just water.
Many walkers flavor their water with fruit juice, which makes it a lot more palatable. You could even
use one of the isotonic drinks made for athletics, which replace the body’s salts lost through sweating.
Powders such as Dioralyte, which you may have in the house as a treatment for diarrhea, will do the job just
as well, as its main aim is also effective rehydration.
Getting wet is one way of keeping temperatures down.
Given that evaporation is your body’s cooling mechanism, you can help things along with an external
application of water. Soaking your hat with water is a great way to cool the head, though if the sun is beating
down, it will probably dry off almost immediately. Better still then if you can plunge into a river or the sea
fully-clothed. And if that’s not possible, then at least take off your boots and socks and paddle in a cool
stream.
One part of your body which can suffer is your feet.
Walking in the heat increases the rate at which your feet swell, which can lead to them feeling tight in
your boots. Cool water from a stream reduces any swelling and helps general foot comfort. At the same time,
you can check out your feet for signs of blisters. Extra sweating makes the skin softer and increases the
chance of blisters forming, in the same way as when water leaks into your boots and gets to your feet.
The wrong clothing can cause problems.
As for what clothing you can wear, this should be lightweight and reasonably loose-fitting. Tight
clothing will feel uncomfortable and may even lead to the formation of an irritating rash as “quickly heat” on
your skin. The answer, if it does develop, is to try and stay cool as much as possible. Do this by either
keeping in the shade, or washing the affected area with cold water, but without soap. But prevention is by far
the best approach, so keep your clothing light.
Your clothing acts as an important defense against the sun.
It’s understandable to want to remove any extraneous clothing when it’s extremely hot, but it doesn’t
really make much sense to take off T-shirts. The sun’s rays can be quite strong, and shoulders are always
very sensitive tosunburn. This is the worst place to be red and sore when you are wearing a heavy rucksack
on your back. Wearing shorts can also create problems for walkers, as the backs of the legs can catch the sun
very easily.
Always protect those parts of your body which are not covered by clothing.
In fact, those days when an apparently harmless breeze is blowing can be the most deceptive. It might
not feel so hot, so you probably won’t notice the damage being done so soon. As on every other day then, a
good strong sun cream should therefore be applied to any skin which is exposed. Make the most use of the

12
summer, but the sun with the respect it deserves.
1. What does the writer say about “Dioralyte ”?
A. It helps to reduce sweating.
B. It prevents the loss of body salts.
C. It works in the same way as an isotonic drink.
D. It will help you get diarrhea.
2. The word “it ” in line 3, paragraph 4, refers to..…………..
A. the sun B. water C. the head D. your hat
3. According to the text, when might your feet suffer?
A. when they cool down.
B. when they are wet
C. if you are wearing tight-fitting boots
D. if you have to walk through water
4. According to the writer, it is better to wear loose fitting clothing because………………
A. it is less likely to create problems for your skin.
B. it is very light
C. it keeps you cool.
D. it lasts longer than tight-fitting clothing.
5. What does the writer mean by “extraneous clothing” in paragraph 7?
A. clothing which is too tight.
B. clothing which is too heavy to wear.
C. clothing which is no longer needed to keep you warm.
D. clothing which most people would consider unusual in hot weather.
6. According to the writer, when are walkers particularly at risk from the effects of the sun ?
A. if they have suffered an injury B. if their sun cream is not strong enough
C. when there is a strong wind D. when they are unaware of the heat
7. Who has the text been written for?
A. people who go walking in the mountains
B. people who go walking in hot weather
C. walkers who are unfit
D. people who only go walking in summer

1. 2. 3. 4. 5. 6. 7.

II. Read the following magazine article about ways of reducing the environmental harm we do. Choose
from the people A, B, C or D.
A. Carla
School student Carla Ruiz lives in a hot country and has become very aware of the need to save water.
“Spring and autumn used to be quite wet, but these days it hardly rains at all”, she says. “Nearly all the rivers
have dried up, destroying all the wildlife in and around them, and no matter what we do they’ll never be the
same again. At least, though, we can use what little water there is more sensibly. That’s why at home I
recently decided to do simple things like making sure there are no dripping taps, or taps left on while I’m
brushing my teeth or washing food; also having showers instead of baths and not overwatering the plants.
Within a few days I was regularly doing these things without even thinking, and I know they made a
difference because the water bills went down quite a bit. My parents noticed that so they started doing the
same, and our bills are now a lot lower”.
B. Vincent
Trainee manager, Vincent Owen, is doing his bit to save the planet by using less electricity around the home.
“I was talking to this guy at work and he told me that we waste a huge amount of energy every year by

13
leaving things like the TV, DVD and computer on standby all the time, so nowadays I try to remember - not
always successfully - to switch them off at night. Something I always do now, though, is keep the air-
conditioning off, even if I get a bit too sweaty here in summer.
Incidentally, I've now got solar panels on the roof so that all the hot water is powered by the sun. That was a
big investment, and it ended up well over budget, but I’m sure it’ll pay for itself in the end. I was hoping the
neighbors might go for solar energy too, but as yet there’s no sign they will.”
C. Lin
While Lin Chen is on a gap year, she is traveling round Europe with friends. “We had intended to fly
everywhere,” she says, “but when we worked out just how much extra pollution that would cause, we
decided to do it by train instead. It was cheaper, too.” They began their tour in Greece: “We all felt the
obvious place to start was where European civilization began, so our first rail journey began in Athens. We
traveled to Patras on the west coast, taking the ferry across to Bari in southern Italy. Unfortunately, it was
very windy and I had a bad case of seasickness, though, by the time we were on the train to Bologna, I’d
recovered. From there we took the overnight train to Paris, and a few days later we went on the Eurostar to
London. We saw far more of the countryside than we would have done by plane, and it was much more
relaxing, too.”
D. Tanya
Tanya Petrov works in a restaurant with an extensive menu, but at home she will only eat local or seasonal
food: “I strongly believe that transporting food thousands of kilometers, or storing it under refrigeration for
months on end, ultimately has a highly negative impact on the climate. I always try to buy food that is
produced locally and I have a special calendar to show me which kinds of food are in season so that I know
what I’m buying is really fresh. And I always check the “best-before” dates of fresh fruit and vegetables
before I choose them so I don’t end up having to throw any out. Apart from the environmental
considerations, I’m convinced the food I eat, which has far fewer chemicals in it, helps me avoid the kind of
illness that seems to be so common these days.”
Which person

1. avoids waste by selecting items carefully?


2. says other people have followed their example?
3. was ill for a short time?
4. sometimes forgets to do something that they feel should do?
5. found it quite easy to change their daily habits?
6. has followed the advice of a colleague?
7. changed their original plans for environmental reasons?
8. sometimes feels physically uncomfortable because of a change they made?
9. says the damage to the environment is a permanent?
10. makes different choices according to the time of year?

1. 2. 3. 4. 5. 6. 7. 8. 9. 10.

SECTION D: WRITING
I. Complete the second sentence so that it has the same meaning as the first one, using the word given
in each bracket. DO NOT CHANGE THE WORD GIVEN. You must use two and five words including
the word given.
1. Joe isn’t feeling very well today. (weather)
→ Joe is feeling a bit……………………………………………………………………………..
2. That apartment is furnished. (some)
→ There……………………………………………………………………………..in that apartment.
3. He had a very traditional upbringing, didn’t he? (traditionally)

14
→ He was……………………………………………………………………………..,wasn’t he?
4. It was wrong of you to allow a 4 year-old child to walk home alone. (shouldn’t)
→ You……………………………………………………………4 year-old child to walk home alone.
5. Success depends on hard work. (more)
→ The harder……………………………………………………………………………..you are.
II. Write an essay about 180 words on the following topic:
“Helping the poor in Vietnam is everyone’s responsibility. What should students do to help the poor? ”

ĐỀ SỐ 04
(Đề thi vào lớp 10 THPT Chuyên Long An)
Thời gian làm bài: 120 phút
I. READING (2 points)
PART 1: Read the following passage, then choose the answer (A, B, C or D) which you think fits best
according to the text. Write your answer in the numbered box on your answer sheet. (1 point)
Dinosaurs were reptiles that lived during a period of earth's history called the Mesozoic Era, which is also
known as the Age of Reptiles. The first dinosaurs appeared more than 200 million years ago. For many
millions of years, they dominated the land with their huge size and strength. Then about 65 million years ago,
they died out rather suddenly, never to re-emerge.
The word “dinosaur” comes from two Greek words meaning “terrible lizard”. Dinosaurs were not lizards, but
their appearance could be truly terrifying. The biggest ones weighed more than ten times as much as a mature
elephant and nearly equaled the size of most modern-day whales. The famous kinds of dinosaurs, including
the brontosaur, and tyrannosaurus rex, reached 80 to 90 feet in length. Not all dinosaurs were giants,
however; some were actually no larger than a chicken.
Scientists still do not know what caused dinosaurs to disappear. One theory involves a change in the earth's
climate. It is believed that temperatures dropped significantly towards the end of the Cretaceous Period. Too
large to hibernate and not having fur or feathers for protection, it is possible that the climate became too
chilly for dinosaurs. In contrast, other species having protection, such as the mammals and birds, were able to
survive.
1. What is the best title for this passage?
A. The History of Earth B. Earth's Largest Reptiles
C. The Metabolism of Dinosaurs D. The Domination of the Land
2. It can be inferred from the passage that the Age of Reptiles lasted about……………
A. 65 million years B. 80 million years
C. 135 million years D. 200 million years
3. In line 4, the author uses the phrase "never to re-emerge" to indicate that the dinosaurs…………….
A. went into hiding B. became extinct C. lost their way D. never died out
4. According to the passage, what is true about the size of dinosaurs?
A. It was rather uniform.
B. It guaranteed their survival.
C. It varied quite greatly.
D. It made them the largest creatures ever on earth.
5. The paragraph following the passage most likely discusses……………
A. another theory about the disappearance of dinosaurs
B. other changes in the climate
C. the ability of mammals to survive
D. the protection of other species

15
PART 2: You are going to read a newspaper article about a schoolboy who set up a successful Internet
business. Five sentences have been removed from the text. Choose from the sentences A-F the one
which fits each gap (1-4). There is one extra sentence which you do not need to use. There is an
example at the beginning (0). Write your answer in the numbered box on your answer sheet. (1 point)
MY SON’S A COMPUTER GENIUS
Tom Hadfield created a $15 m Internet business by the age of 16. His father describes life with a child
prodigy. Tom was always advanced for his age. He learned to walk and talk early, and was fascinated by
words and numbers. He was only two when he got his first computer. In fact, it was bought for his older
sister. Even before his aptitude for mathematics became apparent, Tom was teaching himself to play chess on
it. (0) F.
Although Tom’s sister attended the local primary school, Tom never quite settled in at the neighbourhood
playground. Instead, he preferred to play by himself at home. Early pastimes included shuffling and
memorizing the order of two packs of playing cards, or working out how many seconds there are in a day, a
week, a year.
We hoped that school would keep him occupied, but although he enjoyed the many friends he made, he soon
grew bored with lessons. It wasn’t the teachers’ fault. (1)……………. What about his friends? What would
happen when he was in the top year? The proposed solution raised more problems than it tried to solve.
By the time he was seven or eight, Tom was playing football regularly for his school and for a team
organized by supporters of the local football club. His skills as a businessman had begun to show through as
well. He washed our car, and those of the neighbours, for 50p each. (2)……………. He preferred to invest it
in a bucket, sponge and bottle of car shampoo.
Secondary schools began and Tom soon discovered the World Wide Web. For his twelfth birthday, we got
him “wired up,” largely because this was the only way to get him home from the house of a friend who
already had Internet access. (3) ……………. We learned later that Tom was already planning then to
postpone serious studying until the year he took his final exams at 16.
Tom says that he drew up the business plan for Soccernet, now the world’s most popular football website,
while he was daydreaming in a lesson. Since then, its success has provided his father with full-time
employment, generated millions of dollars in advertising, and now attracts more than seven million readers a
year. The site is now valued at $ 15m. (4) …………… He has also had the chance of leaving school to
pursue a football career as a goalkeeper with Brighton F.C. or stay at school and follow a business career.
A. But he didn’t want to spend the money on sweets.
B. His other interests haven’t harmed his education; they have added to it, made it more meaningful.
C. They suggested the possibility of moving him up a year but that was no answer.
D. From that day, school began to recede further and further into the background.
E. As a result, Tom has been offered employment around the globe, frequently by corporations that did not
realize that he is still a schoolboy.
F. By the age of three, he had learned how to break into the program and change sides every time the
computer was about to declare checkmate.
II. USE OF LANGUAGE (4 points)
PART 1: Choose the word or phrase that best completes each sentence. Write your answer A, B, C, or
D in the numbered box on your answer sheet.
1. He seldom goes to the market,……………?
A. does he B. doesn’t he C. is he D. isn’t he
2. The textbook……………to my teacher has just been stolen.
A. belonged B. belonging C. belongs D. belong
3. Margaret didn’t remember what I……………her the day before.
A. would tell B. told C. had told D. have told
4. I wonder……………
A. where are my glasses B. the place of my glasses

16
C. where did I leave my glasses D. where I left my glasses
5. Students in hostels are……………to keep their rooms clean and tidy.
A. hoped B. expected C. wanted D. desired
6. He was made……………a fine.
A. pay B. paying C. to pay D. paid
7. If……………sees Sophie, can they give her a message?
A. anyone B. anything C. anywhere D. any
8……………….idea was it to visit the exhibition?
A. What B. Where C. Who D. Whose
9. ………………he had won a million pounds, he refused to give up work.
A. Even though B. Despite C. However D. As though
10. I’m sure they were……………lies!
A. speaking B. talking C. saying D. telling
PART 2: Read the following passage and decide which answer (A,B,C, or D) best fits each gap. Write
your answer in the numbered box on your answer sheet. (1 point)
The wind controls our planet's weather and climate. But how much do we understand about this complete
force (1) ………………can kill and spread fear?
On the night of October 15, 1987, the south of England was (2) ………………by strong winds. Gusts of over
130 km/h (3) ………………through the region. Nineteen people were killed, £1.5 billion worth of damage
was (4) ………………and 19 billion trees were blown down in just a few hours.
Although people thought of this (5) ………………hurricane, the winds of 1987 were only a (6) ……………
7 storm. They remain far better known than much more serious storms of January 25, 1990, (7)……………
most of Britain was hit by daytime winds of up to 173 km/h. On this occasion, 47 people were killed, even
though, (8) ………………in 1987, the weather forecasters issued accurate warnings.
Extreme weather events such as these are dramatic (9) ……………… of the power of the wind. It is one part
of the weather that people generally do not give a second (10) ………………to, but across the world the
wind plays a crucial role in people's lives.
1. A. what B. which C. when D. where
2. A. attacked B. besieged C. struck D. beaten
3. A. ran B. blew C. flew D. spread
4. A. caused B. created C. resulted D. paid
5. A. like B. unlike C. same as D. as
6. A. strength B. length C. power D. force
7. A. until B. why C. when D. while
8. A. when B. like C. unlike D.such as
9. A. recalls B. remains C. memories D. reminders
10. A. thought B. think C. care D. help
PART 3: Fill each blank with ONE suitable word. Write your answer in the numbered box on your
answer sheet. (1 point)
Elvis Aaron Presley, Elvis Presley, or the King of Rock ‘N’ Roll was (1) ………………in Mississippi, in
1935. (2) ………………family moved to Memphis, Tennessee, in 1948, and Elvis graduated from high
school in 1953. Elvis’ musical influences were the pop and country (3) ………………. As a Memphis
teenager, he was influenced (4) ………………the gospel music in church and the R&B. Elvis began his
singing career with the Sun Records in Memphis in 1954. In late 1955, his recording contract was sold to
RCA Victor. By 1956, he was (5) ……………… international sensation. He began a whole new era (6)
……………. American music and popular culture with his uniquely combined sound and style which
challenged the social and racial barriers of the time.
Elvis Presley starred (7) ………………33 successful films. He was acclaimed through his record-breaking,
live concert performances on tour and in Las Vegas. He has sold over one billion records, more (8) ……….

17
any other artist. His American sales have earned him gold, platinum, or multi-platinum awards. He got 14
Grammy nominations and won 3 awards from the National Academy of Recording Arts & Sciences. He got
the Grammy Lifetime Achievement Award at the age of 36, and was named One of the Ten Outstanding
Young Men of the Nation for 1970 by the United States Jaycees. He is regarded (9) ………………one of
the (10) ……………… important figures of twentieth century popular culture. Elvis died at his Memphis
home, Graceland, in 1977.
PART 4: For questionsl-10, give the correct form of the words in brackets. Write your answer in the
numbered box on your answer sheet. There is an example at the beginning (0) (1 point)
SAILING AWAY
One Sunday morning Aunt Emily made an (0. ANNOUNCE) announcement. She told us (1. HAPPY) ……..
that she was going to take us on a cruise! I was surprised, knowing how (2. EXPENSE)……………a holiday
like that could be. We weren’t a (3. WEALTH)……………family, but we had put some money aside over
the years, so in the end we used some of our (4. SAVE) …………… for the holiday.
When the day of our (5. DEPART)……………finally came, we were delighted and thrilled to see how huge
and (6. LUXURY)……………the ship looked. Our cruise liner sailed elegantly out to sea and our holiday
began. But it was such a(n) (7. DISAPPOINT)……………!
There was so little to do on board. The (8. BORE)……………almost drove us mad. We visited several ports,
but we didn’t have the (9. FREE) …………… to do what we wanted. We had to follow a very tight schedule
of guided tours and visits to museums. It was a (10. DISASTER)……………holiday!
PART 5: For questions 1-5, complete the second sentence so that it has a similar meaning to the first
sentence, using the word given. Do not change the word given. You must use between three and six
words, including the word given. Write your answer in the numbered box on your answer sheet.
1. It's not worth asking the manager for the day off. POINT
→ There…………… the manager for the day off
2. Our house is going to be painted by a local firm. HAVE
→ We are........................................... by a local firm.
3. George made the same number of mistakes as Peter. AS
→ George ........................................... Peter.
4. “You broke that window Tom,” said Mr Smith. ACCUSED
→ Mr. Smith……………………..the window.
5. I only learnt to drive because you taught me. NEVER
→ I ……………………..drive if you hadn’t taught me.
For questions 6-10, finish each of the sentences in such a way that it means exactly the same as the
sentence(s) printed before it. Write your answer io the numbered box on your answer sheet.
6. Five is an odd number. Seven is an odd number, too.
→ Both…………………………………………………………………………………………….
7. It is a three-hour climb to the top of the hill.
→ It takes …………………………………………………………………………………………
8. I think that this law should be abolished.
→ I think they ……………………………………………………………………………………..
9. It's the first time he’s ever seen a skyscraper.
→ He has…………………………………………………………………………………………..
10. The sun is one of millions of stars in the universe. It provides us with heat and light.
→ The sun, ………………………………………………………………………………………..
III. WRITING (2 points)
Describe a favorite member of your family. Compare yourself with him/her using appropriate language for
describing people and making comparisons.
You should write about 120-180 words and include sections on:
- Physical appearance

18
- Personality
- Hobbies

B. ĐỀ THI NĂM 2015 - 2016


ĐỀ SỐ 05
(Đề thi vào lớp 10 THPT Chuyên Long An)
Thời gian làm bài: 120 phút
PART 1: GRAMMAR AND VOCABULARY (5 POINTS)
I. Supply the correct tense o f the verbs in parentheses. (2 points)
I come from a very large family, and recently my parents decided that they (1) ………….. (spend)
long enough living in an overcrowded house in Birmingham. “We (2)…………..(move) to the country,” my
father (3)…………..(announce) one evening. So last week we loaded all our possessions into two hired
vans, and for the last few days we (4)…………..(try) to organize ourselves in our new home. Yesterday, for
example, my three brothers and I started painting the downstairs rooms. Unfortunately while I (5)…………..
(mix) the paint, one of my sisters opened the door. Nobody (6)…………..(tell) her that we were in the
room, you see. So instead of painting the walls, we spent all morning cleaning the paint off the floor. This
morning when I (7)…………..(wake) up, water (8)………….. (drip) through the ceiling next to my bed. We
(9)…………..(spend) today so far repairing the roof. It’s not all bad news, though. The school in the village
nearby closed down two years ago, and my parents (10) …………..(not find) another school for us yet.
II. Fill in each blank with an appropriate preposition. (2 points)
1) It was careless……………..him to leave the door open last night.
2) Wait …………….. the end of the street and I’ll come and meet you.
3) I’ll never forgive you……………..what you’ve done!
4) Picasso has been a huge influence……………..me as a painter.
5) Michael is absorbed……………..his work and didn’t notice me coming.
6) My uncle is known …………….. his generosity.
7) Would you like to contribute……………..our campaign fund?
8) Admission to this club is open …………….. people under 30 only.
9) We were lucky to be able to finish the project ahead……………..schedule.
10) Don’t blame the theft …………….. Tim. He didn’t steal anything.
III. Use the correct form of the words given in brackets. (1 point)
1. The advantage of living in the countryside is that the air is…………….. (POLLUTE)
2. Children normally feel a lot of……………... about their first day of school (ANXIOUS)
3. The police were unable to……………..that she had committed the crime. (PROOF)
4. You are never too old to go to college and gain some ……………..(QUALIFY.)
5. The fans waved……………..as the film star stepped out of the limousine. (EXCITE)
PART 2: READING COMPREHENSION (6 POINTS)
I. Read the following passage and choose the best option (A, B, C or D) to complete the blank or
answer the question . (2 points)
Craigie Aitchison
The painter Craigie Aitchitson was born in Scotland. He came to London intending to study law, but went to
art school instead. There he found the traditional drawing classes difficult, but still kept on painting.
In his late twenties, he was given money by the Italian government to study art, and became interested in
early Italian artists, which shows in some of his work. He loved the greens and browns of the Italian fields
and the clear light there, and wanted to put this light into his paintings.

19
This led him to paint colors thinly one on top of another from light to dark, but he insists he’s never sure
what the results will be. He says, “It’s a secret - because I don’t know myself. I don’t start by painting
yellow, knowing I’m going to put anything on top.” Like most talented people, Atchison makes it sound
easy. “Anyone can do the colors - you can buy them. I simply notice what you put the color next to.”
Unlike some artists, he never does drawings before he starts a painting, as he feels that if he did, he might get
bored and not do the paintings afterwards. Instead, Aitchison changes his paintings many times before they
are finished. This explains why his favorite models are people who don’t ask to see their pictures while he’s
painting them. “If I feel they’re worried and want to look at the painting, I can’t do it.”
Since moving to London years ago, he has not felt part of the Scottish painting scene. He says he is not
interested in following any tradition, but just paints the way he can. However, his work still influences young
British painters.
1. What is the writer trying to do in the text?
A. describe popular works by Craigie Aitchison
B. teach readers how to paint like Craigie Aitchison
C. introduce readers to the artist Craigie Aitchison
D. explain how Craigie Aitchison has made money from painting
2. What can the readers learn about Aitchison from the text?
A. He works in a different way from other artists.
B. He often gets bored with his paintings.
C. He improved his drawing by going to art school.
D. He did some paintings for the Italian government.
3. What does Aitchison say about his use of color?
A. He likes starting with the darkest colors first.
B. He knows the colors he’s aiming for when he begins.
C. He prefers to paint with yellows, greens, and browns.
D. He understands how different colors work together.
4. Aitchison prefers models who don’t…………….
A. keep talking to him while he’s working
B. ask him about his strange method of working
C. worry about how long the work will take
D. feel anxious to see the work as it’s developing
5. What might a visitor at an exhibition say about Aitchison’s work?
A. I love his recent paintings of Scotland, which are very similar to a number of other Scottish painters.
B. You can still see the influence of his trip to Italy in some of these pictures.
C. You can tell he spent a lot of time drawing the picture before he started painting.
D. I wonder if his law training helps him at all, especially in selling his work .
II. Read the text below and fill in each of the blanks with ONE suitable word (4. points)
“The moon Trap” is a new film made by a young Canadian (1) …………… called Melvin Strang. The main
parts in the film are (2) …………… by Sid Cheung and Julie Plein who last appeared in “Music For Ever”.
In this new film (3) ……………star as a young married couple who buy an old house in the country. After
living there (4) ……………a few weeks, strange things begin to happen. Some of the furniture in the house
disappears and can’t be (5) ……………; windows break and pictures fall off walls. At night they (6) ………
crying noises, and when the moon is up loud screams can be heard from the woods nearby.
As you might expect, the young couple try to discover the (7) ……………; for all these strange events and
this leads them into some very frightening situations. (8)……………you are easily scared, don’t go and see
this film! But if you enjoy film with (9) ……………of adventure (10) ……………this is the film for you.
PART 3: WRITING (5 POINTS)
I. Sentence transformation (1 point)
Complete the second sentence so that it has a similar meaning to the first sentence, using the word

20
given. DO NOT CHANGE THE WORD GIVEN. YOU MUST USE BETWEEN TWO AND FIVE
WORDS, INCLUDING THE WORD GIVEN.
Example:
0. It's three weeks since we last went out. BEEN
We……………three weeks.
The gap can be filled by the words “have not been out for”, so we write:
0 have not been out for
1. Take my advice and don’t believe everything he says. WERE
→ If I……………believe everything he says.
2. I 'd rather you didn't use that red pen. MIND
→ Would……………that red pen.
3. On arriving at the restaurant, we went straight to the head waiter. SOON
→ We went straight to the head waiter……………at the restaurant.
4. Experts are repairing the van. HAVING
→ We……………by experts.
5. Clothing costs the same as it did last year. CHANGED
→ Clothing prices……………last year.
II. Writing (4 points)
Write a paragraph (at least 90 words) to describe dinner time in your family.
……………………………………………………………………………………………………………
……………………………………………………………………………………………………………
……………………………………………………………………………………………………………
……………………………………………………………………………………………………………
……………………………………………………………………………………………………………
……………………………………………………………………………………………………………
……………………………………………………………………………………………………………

ĐỀ SỐ 06
(Đề thi vào lớp 10 THPT Chuyên Đại học Sư Phạm Hà Nội)
Thời gian làm bài: 120 phút
Mã đề số: 210
Choose the word whose underlined part is pronounced differently from that of the others.
Question 1:
A. laughter B. caught C. daughter D. augment
Question 2:
A. incline B. eradicate C. exacerbate D. enclosure
Question 3:
A. friends B. opinions C. picnics D. computers
Question 4:
A. chimpanzee B. interviewee C. refugee D. committee
Question 5:
A. politeness B. conversation C. resolution D. introduction
Choose the best answer to complete each of the following sentences.
Question 6: I'd love to have lived in the old days, when people…………….to market by horse and carriage.
A. have been travelling B. got used to travelling
C. would travel D. had been travelling
Question 7: I'll give you the phone number of my hotel so that you can reach me if anything happens

21
…………….anything happen. I want you to look after my children.
A. Can B. Might C. Will D. Should
Question 8: We’ve been together through…………….in our friendship, and we won’t desert each other now.
A. bad and good B. thick and thin C. odds and ends D. spick and span
Question 9: …………….a scholarship, I entered the frightening and unknown territory of private education.
A. To award B. Having awarded
C. To be awarded D. Having been awarded
Question 10: Millions of people say Coke tastes best from a bottle, and whether this is scientifically provable
or not. These millions know they like the look of the bottle and the way it fits so…………….into the hand.
A. neatly B. orderly C. tidily D. finitely
Question 11: Language is so…………….woven into human experience that it is scarcely possible to imagine
life without it.
A. tightly B. tautly C. rigidly D. stiffly
Question 12: …………….I'd like to help you out, I'm afraid I just haven't got any spare money at the
moment.
A. Much as B. Try as C. Even D. Despite
Question 13: She was very badly depressed after the car accident. Now she is beginning to think that there
could be light at the end of the…………….
A. tunnel B. subway C. passage D. journey
Question 14: The government must…………….strong measures against crime.
A. be seen be taking B. see to be taking
C. be seen to be taking D. seen to take
Question 15: Athough she would have preferred to carry on working, my mum…………….her career in
order to have children.
A. devoted B. aholished C. repealed D. sacrificed
Question 16: A number of landslides have…………….Nepal since the 7.8-magnitude earthquake on 25th
April which killed more than 8,000 people and injured many more.
A. stroke B. caused C. hit D. blown
Choose the word whose primary stress is placed differently from that of the others.
Question 17: A. compliment B. excellent C. nominate D. distinguish
Question 18: A. remember B. quality C. occasion D. terrific
Question 19: A. worldclass B. wheelchair C. firewood D. blackmail
Question 20: A. volunteer B. referee C. spiritual D. recommend
Question 21: A. influential B. accessible C. rudimentary D. incidental

Read the passage and choose the best answer to each of the questions that follow.
GETTING THE BEST OUT OF OUR CHILDREN
There is a strange paradox to the success of the Asian education model. On the one hand, class sizes are huge
by western standards with on average between 30 and 40 students per class in countries like Japan and Korea.
On the other hand, school children in developed Asian economies rank among the highest in the world for
academic achievement in the areas of science and mathematics, especially on standardised tests. Meanwhile,
British secondary school students fail to shine in conditions most educational researchers would say are far
more likely to help them succeed.
Why do Asian students seem to perform so well then? Is it their legendary discipline? Certainly, classroom
management seems to be a whole lot easier in places like Korea, and perhaps lessons are more effective as a
direct consequence. After all, we are only too aware of the decline in discipline standards in our own schools;
belligerent and disrespectful students appear to be the norm these days. Teachers in Britain seem powerless
to control what happens any more. Surely this situation cannot create a very effective learning environment,
so perhaps the number of students is far less relevant than is the manner in which they conduct themselves.

22
But there are other factors to consider, too. Korean students spend a lot more time with their teachers. It
seems logical to suggest, therefore, that they might form stronger bonds and greater trust, and that Korean
teachers, in understanding their pupils better, might be able to offer them a more effective learning
programme. Of course, trust and understanding leads to greater respect as well, so Korean students are
probably less likely to ignore their teachers’ advice.
Then there is the home environment. The traditional family unit still remains relatively intact in Korea. Few
children come from broken homes, so there is a sense of security, safety and trust both at home and at school.
In Britain meanwhile, one in every two marriages fails and divorce rates are sky-high. Perhaps children
struggle to cope with unstable family conditions and their only way to express their frustration is by
misbehaving at school. Maybe all this delinquent behaviour we are complaining about is just a cry for help
and a plea for attention.
But while the Japanese, Korean and Asian models generally do seem to produce excellent results, the
statistics don’t tell the truth. Asian students tend to put their education before literally everything else. They
do very few extracurricular activities and devote far more time to their studies than their British peers. And
this begs the question: is all that extra effort justified for a few extra percentage points in some meaningless
international student performance survey? So Asian students are on average 3-5% better at maths than
Britons - big deal! What is their qualily of life like? Remember, school days are supposed to be the best; are
they not?
There has been a lot of attention and praise given to these Asian models and their impressive statistics of late.
And without question, some of this praise is justified, but it seems to be a case of two extremes in operation
here. At one end, there is the discipline and unbelievably hard work ethic of the Asian students success in
education before all else. At the other end, Brilish students at times appear careless and extremely
undisciplined by comparison, but at least they DO have the free time to enjoy their youth and explore their
interests. Is either system better outright? Or is it perhaps about time we stopped comparing and started
trying to combine the best bits of both, so that we can finally offer our students a balanced, worthwhile
education? We are not just dealing with statistics; never forget that every statistic is a little human being
somewhere who desperately needs our help and guidance - who deserves it.
Question 22: What does the writer mean when he says there is a paradox in the Asian education model?
A. There are too many students in each class.
B. Larger classes are expected to lead to poorer results but they do not.
C. Asian students outperform their peers in other countries.
D. Class sizes in Asia are much smaller in other parts of the world.
Question 23: British secondary school students........
A. do better on standardised tests B. have larger class sizes
C. fail at school more than they succeed D. enjoy better classroom conditions
Question 24: What does the writer suggest might make lessons in Korean schools more successful than in
Britain?
A. stricter classroom discipline
B. better school Boards of Management
C. more effective lesson planning
D. better teachers
Question 25: What can be inferred from the utterance perhaps the number of students is far less relevant than
is the manner in which they conduct themselves?
A. Class size does not affect student performance.
B. Class size is important to maintaining control.
C. How students behave might be more important than class size.
D. How teachers conduct classes affects student performance.
Question 26: The traditional family unit.......
A. is unstable in Korea due to conditions in the home

23
B. is bad for children that come from broken homes
C. is disappearing in Korea due to high divorce rates
D. is more common in Korea than in Britain
Question 27: Look at the following sentence.
You see, behind those great maths and science scores, there is a quite remarkable work ethic.
Where does the sentence best fit in the paragraph?
But while the Japanese, Korean and Asian models generally do seem to produce excellent results, the
statistics don’t tell the truth. [1] Asian students tend to put their education before literally everything else.
They do very few extracurricular activities and devote far effort more time to their studies than their British
peers, [2] And this begs the question: is all that extra effort justified for a few extra percentage points in some
meaningless international student performance survey? So Asian students are on average 3-5% better at
maths than Britons - big deal! [3] What is their quality of life like? Remember, school days are supposed to
be the best, are they not? [4]
A. [4] B. [3] C. [2] D.[l]
Question 28: According to the writer, Asian students ........
A. don’t allow themselves much time to relax and have fun
B. don’t perform well in schools
C. don’t have as good a work ethic as British ones
D. make a big deal of their good results
Question 29: What are the 'too extremes' mentioned in the last paragraph?
A. good discipline and a hard work ethic
B. success and failure
C. carelessness and indiscipline
D. neglecting school and neglecting free time
Question 30: Which conclusion about the two educational systems discussed would the author most
probably agree with?
A. Neither system is perfect.
B. Both systems are quite satisfactory.
C. The Asian system is obviously better.
D. The British system is too strict.
Read the text below and decide which answer best fits each gap. The first one has been done as an
example (0).
Example: (0). A . beginning B. first C. primary D. basic
VOLUNTEERING
When Pamela Janett left university to become a (0)……………school teacher, it was by no (31)
………… easy to find a job. She therefore decided to go abroad as a volunteer teacher for a year. When she
realized she would be teaching deaf and blind children, she was a (32)……………..taken aback. But after a
month's training she felt more confident that she would be able to cope with the situation. The basic living
conditions also (33) ……………..as something of a shock.
Pamela's school was situated in a remote Ethiopian village, where her accommodation (34) …………
of one room and a shared bathroom. Not only was the space cramped, but there was no electricity, Internet or
telephone access, so she felt totally cut (35) …………….. from the outside world. Now back home in
Britain, Pamela has used her (36)……………experience to set up a similar school for the deaf and blind
which has made a huge (37) ……………..to dozens of children who would (38) ……………..find
themselves struggling to learn. She is now concentrating her efforts on (39) ……………..her school to cater
for children with other learning difficulties, too. It seems as if the more people get to know her, the (40)
……………...the demand is for her skills.
Question 31: A. means B. extent C. ways D. account
Question 32: A. rather B. quite C. little D. somewhat

24
Question 33: A. appeared B. came C. proved D. arrived
Question 34: A. combined B. consisted C. contained D. composed
Question 35: A. down B. away C. out D. off
Question 36: A. worthy B. valuable C. priceless D. precious
Question 37: A. contrast B. difference C.change D. transformation
Question 38: A. furthermore B. nonetheless C. otherwise D. however
Question 39: A. growing B. widening C. expanding D. stretching
Question 40: A. greater B. larger C. steeper D. taller
Put each verb in brackets into a suitable tense or form. The first one has been done as an example (0).
Example : (0) took
LEARNING TO SURVIVE
Last summer I (0. take)……………a three-week survival course. The purpose of the course was (41. teach)
……………us how to survive outdoors, where there are no shops, no houses and no electricity. I had never
slept outdoors before the course, and here I was (42. learn)……………to make a fire, navigate and find food
in the forest. The part that I (43. like)……………the most was catching our own fish and cooking them over
a fire. While (44. do)……………the course, I realised how much people (45. depend)……………on
modern technology. They think that they can't survive without it. but they can if they (46. learn)………how.
I did! I (47. never forget)……………that course - it was the greatest experience I (48. ever have)………..
so far. Now I think if I (49. not take)……………the survival course, I (50. not be)……………able to deal
with so many difficult situations in life.
Use the word given in brackets to form a word that fits in the space. The first one has been done as an
example (0). Example: (0.) likely
MODERN CULTURE?
When people talk about contemporary culture they are just as (0. LIKE)……………to be talking
about fast cars, trainers or high heels as they are to be talking about Shostakovich or Shakespeare.
Goods have become as (51. MEAN)……………a measure and marker of culture as the Great and the
Good. The word "culture" can now cover just about anything, culture is no longer merely the beautiful and
(52. SINGLE)……………until the late twentieth century that a (53. SCHOLAR)……………interest in
objects began to (54. PLACE)……………the traditional interest in -isms, with historians,(55.LITERATE)
……………critics and philosophers all suddenly becoming fascinated by the meaning of objects, large and
small. Is this a sign, perhaps, of a society cracking under the strain of too many things?
Our current (56. OBSESSIVE)…………… with material culture, one might argue, is simply a (57.
RESPOND.)……………to the Western crisis of abundance. There are obvious problems with this (58.
MATERIAL) …………… conception of culture. If our experience of everyday life is so (59 SATISFY)
……………,then how much more so is the (60. SPECTATOR)……………of our everyday things under
scrutiny.
Read the text below and think of the word which best fits each space. Use only ONE word in each
space. The first one (0) has been done as an example.
Example: (0) with
When presented (0) …………… the idea of international boarding school, parents are (61) …………
to baulk of emptying their bank (62)……………to send their darlings to live a life permeated by
blackboards, bunk beds and Bovril. (63)……………conversely, will feel overjoyed at the prospect of signing
(64)……………for a literally fantastic life at Hogwarts. Such misconceptions neatly miss the point of
international boarding education, sidestepping its capacity to (65) ……………solid foundations for students'
academic and professional success in today’s increasingly global society.
Parents, understandably, require justification for (66)…………… a considerate amount of money in
their offspring's education. Their offspring, on the other hand, should be made (67)…………… of the huge
number of opportunities provided by boarding school life. Most of the world's leading boarding schools offer
stimulating, progressive (68)……………which prioritizes their students' needs, thus consistently produce

25
graduates (69)……………are confident learners, critical thinkers and self starters.
Therefore, the time has come for parents and students (70)……………to discover the unique,
intellectually challenging experiences offered by intemalional boarding schools worldwide.
Complete each of the following sentences with one of the phrasal verbs give. in the box. Each phrasal
verb is used only ONCE. Make any necessary changes.
put up go up come off make up for call for
come up drop out of set up take up grow out of
Question 71. The hotels were all full so we offered to…..Carla….for the night.
Question 72. Early that morning, we set off on our journey as the sun was…………….
Question 73. An enquiry was…………….into the use of chemicals in farming.
Question 74. You’re such a good singer that you, should………it.......... professionally.
Question 75. How are they going to…………….the time they wasted playing cards in the barracks?
Question 76. Stop wasting your time. The whole situation............... an immediate response that could bring
more decisive effects.
Question 77. I like this photograph so much that I am going to have it…………….
Question 78. As far as I know, the idea of the party does not quite appeal to him and that's why he's thinking
of...................
Question 79. I can't stop thinking there's something more that the scheme needs..................
Question 80. Peter used to be fond of collecting mascots, but after his military service, he..............the hobby.
Complete the second sentence using the word given so that it has a similar meaning to the following
sentence. Write between two and five words in the space provided on the answer sheet. Do NOT
change the word given in brackets in any way.
Question 81. That historian is famous for his vast knowledge of primitive life. (AUTHORITY)
→ He is................................................................................................................................................
Question 82. Nobody could possibly believe the story about her achievements. (BEYOND)
→ The story about her achievements .................................................................................................
Question 83. I couldn't make sense of the radio message because of the interference. (IMPOSSIBLE)
→The interference on the radio………………………………………to make sense of the message.
Question 84. They arrived at the station with only a minute to spare. (NICK)
→ They arrived at the station........................................................................................................time.
Question 85. David praised her exceptionally good choice of venue for the party. (CONGRATULATED)
→ David.................................................................................................a good venue for the party.
Complete the unfinished sentence in such a way that It means nearly the same as the sentence printed
before it.
Question 86. Were Jack not so affluent a man, she would not be dating with him.
→ But...........................................................................................................................................
Question 87. It is likely that they forgot about the extra class.
→ They........................................................................................................................................
Question 88. Do you have any scarves? I'm looking for one that's woolen, green and fairly long.
→ I'm looking for a......................................................................................................................
Question 89. You should not lock this door for any reason when the building is open to the public.
→ Under no.................................................................................................................................
Question 90. A bee sting is more likely to cause death these days than a snake bite.
→Death.......................................................................................................................................
Write a paragraph of about 150 words about the benefits of studying at a gifted school.
.....................................................................................................................................................
.....................................................................................................................................................
.....................................................................................................................................................
.....................................................................................................................................................

26
.....................................................................................................................................................
.....................................................................................................................................................
.....................................................................................................................................................

ĐỀ SỐ 07
(Đề thi vào lớp 10 THPT Chuyên Bắc Giang)
Thời gian làm bài: 120 phút
I. Choose the correct answer by circling A, B,C or D to complete the sentences. (5 pts)
1. Joan: “Our friends are coming………….., Mike?”
Mike: “I'm sorry, but I can’t do it now.”
A. Shall you make some coffee, please
B. Would you mind making some coffee
C. Why don’t we cook some coffee
D. Shall I make you like some coffee
2. Welcome to the interview. Please, take a…………...
A. seat B. look C. cup of coffee D. bow
3. We went to the restaurant…………..there was no food in the house.
A. although B. however C. because D. but
4. Barcelona football club…………..Champions League history by defeating Juventus 3-1 in the final match
last Sunday to have the cup for the 5th time.
A. marked B. done C. won D. made
5. Some designers have…………..inspiration from Vietnam's ethnic minorities to change the traditional ao
dai.
A. made B. brought C. taken D. done
6. She is one of the few people…………...
A. I look up to them B. to who I look up
C. to that I look up D. to whom I look up
7. According to a recent survey, most people are on good…………..with their neighbors.
A. relations B. acquaintance C. terms D. relationships
8. Bien Cuong, a commentator, came…………..a lot of criticism for his speech in a match of the 2015 SEA
Games.
A. in for B.over C. out of D. off
9. I have bought a…………..car.
A. red small French B. small French red
C. French small red D. small red French
10. The bomb…………..in the street; fortunately no one hurt.
A. went up B. went off C. went by D. went out
II. Give the correct tense or form of the verbs in brackets to complete the following sentences. (5 pts)
1. Were it to rain heavily, we (stay)…………..at home. 1.…………..
2. I do not understand why Hue (always forget)…………..her umbrella. 2.…………..
3. His mother, together with her friends, (just arrive)…………..at the station. 3.…………..
4. Not only Tom (visit)…………..us but he also brought us many presents. 4.…………..

27
5. There is no point in (complain)…………..about your son if you can do nothing 5.…………..
(help)…………..him.
6. It was my fault to keep you waiting so long. I (inform)…………..you in advance. 6.…………..
7. The virus (know)…………..as MERS has spread enough for the WHO to say the 7.…………..
situation is now more serious and urgent.
8. Taylor Swift (name)…………..among Forbes' 2015 list of the 100 Most Powerful 8.…………..
Women in the World recently.
9. They suggest that the heater (repair) ………….. before winter comes. 9.…………..

III. Fill in each blank in the following sentences with a proper preposition. (5 pts)
1. Hundreds…………..people died although scientists had warned them…………..the eruption of Mount
Pinatubo.
2. The ao dai consists of a long silk tunic that is slit on the sides and worn…………..loose pants.
3. Before Tet holiday, I like to clean…………..my house and decorate the living room…………..flowers.
4. I’m afraid I will be…………..debt soon because I’m extremely pressed…………..money these days.
5. My teacher divided the class…………..three groups ………….. ten students.
6. Paula applied for the post but she was turned…………...
IV. Give the correct form of the given words to complete the text below. (5 pts)
1. All necessary…………..for the 28th SEA Games in Singapore have been completed so far. (PREPARE)
2. Unfortunately, the film got…………..reviews. (FAVOR)
3. A recent survey has…………..revealed that home computers do not…………..relationships within
families but bring children and parents together. (SURPRISE - WEAK)
4. Teenagers always try to be…………..of their parents. (DEPEND)
5. He wished the young couple a life of happiness and………….. (PROSPER)
6. The mayor of Hanoi ordered the suspension of officials responsible for a…………..tree-cutting. (MASS)
7. In my factory, women tend to…………..men by six to one. (NUMBER)
8. Are there any…………..rivers left in the world? (POLLUTE)
9. Young children are very…………..and shouldn’t be allowed to watch violent movies. (IMPRESSION)
V. The following conversation is between a shop assistant and a customer who is buying a camera. Complete the
conversation with the words or phrases given in the box. Write your answer in the space provided. (5 pts)
A. What do I do now?
B. How do I use this camera?
C. What happens iff
D. it’s pulled up
E. if it’s the wrong way
F. I’m ready to take my first picture
G. first take the film
H. Now, the camera is open
I. close the cover
J. faces upward

Customer: (1)…………..?
Assistant: Well, can you see this black button here? If (2)………….., the back cover opens. The button
unlocks the back cover.
(3)…………..
Customer: Do I put the film in the camera now?
Assistant: Yes, (4)…………..out of the small can. Make sure this part of the film (5)…………..like this.
Customer: (6)…………..put it in the other way round?
Assistant: Well, it won’t fit, (7)…………...

28
Customer: (8)…………..?
Assistant: Now put the end of it in the spool here and wind the film on. Turn this handle and then press the
red button. Now (9)…………..of the camera. The film must then be wound on once again.
Customer: The number “ 1 ” appears in the window, and (10)………….. !
VI. The passage below contains 10 errors. UNDERLINE and CORRECT them. Write your answers in the numbered
blanks provided. There is an example. (10 pts)
1 People began to keep animals in zoos over 3,000 years before, when
2 the rulers of China opened an enormous zoo called the Gardens of
3 Intelligence. In much of the early zoos, animals taught to perform
4 for the visitors. This no longer happens and it is accepted that the
5 purpose of zoos are for people to see animals behaving natural.
6 Today, most cities have a zoo or wildlife park. However, not
7 everybody approves of zoos. Person who think that zoos are a good
8 idea say they provide us for the opportunity to learn about the
9 natural world and be close to wild animals. Both of these would
10 not be possible with zoos. On the other hand, some people disapprove
11 of zoos because they believe it is wrong putting animals in cages, and
12 argue that in zoos where are not managed properly, animals live in
dirty conditions and eat suitable food.

0. Line 1: before → ago


Your answers:
1. Line... ……………→ ……………
2. Line... ……………→ ……………
3. Line... ……………→ ……………
4. Line... ……………→ ……………
5. Line... ……………→ ……………
6. Line... ……………→ ……………
7. Line... ……………→ ……………
8. Line... ……………→ ……………
9. Line... ……………→ ……………
10. Line... ……………→ ……………

VII. Fill in each space in the following passage with one suitable word.
The 2015 Nepal earthquake, which (1) ………….more than 8,000 people and injured more than 18,000,
occurred at 11:56 on 25th April. The earthquake (2) ………….about twenty seconds. Its epicenter was the
village of Barpak, Gorkha district, and its hypocenter was at a depth (3) ………….approximately 15 km. It
was the worst (4) ………….disaster to strike Nepal since the 1934 Nepal-Bihar earthquake.
Hundreds of thousands of people became (5)………….when their houses collapsed, entire villages were
flattened. Many old buildings were completely (6) ………….. The country also had a continued risk of
landslides.
Two other powerful earthquakes struck Nepal at 06:11 and 06:45. The (7) ………….earthquake measured
7.9 Mw and its epicenter was identified at a distance of 80km to the northwest of Kathmandu, the capital of
Nepal. Bharatpur was (8)…………..nearest major city to the main earthquake, 53km from the epicenter. The
second one was somewhat less powerful (9)…………..the first one. It occurred 65km east of Kathmandu.
These (10)…………..were really terrible.
VIII. Circle the correct answer A, B, C or D that best fits each of the blank spaces. (10 pts)
MARKETS

29
In practically any country in the world, you are (1) ………….to find a market somewhere. Markets have
been with us since (2) …………. times, and arose wherever people need to exchange the goods they
produced. For example, a farmer might have exchanged a cow for tools. But just as times have (3) …………
so have markets practices. So, (4) ………….in early times the main activity (5) …………. with markets
would have been ‘bartering’- in (6) …………. words exchanging goods - today most stall holders wouldn’t
be too (7) …………. on accepting potatoes as payment, for instance, instead of cash.
In contrast, what might be a common (8) …………. in a modern market in some countries is a certain
amount of ‘haggling’, where customer and seller eventually (9) ………….on a price, after what sometimes
be quite a heated debate. However, behavior which is expected in a market in one country may not be
acceptable in another. Even within one country, there may be some markets where you could haggle quite
(10) ………….and others where it would be advisable not to try.
1. A. inevitable B. confident C. sure D. definite
2. A. ancient B. antique C. old D. past
3. A. changed B.turned C. developed D. differed
4. A. however B. despite C. nevertheless D. whereas
5. A. associated B. relating C. connecting D. attached
6. A. different B. other C. new D. alternative
7. A. fond B. keen C. eager D. pleased
8. A. look B. vision C. sight D. view
9. A. confirm B. consent C. approve D. agree
10. A. simply B. plainly C. easily D. clearly
IX. Choose from the sentences (A-F) the one which fits each gap (1-5). Write your answer in the space provided.
Number 0 is an example.(5 pts)
A. It was the river, the Ryburn, which normally flowed so gently, that threatened us most.
B. It was the year when the storms came early,before the calendar even hinted a winter, even before
November was out.
C. They twisted and turned, rising eastwards and upwards, warning of what was to come.
D. It was far deeper than we’d ever seen it so near our home, lunging furiously at its banks.
E. There in the heights it was like the Niagara Falls, as the water surged over the edge of the dam and poured
into the stream below.
F. It almost completely blocked our lane and made the streamside path slippery and dangerous.

LIVING IN THE VALLEY


We had been living in our valley for sixteen months when we first realized the dangers that could exist in the
surrounding hills and threaten our very survival.
(0)….B…..Until that time, we had felt safe and sheltered in our valley below the protecting hills.
Soon snow began to fall. Within a day it lay some 15cm deep. (1) ………….But on the neighboring heights
the snow was much deeper and stayed for longer. Up there the wind blasted fiercely. Deep in our valley, we
felt only sudden gusts of wind; trees swayed but the branches held firm.
And yet we knew that there was reason for us to worry. The snow and wind were certainly inconvenient but
they did not really trouble us great. (2) ………….; It reminded us of what could have occurred if
circumstances had been different, if the flow of water from the hills had not, many years before, been
controlled, held back by a series of dams.
In a short time, the snow started to melt. Day after day, we watched furious clouds pile up high over the hills
to the west. Sinister grey clouds extended over the valleys. (3) ………….We had seen enough of the sky;
now we began to watch the river, which every day was becoming fuller and wilder.
The snow was gradually washed away as more and more rain streamed from the clouds, but high up in the
hills the reservoir was filling and was fast approaching danger level. And then it happened - for the 1st time in
years the reservoir overflowed. (4)…………..

30
The river seemed maddened as the waters poured almost horizontally down to its lower stretches. Just a
couple of meters from our cottage, the stream seemed wild beneath the bridge. (5) …………. For three days
we prayed that it would stay below its wall. Fortunately, our prayers were answered as the dam held and the
waters began to subside.Thanks to this protection, we can feel our home in the valley is still secure and safe.
X. Rewrite the sentences, beginning with the words given so that the meanings stay the same as the
first ones. (5 pts)
1. I listened to the news last night. It was very informative.
→ The news………………………………………………………………………………………….
2. A lot of children and old people have to go to hospitals these days because of the hot weather.
→ Because …………………………………………………………………………………………..
3. His brother is a professional tennis player, isn’t he?
→ His brother plays………………………………………………………………………………….
4. He listens to the radio almost every day.
→Hardly……………………………………………………………………………………………..
5. They did not decide to move to a bigger house until after the birth of their second son.
→ Only when…………………………………………………………………………………………
XI. Complete the second sentence so that it has similar meaning to the sentence, using the word given.
DO NOT change the word given. You use between TWO and FIVE words, including the word given. (5
pts)
1. The firemen managed to put the fire out after five hours. succeeded
→ The firemen………………………………………………… out after five hours.
2. It takes five hours by car to reach the nearest hotel. drive
→ It is ………………………………………………… to the nearest hotel.
3. Are you familiar with his teaching style yet? used
→ Have you…………………………………………………his teaching style yet?
4. David impressed his new boss by settling down to work. good
→ David…………………………………………………his new boss by settling down to work.
5. “Is Peter likely to change his mind?” Rob asked. chance
→ “Is there…………………………………………………changing his mind?'
Rob asked.
XII. Write a paragraph. (10 pts)
Write a paragraph of about 80 to 100 words on how to protect our environment.
………………………………………………………………………………………………………
………………………………………………………………………………………………………
………………………………………………………………………………………………………
………………………………………………………………………………………………………
………………………………………………………………………………………………………
………………………………………………………………………………………………………
………………………………………………………………………………………………………
………………………………………………………………………………………………………

31
ĐỀ SỐ 08
(Đề thi vào lớp 10 THPT Chuyên Ngoại ngữ Đại học Quốc gia Hà Nội)
Thời gian làm bài: 120 phút
I. Câu 01 - 05: Chọn từ (ứng với A, B, C hoặc D) có phần gạch dưới được phát âm khác với những từ
còn lại trong mỗi câu.
01. A. eliminate B. equal C. elaborate D. ejection
02. A. carriage B. dosage C. massage D. voyage
03. A. suit B. bruise C. suite D. fruit
04. A. calculate B.populate C. contemplate D. fortunate
05. A. apprehension B. division C. precision D. measure
II. Câu 06 - 10: Chọn từ (ứng với A, B, C hoặc D) có trọng âm chính nhấn vào âm tiết có vị trí khác với
những từ còn lại trong mỗi câu.
06. A. ignorant B. decisive C. horizon D. museum
07. A. suffice B. product C. nuclear D. province
08. A. neurosis B. nocturnal C. nominate D. nostalgia
09. A. psychological B. contributory C. argumentative D. hypersensitive
10. A. crescendo B. attorney C. compromise D. endeavor
III. Câu 11 - 20: Đọc đoạn văn sau và chọn phướng án đúng nhất (ứng với A, B, C hoặc D) cho mỗi câu
hỏi.
(1) On the night of September 2, 1666, a fire broke out in a baker's shop near Fish Street Hill in London.
Before the flames were finally extinguished, nearty the entreaty had been reduced to ashes. Over thirteen
thousand homes, fifty churches and numerous public buildings and hospitals were lost in the blaze. For all
practical purposes, London was destroyed.
(5) The Great Fire was not seen as a total tragedy, however. The deplorable conditions of the city had been
attacked by physicians and humanitarians for years before the fire, thus, with the opportunity dearly
presented to create a shining new dty, artists and craftsmen from all over England hurried to submit their
designs for the rebuilding of London.
Among those who submitted plans was Sir Christopher Wren, one of England's leading architects and the
(10) Surveyor of London. The task of rebuilding the city was given to him. Wren realized that the Great Fire
would not have been so damaging if the city had been better laid out: broader streets were needed to replace
the crooked, narrow alleys overhung with dilapidated wooden houses and shops. He also felt that redesigning
the main thoroughfares of London would result in increased and more effective transportation within the
city.
Shortly after Wren began working on his first drafts for the rebuilding. King Charles I issued a proclamation
(15) prohibiting the construction of any house or shop within the city limits until after the plans were
completed. When the plans were unveiled to the citizens of London, however, they were overwhelmingly
rejected. The most vocal leaders of the opposition were the landlords, who feared that such a drastic
widening of the streets would reduce the amount of land available for development.
(20) Winter was approaching; consequently, it was necessary for the rebuilding to proceed at once.
Permission was, therefore, granted for the townspeople and landlords to commence reconstruction of their
houses and shops at the sites where they had been before the fire. Had the need for immediate action not been
so pressing, some kind of compromise could likely have been reached. This was not to be, however, and the
ideas that could (25) have made London one of the world's most beautiful cities never came to pass.
11 The probable meaning of “reduced to ashes” (line 2) is…………….
A. made unbearably hot B. discolored by the fire and smote
C. covered with ashes D. destroyed by fire
12. Which of the followings was probably among the deplorable conditions of the city?
A. lack of adequate lightning

32
B. the rough streets and alleys
C. the run-down condition of many houses and shops
D. all of the above
13. What did Sir Christopher Wren describe as the reason for the damage of the city?
A. the configuration B. the construction C. the landscape D. the pathway
14. The word thoroughfares (line 13) refers to…………….
A. public buildings B. streets
C. alleys D. houses and shops
15. The word proclamation (line 14) refers to…………….
A. promulgation B. speech C. request D. law
16. The word unveiled (line 16) refers to…………….
A. concealed B. disclosed C. obscured D. edipsed
17. What became of the plans for the rebuilding of London?
A. Nothing - they were never finished.
B. They were adopted in modified form.
C. They were followed and London was rebuilt at once.
D. Nothing - London was rebuilt as it had been before the fire.
18. To whom or what does they (line 23) refer to…………….?
A. the original building sites B. the townspeople
C. the burned-down houses and shops D. the landlords
19. Which of the following describes the author's probable attitude toward Sir Christopher Wren's plans?
A. He feels it would have been a mistake to rebuild London according to Wren's designs.
B. He feels it was a mistake for London not to have been rebuilt according to Wren's designs.
C. He feels that someone rather than Wren should have been chosen to plan the rebuilding.
D. He feels that Wren's ideas had much more influence than commonly acknowledged.
20. Which of the following would be an appropriate title for the passage?
A. "The Life and Times of Sir Christopher Wren"
B. "The Rebuilding of London"
C. "The Holocaust of Fish Street Hill"
D. "The London That Might Have Been"
IV. Câu 21 - 30: Đọc đoạn văn sau và chọn phương án đúng nhất (ứng với A, B, C hoặc D) cho mỗi chỗ
trống.
Smart Shoes
Smart shoes that adjust their size throughout the day could soon be available. A prototype has already been
produced and a commercial (21) ……………may be in production within a few years. The shoe contains
sensors that constantly check the amount of room left in it. If the foot has become too large, a tiny valve
opens and the shoe (22) ……………slightly. The entire control system is about 5 mm square and is located
inside the shoe. This radical shoe (23) ……………a need because the volume of the (24) …………… foot
can change by as much as 8% during the course of the day. The system is able to learn about the wearer's feet
and (25) ……………up a picture of the size of his or her feet throughout the day. It will allow the shoes to
change in size by up to 8% so that they always fit (26) ……………They are obviously more comfortable
and less likely to cause blisters. From an athlete's point of view, they can help improve (27) ……………a
little, and that is why the first (28) ……………for the system is likely to be in a sports shoe. Eventually, this
system will find a (29) ……………In many other household items, from beds that automatically change to fit
the person sleeping in them, to power tools that (30) ……………themselves to the user's hand for better grip.
21. A. assortment B. version C. style D. variety
22. A. amplifies B. develops C. expands D. increases
23. A. detects B. finds C. meets D. faces
24. A. average B. general C. usual D. medium

33
25. A. build B. pick C. grow D. set
26. A. exactly B. absolutely C. completely D. totally
27. A. achievement B. performance C. success D. winning
28. A. purpose B. exercise C. use D. operation
29. A. function B. part C. way D. place
30. A. shape B. change C. respond D. convert
V. Câu 31 - 40: Chọn từ/ cụm từ thích hợp nhất (ứng với A, B, C hoăc D) để hoàn thành mỗi câu sau.
31. Save your money. Don't……………it too quickly.
A. go through B. die down C. hold on D. touch on
32. They all have to follow the rules, and none of them is……………the law.
A, over B. beyond C. above D. onto
33. They……………the aid of the United States but did not receive it.
A. asked B. found C. sought D. obtained
34. The police spokesman said he was…………….to believe that the arrested man was the serial killer they
had been looking for.
A. inclined B. seemed C. suspected D. supposed
35. I'm…………….my brother is.
A. nowhere near as ambitious as B. nothing near as ambitious as
C. nowhere like so ambitious as D. nothing as ambitious as
36. He says he might come, but I…………….
A.doubt B. doubt it C.doubt so D. doubt about it
37. I'm sure your bank manager will lend you a…………….ear when you explain the situation to him.
A. merciful B. sympathetic C. pitiful D.
compassionate
38. Have you seen the girl…………….?
A. that I told B. I told you of C. whom I told you D. I told you of her
39. Children usually…………….a flu much more quickly than adults.
A. pick up B. pick at C. pick on D. pick out
40. The accident…………….was the mistake of the driver.
A. causing many people to die B. caused great human loss
C. to cause many people die D. which causes many people to die
VI. Câu 41 - 50: Sử dụng từ trong ngoặc ở dạng thích hợp nhất để điền vào các chỗ trống.
Shadow puppetry is a traditional art form that often goes (41. appreciate)…………….in modem
times. A large part of the (42. appealable)…………….of puppet shows is the (43.craft)…………….behind
the creation of the actual puppets. In shadow puppetry, on the other hand, the puppets remain (44. see)……,
so the real artistry is in the presentation. The combination of the puppets’ shape, the background screen, and
the light itself creates the overall effect of the shadow puppet show. The task of the director is to ensure these
elements are working together (45. harmony)…………….in order to produce the optimal experience for the
audience. The screen is the medium through which the audience experiences the performance, so selecting
the best screen is among great (46. essential)…………….One unique challenge for the
director is that the presentation is two-dimensional. The screen is flat, so puppets can only move forwards
and backwards. Having chosen a screen and designed the set, the next step is to determine the light that will
be used. There are several factors to be considered: (47. intense)……………., spread, and angle. Therefore,
finding the optimal (48. combine)…………….of light, shadow involves careful (49. plan)……………..and
scrupulous design. Every detail must be controlled in relation to others, making shadow puppetry an art of
(50. precise)…………….
VII. Câu 51 - 60: Tìm từ thích hợp nhất để điền vào chỗ trống trong đoạn văn. Mỗi chỗ trống chỉ điền
MỘT từ.
Son Doong Cave is in the heart of Phong Nha Ke Bang National Park in Quang Binh province of

34
Central Vietnam. Only recently (51)…………..in 2009-2010 by the British Cave Research Association, the
cave has only been open to the (52) …………..since 2013.
Fewer people have seen the inside of Son Doong Cave than have stood on the summit of Mount
Everest. Join us on this otherworldly expedition and become one of the lucky few (53) …………..have had
the life changing experience of exploring the world's largest cave.
Imagine trekking straight into the (54) …………..of the world's largest cave on an expedition unlike
any other. A cave is (55) …………..massive that a Boeing 747 could fly through its largest cavern. Foreign
landscapes found (56) …………..else, enormous stalagmites rising from the ground and statuesque stalactites
hanging from the celling like an alien species. Jungles emerge (57) …………..the cave itself, which is so
surreal that it's worth seeing once. Misty clouds envelop the whole scene,, a result of the cave's own localised
weather system. Passages adorned (58) …………..ancient fossils offer evidence of the millions of years that
have passed on this Earth.
As you approach the Jungle just outside the entrance, the (59) ………….. of cool wind that cascades
out brings to life everything inside of you. Hazy, cold and exhilarating, it is apparent that there's (60) ………
magical waiting just beyond the opening to the cave.
VIII Câu 61 - 70: Sử dụng từ cho sẵn trong ngoặc và các từ khác để hoàn thành câu thứ hai sao cho ý
nghĩa của nó tương tự như câu thứ nhất. Chỉ được viết thêm từ 2 đến 5 từ và không được thay đổi
dạng thức của từ chữ sẵn.
61. Mary cried her eyes out just after she was told she'd been rejected for the Job. (broke)
Mary………………………………………….as soon as she heard she'd been rejected for the job.
62. I never thought that I would win the lottery. (occurred)
It never………………………………………….that I would win the lottery.
63. I failed to make him understand that I didnt want to see him anymore. (across)
I failed………………………………………….that I didn’t want to see him anymore.
64. Would you give us your answer as soon as possible? (convenience)
Please reply………………………………………………………………………………..
65. I'm so sorry, I didn't realise it was so late. (track)
Sorry, I……………………………………………………………………………..time.
66. Everyone who spoke to the victim is a suspect. (under)
Everyone who spoke to the victim…………………………………………………………
67. She will probably be chosen for the beauty contest. (stands)
She…………………………………………………………chosen for the beauty contest.
68. "Can you tell me what happened yesterday, John?" She asked. (account)
She asked John………………………………………………what had happened the day before.
69. Those two makes of car are practically the same. (hardly)
There are…………………………………………………………those two makes of car.
70. There is no way that I can meet you tomorrow. (question)
Meeting you tomorrow…………………………………………………………
IX. Câu 71 - 80: Viết lại các câu theo gợi ý (kể cả những từ bắt đầu câu cho sẵn) sao cho ý nghĩa ban
đầu của câu không thay đổi.
71. It's sad, but the crime rate is unlikely to go down this year.
Sad as……………………………………………………………………………………….
72. You must concentrate on your study more.
You must apply.…………………………………………………………………………….
73. It has been nearly one year since they stopped subscribing to that magazine.
They cancelled………………………………………………………………………………
74. I don't feel like going to the party.
I am not in…………………………………………………………………………………..
75. "Please don't run so fast!" Suzy begged her friend.

35
Suzy pleaded………………………………………………………………………………..
76. You should not only balance your diet but also do sports.
In addition…………………………………………………………………………………..
77. Immediately after their arrival, the meeting was delayed.
Barely
78. If you don’t pay on time, your booking will be cancelled.
Failure ……………………………………………………………………………………….
79. David is proud of the fact that he is never late.
David prides………………………………………………………………………………….
80. It’s nobody's fault that the meeting was cancelled.
Nobody is to………………………………………………………………………………….
X. Câu 81: Viết một ĐOẠN VĂN khoảng 120 - 150 từ, sử dụng lập luận và dẫn chứng cụ thể để bình
luận về chủ đề sau.
To be admitted to university, using the result of study during high school years is better than that of the
university entrance examination. Do you agree or disagree?
………………………………………………………………………………………………………………
………………………………………………………………………………………………………………
………………………………………………………………………………………………………………
………………………………………………………………………………………………………………
………………………………………………………………………………………………………………
………………………………………………………………………………………………………………
………………………………………………………………………………………………………………
………………………………………………………………………………………………………………

ĐỀ SỐ 09
(Đề thi vào lớp 10 THPT Chuyên TP. Hồ Chí Minh)
Thời gian làm bài: 120 phút
I. CHOOSE THE CORRECT ANSWER TO FILL IN THE BLANK (2 PTS)
1……………could only have been made by someone incompetent.
A. How serious a mistake B. Such serious mistake
C. So serious is this mistake D. So serious a mistake
2. She insisted that the report……………as his source of information.
A. not be mentioned B. not to mention C. not mentioning D. not mention
3……………traveling is educational, it can also be stressful and expensive.
A. Since B. Providing C. While D. Even
4. They arrived home only to find their house……………in their absence.
A. was burgled B. has burgled C. had been burgled D. was burgling
5. There isn't a……………of truth in his words. What a liar!
A. wealth B. trace C. pack D. grain
6. Don't count your……………before they
A. ducklings B. chickens C. puppies D. birds
7. Manh: Should I buy a new bike or a used one? ~ Dung: Buy……………you can afford.
A. the one whichever B. which one that

36
C. those that D. whichever one
8. Dung: “Merry Christmas!” Manh."……………"
A. You, too B. Merry Christmas C. welcome D. You bet
9. I hope everything is OK. They……………several hours ago.
A. Would have called B. supposed to call
C. must have called D. were to have called
10. He was a great athlete when he was younger. He now still ……………if he hadn't had that acident.
A. Would have been B. had been C. will be D. would be
11. After a month-long investigation, the police still aren’t sure who ……………
A. is blamed by them B. do they blame
C. to lay the blame on D. to be blaming
12. The food wasn't bad, but I’d……………to go somewhere else next time.
A. rather B. prefer C. better D. liked
13. The company is famous for making sturdy work clothes that can……………hard wear.
A. stard up for B. stand up to C. stand in for D. stand up against
14. Employees are advised to take……………of the changes in the company policy.
A. hold B. effect C. root D. notice
15. The day their first child was born, the new parents were……………with joy.
A. overpowered B. overtaken C. overcome D. overthrown
16. By the time you finish your job, everyone……………home.
A. has gone B. will have gone C. had gone D. was going
17. The……………child is forever asking questions. He's incredibly curious.
A. acquisitive B. acquitted C. inquisitive D. exquisite
18. If you think you are right, then…………… Don't let him bully you into changing your mind.
A. stand your ground B. hit it off
C. kick the bucket D. give the green light
19. At the end of the broadcast, the TV station received a……………of complaints.
A. hurricane B. tsunami C. tornado D. torrent
20. Dung: "Stop talking. You are so noisy." ~ Manh: "He is the noisiest in this class.”
A. Why not? B. Look who’s talking
C. You can say that again. D. You bet!
21. A: “Have you told your dad what's bothering you? ~ B: “I'd……………in you than in my parents."
A. rather to confide B. better confide C. rather my confiding D. sooner confide
22……………is over your head is just an exaggeration.
A. That this entrance exam B. When this entrance exam
C. This entrance exam which D. This entrance exam
23. When I decided to study two foreign languages at the same time, I knew I ……………off more than I
could chew.
A. took B. bit C. cut D. broke
24. My visit was supposed to be a surprise but my sister……………it away.
A. gives B. takes C. brings D. shows
25……………he couldn't solve that mathematical puzzle.
A. As intelligent as was he B. Intelligent as he was
C. So intelligent was he D. Although he is intelligent
26. The test was given……………short notice, so we all couldn't finish it well.
A. by B. in C. for D. on
27. Things would have been better……………to my advice.
A. you not listened B. were you listened
C. had you listened D. if you listened

37
28. They will hold a party……………their son's 16th birthday.
A. in recognition B. in celebration to
C. with a view to celebrate D. in celebration of
29. Where have you been? I've been trying to get……………of you all day.
A. touch B. hold C. contact D. rid
30. The break-in turned out to be …………… because it made us more aware of the insecurity of the
neighbourhood.
A. a Jack of all trades B. a storm in a teacup
C. a blessing in disguise D. a matter of life and death
31. My new hiking boots will be great once I've……………them……………
A. broken - in B. put - through C. taken - to D. fixed-up
32. We admire his……………to his students.
A. generosity B. sympathy C. assumption D. thoughtfulness
33……………, the meeting began.
A. After we have at down B. All of us having taken the seats
C. Our having seated D. Once we had been sat down
34……………he tried to fix the machine.
A. Not being a computer expert notwithstanding
B. Despite of not being a computer expert
C. Though no computer expert
D. In spite of being no computer expert
35. The draw took place yesterday but the competition winners……………
A. are yet to be announced B. haven't been being announced
C. are as yet to have been announced D. haven't announced yet
36. I had……………on my face after I remembered the wrong day for her birthday party.
A. egg B. sugar C. salt D. tomatoes
37. The tickets cost only a small amount of money……………
A. off his head B. by head C. offhand D. per head
38. The robbery is being……………
A. broken out B. taken on C. looking into D. putting in
39. The book you are looking for is out of……………
A. order B. print C. the question D. work
40. He was too tired to mow the lawn, so he……………one of his sons to do it.
A. had B. made C. got D. took
CHOOSE THE WORD OR PHRASE THAT BEST FITS EACH BLANK IN
THE FOLLOWING PASSAGE. (1.5 PTS)
On a yearly (1)……………, the USA is afflicted by hurricanes on the east coast, flooding in the Midwest,
forest fires, earthquakes and any number of tornadoes, blizzards and storms. Historically, the weather has (2)
……………as harsh a chord in people’s lives as any we feel today. Historians list the four major events to
wreak devastation (3) …………… the country as the Chicago fire in 1871, the Johnstown flood and the
Galveston hurricane of 1900, and San Francisco earthquake in 1906. These (4) ……………provide a
flavor of the havoc that natures storms have wrecked on humanity, and our reactions to them. Galveston,
Texas, (5) ……………on the island of Galveston in the gulf of New Mexico, (6) …………… on industry
and exports. Its over 30,000 citizens in the early 1900's were generally economically successful and
comfortable with existing conditions. (7) ……………the ill-fated morning of Sept 7, 1900,the people of
Galveston enjoyed their life on the remote island. (8) ……………weather watchers had been following a (9)
……………storm in the Atlantic for several days and were warning of the (10)……………danger of a
hurricane. Unfortunately, people (11) ……………the warnings. A day after the hurricane had hit the island,
all that (12) ……………of the beautiful city was a mass of crumbled buildings, debris and hopeless survivors

38
(13) ……………aimlessly with the stench of rotting flesh all around. However, a few days later, that the city
was beginning to heal became (14) ……………everywhere - soon new buildings were (15) …………… the
burned, and the lifeblood of the ravaged city was returning.
1. A. foundation B.theory C. basis D. institution
2. A. punched B. struck C. pinched D. played
3. A. on B. at C. with D. for
4. A. dangers B. risks C. threats D. catastrophes
5. A. lied B. placed C. located , D. built
6. A. flourishes B. thrives C. grows D. develops
7. A. After B. Up to then C. So far D. Until
8. A. Therefore B. Correspondingly C. However D. Consequently
9. A. growing B. enlarging C. massive D. stretching
10. A. potential B. embedding C. intending D. impending
11. A. refused B. rejected C. ignored D. denied
12. A. remained B. existed C. lingered D. maintained
13. A. wandered B. to wander C. were wandering D. wandering
14. A. concealed B. notorious C. obscure D. evident
15. A. overwhelming B. rebuilding C. designing D. refurbishing
IDENTIFY THE MISTAKE IN EACH SENTENCE BELOW. (0.5 PTS)
1. Tropical forests exist close to the equator, which both high temperatures and abundant rainfall occur all
A B C D
year round.
2. The Smiths are very proud of that their son always gets high marks in his courses.
A B C D
3. We called the baseball park up to ask that when the game was scheduled to begin that afternoon.
A B C D
4. The American Indians killed the buffalo only when necessity to obtain food, clothing and shelter.
A B C D
5. Some relatives of mine like staying at their cabin on Lake Omega every summer holidays.
A B C D
6. In the United States among 60 percent of the space on the pages of newspapers is reserved for
advertising. A B C
D
7. Gilbert Newton Lewis, a chemist, helped to develop the modern electron theory of valence, a theory
A B
explains the forces holding atoms together in molecules.
C D
8. Because of a high birthrate and considered immigration, the United States population in the late
A B C
nineteenth century increased tremendously from 31 million in 1860 to 76 million in 1900.
D
9. For most of their history, especially since the 1860’s, New York City has been undergoing major ethnic
A B C
population changes.
D
10. Rocks have forming, weaning away and re-forming ever since the Earth took shape.
A B C D
IV. READ THE PASSAGE AND CHOOSE THE BEST ANSWERS TO THE QUESTIONS THAT
FOLLOW. (1 PTS)

39
Television’s contribution to family life in the United States has been an equivocal one. For while it
has, indeed, kept the members of the family from dispersing, it has not served to bring them together. By
dominating the time families spend together, it destroys the special quality that distinguishes one family from
another, a quality that depends to a great extent on what a family does, what special rituals, games, recurrent
jokes, familiar songs, and shared activities it accumulates.
“Like the sorcerer of old,” writes Urie Bronfenbrenner, “the television set casts its magic spell,
freezing speech and action, turning the living into silent statues so long as the enchantment lasts. The
primary danger of the television screen lies not so much in the behavior it produces - although there is danger
there - as in the behavior it prevents : the talks, games, the family festivities, and arguments through which
much of the child’s learning takes place and through which character is formed. Turning on the television set
can turn off the process that transforms children into people.”
Of course, families today still do special things together at times: go camping in the summer, go to
the zoo on a nice Sunday, take various trips and expeditions. But the ordinary daily life together is
diminished - that sitting around at the dinner table, that spontaneous taking up of an activity, those little
games invented by children on the spur of the moment when there is nothing else to do, the scribbling, the
chatting, the quarreling, all the things that form the fabric of a family, that define a childhood.
Instead, the children have their regular schedule of television programs and bedtime, and the parents
have their peaceful dinner together. But surely the needs of adults are being better met than the needs of
children, who are effectively shunted away and rendered untroublesome.
If the family does not accumulate its backlog of shared experiences, shared everyday experiences that
occur and recur and change and develop, then it is not likely to survive as anything other than a caretaking
institution.
1. Which of the following best represents the author’s argument in the passage?
A. Television has negative effects on family life.
B. Television has advantages and disadvantages for children.
C. Television should be more educational.
D. Television teaches children to be violent.
2. The word it in the passage refers to…………….
A. dominating B. time C. television D. quality
3. Why is Urie Bronfenbrenner quoted in paragraph 2?
A. To present a different point of view from that of the author
B. To provide an example of a television program that is harmful
C. To expand the author’s argument
D. To discuss the positive aspects of television
4. The word freezing in the passage is closest in meaning to…………….
A. controlling B. halting C. dramatizing D. encouraging
5. Urie Bronfenbrenner compares the television set to…………….
A. a statue B. an educator C. a family member D. a magician
6. Which of the following would be an example of what the author means by a special thing that families do?
A. Going on vacation in the summertime
B. Playing cards together in the evening
C. Reading to the children at bedtime
D. Talking to each other
7. The thing that “form the fabric of a family” in paragraph 3 are…………….
A. special things B. ordinary things C. television programs D. children
8. The word it in the passage refers to…………….
A. the television B. the family C. its backlog D. an institution
9. According to the author, what distinguishes one family from another?
A. Doing ordinary things together B. Watching television together

40
C. Celebrating holidays together D. Living together
10. It can be inferred from the passage that a caretaking institution is one in which care is given…………
A. charitably B. lovingly C. constantly D. impersonally
V. SUPPLY THE APPROPRIATE FORMS OF THE WORDS IN THE BRACKETS. (1.5 PTS)
1. Joining this project is a……………Just do it. (BRAIN)
2. There are a lot of……………articles in this newspaper. Why not read it? (NEWS)
3. My cousin is a……………person. He is aware of all the latest fashions and wanting to follow them.
(FASHION)
4. The burglar gained entry to the building after……………the alarm. (ABILITY)
5. Whatever happens, don't let this failure……………you. (HEART)
6. Your carelessness may do……………harm to people. (CALCULATE)
7. The……………listed for the pills meant that she couldn't take them because she may be allergic to some
of the chemicals in them. (INDICATE)
8. We were defeated because we were……………(NUMBER)
9. It is……………summer, but it's rather autumnal today. (THEORY)
10. This type of……………screen enables drivers to have a clear view even when it is smashed.
(SHATTER)
VI. SUPPLY EACH BLANK WITH ONE SUITABLE WORD. (1.5 PTS)
The human race is not the only one species of beings in the living (1)……………Many other species
exist on this planet. However, human beings have a great influence on the (2)……………of the world. They
are changing the environment by building cities and villages where forests (3)……………stood. They are
affecting the water supply by using water for industry and agriculture. They are changing (4)……………
conditions by cutting down trees in the forests and are destroying the air by adding pollutants to it.
It can be said that human beings are changing the environment in all respects (5)……………their
actions and their habits. This has (6)……………in two serious consequences. The first is that many species
of animals are killed. The second is that the environment (7)……………these animals are living is (8)
……………destroyed. As a result, the number of rare animals is decreasing so rapidly that they are in danger
of becoming (9)……………
In order to make sure that these rare animals do not disappear, (10)……………have been made to
protect endangered nature. Scientists have made (11)……………of these species and suggested ways to save
them. Many organizations have been set up and funds have been (12)……………Thousands of national
parks all over the world have been established to protect endangered animals. (13)……………have been
introduced to prohibit killing endangered animals in the natural habitats where animals are living.
If people’s interference with the environment decreases, more species will survive and (14) …………
offspring. The earth will be a happy planet where human beings, animals and plants peacefully (15)…………

VII. REWRITE THE FOLLOWING SENTENCES WITHOUT CHANGING THE MEANING.


(2PTS)
1. You have to accept the truth although it is hurtful. (TERMS)
Hurtful……………………………………………………………………………………………………
2. The wind was so strong that we couldn’t walk. (HARDLY)
Such………………………………………………………………………………………………………
3. You will start to feel better as soon as this drug is effective. (EFFECT)
The moment………………………………………………………………………………………………
4. My dad is now abroad and I am in charge of the office. (UNDER)
My dad has……………………………………………………………………………………………….
5. He was extremely happy because he won that scholarship. (MOON)
Had……………………………………………………………………………………………………….
6. He received another letter from her as soon as he found the time to reply to her. (ROUND)

41
No sooner…………………………………………………………………………………………………
7. I really don't like it when you cheated me yesterday. (RIDE)
I'd rather you………………………………………………………………………………………………
8. Don't make a fuss over such trivial things. (MOUNTAIN)
Don’t……………………………………………………………………………………………………….
9. We were amazed to know that he was a liar. (UNTRUTH)
To……………………………………………………………………………………………………………
10. Everyday television viewers witness some sort of violence or crime on their screens. (GOES BY)
Hardly . ……………………………………………………………………………………………………

ĐỀ SỐ 10
(Đề thi vào lớp 10 THPT Chuyên Hưng Yên)
Thời gian làm bài: 120 phút
PART A: (0,5 point) PHONETICS
Choose the word whose stress pattern is different from that of the others. Write A, B, C or D on your
answer sheet.
1. A. understand B. engineer C. benefit D. Japanese
2. A. casual B. embroider C. designer D. occasion
3. A. enjoyable B. convenient C. exciting D. interesting
4. A. purpose B. remote C. control D. respond
5. A. appreciate B. experience C. embarrassing D. situation
PART B: (3,0 points) VOCABULARY AND GRAMMAR
I. Choose the best word or phrase to complete the following sentences. Write A, B,C or D on your
answer sheet (2,0 points).
1. Tom made a number of suggestions, most of……………were very helpful.
A. them B. which C. that D. whom
2. The teacher explained the lesson twice……………the students understood it clearly.
A. as long as B. so that C. because D. as if
3. Both Mary and Ellen as well as Jean……………on the tour now.
A. are going B. is going C. has been going D. was going
4. I don’t know whether……………for the job or not.
A. to applying B. apply C. to apply D. applied
5.……………extremely bad weather in the mountains, we’re no longer considering our skiing trip.
A. Due to B. Because C. Since D. Thanks to
6. You must leave at once, you……………miss the train.

42
A. however B. yet C. still D. otherwise
7.……………of English learners have real chance to use it every day.
A. A few B. Few C. A little D. Not much
8.……………becoming extinct is of great concern to zoologists.
A. That giant pandas are B. Giant pandas are
C. Are giant pandas D. Giant panda is
9. I’ve applied for the job……………in the newspaper last month.
A. advertised B. advertising C. be advertised D. being advertised
10. ……………to Jim myself, I can’t tell why he would do such terrible things.
A. Not to be speaking B. Not to have spoken
C. Because of not being spoken D. Not having spoken
11. The government hopes to……………its plans for introducing cable TV.
A. turn out B. carry out C. carry on D. keep on
12. I’ll be back in a minute, Jane. I just want to……………my new tape recorder.
A. try out B. try on C. look up D. take up
13. We can communicate with friends by……………of e-mail.
A. means B. paths C. methods D. uses
14. Learning a foreign language also includes learning the…………… of that country.
A. reputation B. scenery C. culture D. nature
15. We have worked out the plan and now we must put it into……………
A. practice B. reality C. fact D. deed
16. There were 50……………in the talent contest.
A. competitors B. examinees C. customers D. interviewees
17. Her job was so……………that she decided to quit it.
A. interesting B. stressful C. satisfactory D. wonderful
18. She …………….her husband’s job for his ill health.
A. accused B. blamed C. caused D. claimed
19. Anne: “Make yourself at home.” - John: “…………….”
A. Yes, can I help you? B. Don’t mention it.
C. Thanks! Same to you. D. That’s very kind. Thank you.
20. Minh: “Do you mind if I borrow a chair?” - Hung: “……………..Do you need only one?”
A. I'm sorry B. Not at all C. Yes, I do D. Yes, I would
II. FZZZ in each blank with a suitable word deriving from one of the words in the box. Write them on
your answer sheet. (1,0 point)

retire, promote, luck, interviewer, refer,


pensionable, applicant, success, train, employment

ADVICE TO YOUNG PEOPLE ABOUT TO START WORK


In these days of high (1)…………., it is often difficult for young people to find a job. If they are lucky
enough to be asked to go for a(n) (2) …………., they may find that there are at least 20 other (3) ………….
for the job. If a company is thinking of offering you a job, they will ask you for at least one (4) ………….
from either your previous employer or someone who knows you well. Before taking up your job, you may
have to sign a contract. You will probably have to do some (5) ………….,which help you to do the job more
(6) ………….. Once you have decided that this is your chosen career, you will then have to work hard to try
and get (7) ………….,which usually brings more responsibility and more money! If you are (8) ………….,
you may be made redundant, and not be able to find another job. It is also a good idea to pay some money
into a (9) ………….scheme, which will help you to look after yourself and your family when you are (10)
………….. Finally, good luck!

43
PART C: (2.5 points) READING
I. Choose the word or phrase that best fits the blank space in the following passage. Write A, B, C or D
on your answer sheet. (1,0 point)
Most people feel that when they dream, they are (1) ………….off to another world. On the (2) ………….,
dreams are often connected to our daily lives. When our whole (3) ………….is filled with something, when
we are either very upset (4) ………….when we are in good spirit, a dream will represent this reality in
symbols. It is often (5) ………….that we benefit from dreams because they help the spirit to heal itself, when
things (6) ………….wrong. Dreams are therefore a kind of escape, almost a holiday from (7) ………….
life, with its fears and responsibilities. It is, however, a strange kind of holiday because whether we have a
wonderful time or whether it turns (8)…………. to be a nightmare, we quickly forget it. Most dreams
disappear forever (9)…………. you are one of those people disciplined enough to write them down as soon
as you (10) …………..
1. A. carried B. brought C. taken D. guided
2. A. opposite B. contrast C. contrary D. other side
3. A. mind B. spirit C. brain D. soul
4. A. else B. or C. but also D. unless
5. A. spoken B. declared C. said D. stated
6. A go B. turn C. become D. get
7. A. genuine B. real C. actual D. factual
8. A. off B. up C. down D. out
9. A. unless B. except that C. therefore D. thus
10. A. arise B. awake C. rise up D. wake up
II. Read the following passage and choose the best answer to complete the sentences. Write A, B, C or
D on your answer sheet. (0,5 point)
“Women hold up half the sky”. This is an old Chinese saying. However, research shows that perhaps women
do more than the share of “holding up the sky”.
Fifty percent of the world’s population are women, but in nearly two-thirds of all working hours, the work is
done by women. They do most of the domestic work, for example, cooking and washing clothes. Millions
also work outside the home. Women hold forty percent of the world’s jobs. For this work, they earn only 40
to 60 percent as much as men, and of course they earn nothing for their domestic work.
In developing countries, where three-fourths of the world’s population live, women produce more than half
of the food. In Africa, 80 percent of all agricultural work is done by women.
In parts of Africa, this is a typical day for a village woman. At 4.45 a.m she gets up, washes, and eats. It takes
her half an hour to walk to the fields, and she works there until 3.00 p.m. She collects firewood until 4.00 p.m
then comes back home. She spends the next hour and a half preparing food to cook. Then she collects water
for another hour. From 6.30 to 8.30 she cooks. After dinner, she spends an hour washing the dishes. She goes
to bed at 9.30 p.m.
1. What is “domestic work”?
A. housework B. firework C. homework D. gardening
2. The average woman earns…………….the average man.
A. more than B. less than C. the same as D. the most
3. In Africa, women do……………. of all agricultural work.
A. four-fifths B. three-fourths C. two-thirds D. one-fifth
4. What is NOT mentioned as the work of a village woman in Africa?
A. working in the fields B. collecting firewood
C. bringing water home D. preparing food for children going to school
5. What is the main topic of the passage?
A. Women work outside the home.
B. Women work in the field.

44
C. The work of women.
D. The role of women.
III. Fill in each numbered blank with one suitable word. Write it on your answer sheet. (1,0point)
The country is more beautiful than a town and pleasant to live in. Many people think so, and go to the
country (1)……………the summer holidays though they cannot live there all the year round. Some have a
cottage built in a village (2)……………that they can go there whenever they can find the time. English
villagers are not all alike, but (3)……………some ways they are not very different from each other. Almost
every village has a church, the round or square tower of (4)…………… can be seen for many miles around.
Surrounding the church is the churchyard, (5)……………people are buried.
The village green (6)…………… a wide stretch of grass, and houses or cottages are (7)……………round
it. Country life is now fairly comfortable and many villages (8)……………water brought through pipes into
each (9)……………. Most villages are so close to some small towns that people can go there to buy (10)
……………, they can’t find in the village shops.
PART D: (4,0 points) WRITING
I. Choose the underlined word or phrase that needs correcting. Write A, B, C or D on your answer
sheet. (1,0point)
1. The sum of all chemical reactions in an organism’s living cells are called its metabolism.
A. all B. reactions in C. are D. its
2. My father has a mechanic to repair his motorbike monthly.
A. has B. a mechanic C. to repair D. monthly
3. The woman tells them close their eyes tightly and cover them with their hands
A. tells B. close C. tightly D. with
4. The lesson that we are learning now is very interested but difficult.
A. that B. now C. interested D. but
5. It is their nearly perfect crystal structure that gives diamonds their hardness brilliance, and transparent.
A. nearly perfect B. that gives C. hardness D. transparent
6. There's hardly no sugar left, so we must get some when we go shopping.
A. There's B. no sugar C. must D. some
7. Mai and Lan have a row because they have misunderstood one another.
A. have B. because C. misunderstood D. one another
8. Some children, before going to school, go to kindergartens, officially are called nursery schools.
A. before B. go C. officially D. are called
9. Mary Rinehart was a pioneer in the field of journalist in the early twentieth century.
A. a pioneer B. field C. journalis D. in the
early
10. Needles are simple-looking tools, but they are very relatively difficult to make.
A. simple-looking B. but C. very relatively D. to make
II. Using the word given in each bracket and other words to complete the second sentence so that it has
the same meaning as the first. You must use between three and five words. DO NOT CHANGE THE
WORD GIVEN IN I CAPITAL. Write your answer on the answer sheet. (1,0 point)
1. They’re telling me that I must make a decision soon. (PRESSURE)
→ They are putting …………………………………………………a decision soon.
2. She was strongly impressed by his trip to India. (IMPRESSION)
→ His trip to India…………………………………………………her.
3. We needn’t get a visa for Singapore. (NECESSARY)
→ It isn't…………………………………………………a visa for Singapore.
4. It would have been better if he had called the police earlier. (SHOULD)
→ He …………………………………………………the police earlier.
5. As long as you keep calm, you can pass the driving test. (SECRET)

45
→ Keeping calm…………………………………………………your driving test.
III. Finish each of the following sentences in such a way that it means exactly the same as the sentence
printed before it. Write your answers on the answer sheet. (2,0 points)
1. Nam can describe people far better than me.
→ Nam is much better…………………………………………………………………….
2. The boys prefer playing soccer to staying indoors.
→ The boys would rather…………………………………………………………………
3. People say that he sold his house to pay the debt.
→ He is……………………………………………………………………………………
4. I have never seen such a mess in my life.
→ Never in………………………………………………………………………………..
5. You read a lot. You know much about the world.
→ The more……………………………………………………………………………….
6. Do you have a good relationship with your boss?
→Are ……………………………………………………………………………………. ?
7. It’s only when you’re ill that you start to appreciate good health.
→ It is not until……………………………………………………………………………
8. It was her lack of confidence that surprised me.
→What I found……………………………………………………………………………
9. We haven’t decided where to go this weekend yet.
→We haven’t made……………………………………………………………………….
10. The heavy rain prevented us from going camping in the mountain.
→But for…………………………………………………………………………………

C. ĐỀ THI NĂM 2016 - 2017


ĐỀ SỐ 11
(Đề thi vào lớp 10 THPT Chuyên Nguyễn Trãi - Hải Dương)
Thời gian làm bài: 120 phút
Thí sinh làm bài vào tờ giấy thi
Phần trắc nghiệm: Chỉ cần viết phương án chọn A, B, C hoặc D
Phần tự luận: Viết đầy đủ theo yêu cầu của bài
(Thí sinh không được sử dụng bất kỳ tài liệu gì)
A. PHONETICS
Choose the word that has a different stress pattern fromthat of the others (3.0 points).
1. A. congratulate B. celebrate C. compulsory D. communicate
2. A. interactive B. experience C. commercial D. linguistics
3. A. surgeon B. surprise C. surplus D. surface
B. GRAMMAR - VOCABULARY - LANGUAGE FUNCTIONS
I. Choose the word, phrase or expression which best completes each sentence below (17.0 points).
4. Hospital staff are…………….pressure to work longer hours.
A. in B. for C. under D. with
5.“…………….the villagers, I’d like to thank you for your generous donation,”
Mr. Lam said.
A. According to B. On behalf of C. In addition to D. Thanks to
6. The temperatures take place vary widely for different materials.
A. which melting and freezing B. at which melting and freezing
C. which they melt and freeze D. at which they melt and freeze

46
7. The environment is…………….almost everywhere and many species of animals are in danger of
extinction.
A. threatening seriously B. serious threatened
C. seriously threaten D. seriously threatened
8. “After careful consideration of your proposal, I regret…………….that we are unable to accept it,” the
manager said to Mr. Edward.
A. to say B. to saying C. saying D. having to say
9.…………….a tornado spins in a counterclockwise direction in the northern hemisphere, it spins in the
opposite direction in the southern hemisphere.
A. That B. However C. Because of D. Although
10. The boy said, “But for your encouragement, I…………….all the difficulties I had in the English course.
Thank you.”
A. will not have overcome B. would not overcome
C. will not overcome D. would not have overcome
11. One difference between mathematics and language is that mathematics is precise……………..
A. language is not B. but language not
C. while is language D. while language is not
12. The policeman asked Joe…………….
A. where had he gone the previous day
B. where did he go the previous day
C. where he had gone the previous day
D. where he had gone yesterday
13. Parking near the school causes severe traffic……………..
A. accumulation B. concentration C. congestion D. condensation
14. Susan: “What a lovely house you have!” Tom: “…………….”
A. Thanks. I can’t afford it B. Thanks. Hope you'll drop in
C. Oh yes, yes! You’re right D. Of course not! It's costly
15. Woman: “Could you please bring me some more sugar?
Waiter: “……………..”
A. Yes, I can, Madam B. No, thanks
C. Certainly, Madam D. I can’t agree with you more
16. Jeannette: “How are you going to get downtown this afternoon, Ann?”
Ann: “ …………….”
A. I’ll take a cab, perhaps B. It would be interesting
C. About 3 miles, I guess D. Thanks. I’d love to
17. Bill: “What do you think of what Mr. Cook said at the meeting.”
Susan: “…………….”
A. Well, I could help myself.
B. Sorry, I have no idea.
C. Oh, he was good-looking, wasn’t he?
D. Oh, the meeting was boring.
18. Static electricity…………….from one cloud to another or between clouds and the ground creates
lightning.
A. flows B. the flow C. flowing D. is flowing
19. Sarah invited loads of friends to her party, but only a handful of them…………….
A. made up B. pulled up C. picked up D. turned up
20. “Just keep…………….on the baby while I cook the dinner, will you?” Laura said to her husband.
A. a care B. an eye C. a look D. a glance
II. Give the correct form of the words in brackets (5.0 points).

47
21. The golden autumn light provided the (INSPIRE)…………….for the painting.
22. John enjoys working in general medicine, but he hopes to be able to (SPECIAL)…………….in the
future.
23. The children listened (ATTENTION)…………….to the story because it was so interesting.
24. Diana is fun to be with but can be a bit (PREDICT)……………. at times. We never know what kind of
mood she is going to be in.
25. Environmental pollution brings extreme (UGLY) ……………. to man’s naturally beautiful world.
III. Choose the underlined word or phrase in each sentence that needs correcting (5.0 points).
26. “Will you purchase the microwave oven advertising on TV now, Susan?” asked Carol.
A B C D
27. Since his accident last week, Mr. Lee has suffered from constantly headaches.
A B C D
28. The holiday turned out to be so disappointed that they decided to ask for their money back.
A B C D
29. As a matter of fact, the effective of learning with computers depends on many factors.
A B C D
30. The type of jazz known as “swing” was introduced by Duke Ellington when he wrote and records “It
A B C D
Don’ t Mean a Thing If It Ain’t Got That Swing.
D. READING
I. There are five blanks in the passage below. From the options given in the box, choose the MOST
SUITABLE for each blank. There are THREE EXTRA OPTIONS that you do not need to use (5.0
points).
A.certain words B. to simplify C. Directories D. to promote
E. number F. Textbooks G. series of H. grammatical rules
Noah Webster’s goal in life was (31)…………….the adoption of an American language. He wanted
to free Americans from British English as they had freed themselves from the British crown. To this end, he
published a (32)……………. three textbooks: a speller in 1783, a grammar in 1784 and a reader in 1785.
Webster objected to the way (33)…………….had been borrowed from other languages but had not
been respelled. The result, he claimed, was a confusing mixture of letters, many of which were not
pronounced the way they looked, and others of which were not pronounced at all.
Webster urged Americans (34)…………….their spelling. For example, he argued that “head” should
be spelled “hed” and “bread” should be spelled “bred”. Most of Webster’s suggestions did not catch on, but
his (35)…………….sold millions of copies.
II. Read the passage below and decide which option A, B, C or D best fits each space (10.0 points).
Health and fitness are not just for young people. They are for anyone (36)…………….to accept the
discipline for a good diet and (37)……………. exercise. (38)……………. age, there is a tendency to feel
that the body is no longer able to perform. Aches and pains are (39)……………. normal. Instead of (40)
…………….the body to do more, activities become limited. Yet examples after examples have shown us that
older people can - and should - be (41)…………….. Men and women in their sixties have run in marathons,
races of more than twenty-six miles. Some professional athletes stay (42) ……………. into their forties and
fifties. For most people, simple activities like walking and swimming are all that is needed to stay in (43)
……………. . It is important to include exercise in your daily routine. In the winter, (44)……………. push-
ups, sit-ups, and other indoor exercises. Of course, such exercises will be of little use (45)……………. you
follow them with soda and chips.

36. A. suspicious B. willing C. aware D. conscious


37. A. regular B. useful C. abundant D. little
38. A. At B. In C.To D. With

48
39. A. believed B. thought C. hoped D. considered
40. A. making B. pushing C. stopping D. providing
41. A. pessimistic B. active C. hyperactive D. passive
42. A. calm B. competitive C. equal D. comparative
43. A. need B. form C. contact D. shape
44. A. make B. get C. work D. do
45. A. although B. unless C. if D. otherwise
III. Read the following passage and choose the best answer to each question (10.0 points).
Georgia O’Keefe was born in Sun Prairie, Wisconsin, in 1887. She studied art in Chicago and New
York from 1904 to 1908. Beginning as an advertising illustrator, she supported herself until 1918 by teaching
in various schools and colleges in Texas. After that date, she devoted herself entirely to painting. Her
paintings were first exhibited in 1916 at “291”, an experimental art gallery in New York City owned by
Alfred Steiglitz, which was frequented by some of the most influential artists of the time. O'Keefe married
Steiglitz in 1924.
O’ Keefe’s early paintings were mostly abstract designs. In the 1920s she produced depictions of
flowers and precise cityscapes of New York City. Whether painting flowers or buildings, she captured their
beauty by intuitively magnifying their shapes and simplifying their details.
O'Keefe’s style changed dramatically in 1929 during a visit to New Mexico. She was enchanted by
the stark but beautiful landscapes under the bright Southwestern sun, and she then adopted her characteristic
style. Thereafter, she most often painted desert landscapes, often with the blanched skull of a longhorn in the
foreground.
O’Keefe’s paintings were exhibited annually at several New York galleries until 1946, and she is
represented in the permanent collections of most major American museums. In her later years, she settled in
Taos, New Mexico, becoming the dean of Southwestern painters and one of the best known of American
artists.
46. The author wrote this passage in order to …………..
A. give the highlights of Georgia O’Keefe’s artistic career
B. discuss the early career of an important American artist
C. compare abstract art and landscape art
D. criticize Georgia O’Keefe’s style of painting
47. The phrase “that date” in the first paragraph refers to…………...
A. 1887 B. 1908 C.1916 D. 1918
48. According to the passage, Georgia O’Keefe received her formal art training in…………..
A. Texas B. Taos, New Mexico
C. Sun Prairie, Wisconsin D. Chicago and New York
49. The word “frequented” in the first paragraph is closest in meaning to…………..
A. founded B. supported C. favored D. visited
50. Which of the following had the greatest influence on changing O’Keefe’s style of painting?
A. her job as an advertising illustrator
B. meeting influential artists
C. a trip to the Southwest
D. Alfred Steiglitz’s photographs
51. Which of the following became the most common subject of O’Keefe’s after 1929?
A. city scenes B. desert scenes C. flowers D. abstract patterns
52. The word “annually” in the last paragraph means…………..
A. once a week B. once a month C. once a year D. once a century
53. It can be inferred from the passage that, in her later years, O’Keefe…………..success of her early career
A. continued to be successful B. returned to New York City
C. took up photography D. could not match the

49
54. The word “their” in the second paragraph refers to…………...
A. precise cityscapes B. flowers
C. buildings D. flowers and buildings
55. Which of the followings is NOT true, according to the passage?
A. “291” belonged to Alfred Steiglitz.
B. Georgia O’Keefe got married when she was in her thirties.
C. After 1918 Georgia O’Keefe taught in various schools and colleges in Texas.
D. O’Keefe was strongly attracted by the landscapes under the bright Southwestern sun.
E. WRITING
I. Finish each of the following sentences in such a way that it means the same as the sentence printed
before it (5.0 points).
56. Il was such a complicated problem that nobody could solve it.
So……………………………………………………………………………………………………
57. Mr. Jacob only bought the parrot because his son wanted a pet.
If.……………………………………………………………………………………………………
58. Everybody was not surprised to hear that Caroline had not passed her driving test.
It came………………………………………………………………………………………………
59. My sister refused to get the job in spite of the attractive salary.
Attractive ........................................................................................................................................
60. The heavy snow prevented the students from going to the mountain.
The heavy snow made……………………………………………………………………………..
II. Use the suggested words and phrases below to make a complete letter (10.0 points).
Dear Mai,
61. I/ write / tell / holiday.
62. I / be / Kuala Lumpur / two weeks / now / and / I / have / good time / beautiful country.
63. I / spend / first few days / visit / Twin Towers / other / tourist attractions.
64. must / say / Malaysian people / friendly / and / always / ready / help.
65. Most places / Kuala Lumpur / crowded / tourists / over / world.
66. Yesterday / shopping / and / buy / few souvenirs / parents / friends.
67. I / lose / way back / hotel / but / not / matter / because / I / discover / fascinating market / lots / little
stalls / sell / almost everything / needles / elephants.
68. Today /I / be / couple / art galleries / and / meet / interesting people / there.
69.I / guess /I / spend / lot / money / so far, / but / Malaysia / great country / visit.
70.I / take / photos / and / I / show / you / when / get home / June 10th.
See you again.
Trang
III. It is a fact that a lot of Vietnamese students go abroad to study every year. However, studying in a
foreign country has both advantages and disadvantages. To what extent do you agree or disagree with
this statement? Write an essay of about 180 - 200 words to support your opinion (10.0 points).

ĐỀ SỐ 12
(Đề thi vào lớp 10 THPT Chuyên Vĩnh Phúc)
Thời gian làm bài: 150 phút
PART A: LISTENING - TRACK 1 (2.0 points)
Part 1. You are going to listen to a conversation between a university admissions officer and a
prospective student. The recording will be played twice. Questions 1-5. Complete the form below using
NO MORE THAN THREE WORDS AND/OR NUMBERS.
Name: Anne (1)………………….

50
D.O.B: (2)…………………………………1981.
Address: (3)………………………………Simon Place.
Brighton.
Contact number: (4)…………………………
Mobile: (5)……………………………………..

Questions 6-10. Answer each of the following questions using NO MORE THAN THREE WORDS.
6. What is the applicant’s current job?
…………………………………………………………………………………………………………….
7. Which university course has she already completed?
…………………………………………………………………………………………………………….
8. Why might the student not get on the MBA course?
…………………………………………………………………………………………………………….
9. What is her second choice?
…………………………………………………………………………………………………………….
10. Which department will contact the applicant?
…………………………………………………………………………………………………………….
Part 2. Listen to the recording twice and complete the following sentences using ONE word or number
for each gap.
Jim recommends Pagagnini to the whole (1)………………
Jim saw a Pagagnini concert last year in (2)………………
As well as classical music, Pagagnini play rock, (3)………………,country and Western.
Jim says the Pagagnini show has no (4)………………
The first performance of Rhythm of the Dance was in (5)………………in Norway.
More than (6)………………million people have seen Rhythm of the Dance live on stage.
Jim suggests listeners look at the section entitled (7)……………… gallery on the Rhythm of the Dance
website.
The name of the first show that the Cirque Eloize performed at the Regent Theatre was (8)“……………”
Some performers in ID appear on (9)………………and Rollerblades. One review of ID says it is full of
originality, energy and (10)……………….
PART C: LEXICO-GRAMMAR (2.5 points)
I. Write the letter A, B, C or D on your answer sheet to indicate the correct answer to each of the
following questions.
1. How long have you been looking for………………?
A. employee B. employer C. employment D. employ
2. To………………photography professionally requires a lot of skills.
A. do B. make C. commit D. carry
3. We made her an excellent offer, but she………………it.
A. turned B. rejected C. accepted D. denied
4. That old building is going to be………………and a new library will be built in its place.
A. pushed off B. pulled down C. pushed in D. pulled through
5. You are not allowed to bring coffee into the examination room,………………?
A. do you B. aren’t you C. don’t you D. are you
6. When we met that day in Paris, I didn’t know that he was married………………Barbara.
A. with B. to C. in D. of
7. This is………………the most difficult job I have ever tackled.
A. by far B. by the way C. by all means D. by rights
8.………………about gene-related diseases has increased is welcome news
A. Scientific knowledge B. It was scientific knowledge

51
C. Through scientific knowledge D. That scientific knowledge
9. Members of the rock group were asked to modify their behavior………………leave the hotel.
A. or else B. unless C. lest D. in case
10. He………………to the doctor after the accident, but he continued to play instead.
A. should have gone B. shouldn’t have gone
C. needn’t have gone D. must have gone
11. Richard has invited Ann to his study group tonight, but she has refused. What would Richard be most
likely to say in response to Ann’s refusal?
Ann: “I’m afraid I can’t go tonight. I’m just not in the mood for it!”
Richard: “………………!”
A. Off you go B. You are out C. Suit yourself D. Good for you
12. ……………… to the Senate than he began to face some of the realities of being a U.S. senator.
A. No sooner had Obama been elected
B. No sooner Obama had been elected
C. No sooner had Obama elected
D. No sooner Obama had elected
13. Vietnam’s renowned rocker Tran Lap,………………died of cancer on 17 March 2016, will be honored
with a posthumous medal for his contribution to the country’s music industry.
A. that B. who C. whom D. whose
14.………………his brother, Tom is active and friendly.
A. Dislike B. Alike C. Unlike D. Liking
15. Through generations, despite many………………, some Huong Canh families have successfully
preserved their traditional job of clay pot and tile making.
A. out and about B. odds and ends C. ins and outs D. ups and downs
16.………………awful coffee she makes!
A. What B. What a C. How a D. How
17. I told them to keep quiet but they continued………………a noise.
A. to make B. making C. to do D. doing
18. We are going to stay with……………….
A. a relative of we B. relative of us
C. a relative of ours D. one of our relatives’
19. Regular exercise and good diet will bring………………fitness and health.
A. about B. up C. to D. from
20. Mike: “I have bought you a toy. Happy birthday to you!”
Jane: “………………”
A. The same to you. B. Have a nice day!
C. What a pity! D. What a lovely toy! Thanks.
II. Write the letter A, B, C, or D on your answer sheet to indicate the underlined part that needs
correcting.
1. The issue was so controversial, but they were able to settle it in the end of the discussion.
A. was B. but C. able D. in the end
2. Studies by B. F. Skinner indicate that reward positively reinforces behavior and makes that behavior likely
more to recur.
A. Studies by B. reinforces C. and D. likely more
3. A food additive is any chemical that food manufacturers intentional add to their products.
A. additive B. that C. intentional D. products
4. Although the old fisherman was exhausted, but he managed to reel the giant marlin in by nightfall.
A. fisherman B. but C. to reel D. nightfall
5. Long ago there used to be a beautiful island on the Atlantic Ocean.

52
A. on B. Long ago there
C. used to be D. the Atlantic Ocean
III. Complete the following passage with the correct form of the word given in CAPITALS to the right
of each line.

THE KING IN THE CAR PARK


We are used to reading about mysteries in detective novels, but
a real-life mystery was (1)………….solved by archaeologists when RECENT
they found a skeleton under a car park in a British city. The (2)………….of the DISCOVER
remains was of particular (3)………as they turned out to be those of the famous INTERESTING
King of England, Richard III. (4)……., most monarchs have a formal (5) TRADITION
………….and their bones are placed in cathedrals or abbeys, but the final resting BURRY
place of Richard had been (6)………….. The search to find his body had been a KNOW
long one. Now, (7)………….have formally identified the bones as those of Richard, SCIENCE
comparing his DNA with that of another (8)……… Tests have also proven that DESCEND
Richard’s spinal deformity was not as bad as they had
(9)………….thought. However, his reputation of being a ORIGIN
(10)………….still stands and the mystery of what really CRIME
happened to his two nephews remains unsolved.

PART C: READING (3.0 points)


I. Read the following passage and write the letter A, B, C or D on your answer sheet to indicate the
correct answer to each of the following questions.
Icebergs are among nature’s most spectacular creations, and yet most people have never seen one. A
vague air of mystery envelops them. They come into being somewhere-in faraway, frigid waters, amid
thunderous noise and splashing turbulence, which in most cases no one hears or sees. They exist only a short
time and then slowly waste away just unnoticed.
Objects of sheerest beauty, they have been called. Appearing in an endless variety of shapes they may
be dazzlingly white, or they may be glassy blue, green or purple, tinted faintly or in darker hues. They are
graceful, stately, inspiring in calm, sunlit seas. But they are also called frightening and dangerous, and that
they are, in the night, in the fog, and in storms. Even in clear weather one is wise to stay a safe distance away
from them. Most of their bulk is hidden below the water, so their underwater parts may extend out far beyond
the visible top. Also, they may roll over unexpectedly, churning the waters around them.
Icebergs are parts of glaciers that break off, drift into the water, float about awhile, and finally melt.
Icebergs afloat today are made of snowflakes that have fallen over long ages of time. They embody snows
that drifted down hundreds, or many thousands, or in some cases maybe a million years ago. The snows fell
in the polar region and on cold mountains, where they melted only a little or not at all, and so collected to
great depths over the years and centuries.
As each year’s snow accumulation lay on the surface, evaporation and melting caused the snowflakes
slowly to lose their feathery points and become tiny grains of ice. When new snow fell on top of the old, it
too turned to icy grains. So blankets of snow and ice grains mounted layer upon layer and were of such great
thickness that the weight of the upper layers compressed the lower ones. With time and pressure from above,
the many small ice grains joined and changed to larger crystals, and eventually the deeper crystals merged
into a solid mass of ice.
1. The word which in the first paragraph refers to……………..
A. turbulence B. icebergs C. creations D. waters
2. The author states that icebergs are rarely seen because they are……………..
A. surrounded by fog
B. hidden beneath the mountains

53
C. located in remote regions of the world
D. broken by waves soon after they are formed
3. The passage mentions all of the following colors for icebergs EXCEPT……………...
A. yellow B. blue C. green D. purple
4. The attitude of the author toward icebergs is one of……………..
A. disappointment B. humor C. disinterest D. wonder
5. According to the passage, icebergs originate from a buildup of……………..
A. turbulent water B. feathers C. underwater pressure D. snowflakes
6. The word that in the second paragraph refers to……………..
A. bad weather B. icebergs being frightening and dangerous
C. clear weather D. how icebergs look in the seas
7. According to the passage, icebergs are dangerous because they……………..
A. usually melt quickly B. can turn over very suddenly
C. may create immense snowdrifts D. can cause unexpected avalanches
8. In the last paragraph, the expression from above in the fourth paragraph refers to……………...
A. sunlit seas B. polar regions
C. weight of mountains D. layers of ice and snow
9. The word merged in the fourth paragraph in the last paragraph is closest in meaning to……………..
A. became B. combined C. included D. consisted
10. Which of the following is the best title for the passage?
A. The Melting of Icebergs B. The Nature and Origin of Icebergs
C. The Size and Shape of Icebergs D. The Dangers of Icebergs
II. Read the following passage and write the letter A, B, C or D on your answer sheet to indicate the
correct word or phrase that best fits each of the numbered blanks.
CHANGING COUNTRIES
Seeking a new life and hoping for a significant (1)…………. in their standard of living, foreign
workers began flocking into Western Europe during the 1950s. In Britain, some of the first immigrants
arriving (2)………….the West Indies and the Indian subcontinent were welcomed by brass bands, but the
dream of a new life soon (3)…………. sour for many.
Attracted by the promise to earn good money and learn new skills, the reality they found was often
one of (4)………….wages and, in many (5)…………., unemployment. There were times when the
newcomers encountered open hostility; in 1958, riots (6) ………….out in Notting Hill, west London,
when gangs of white youths began taunting immigrants.
Yet despite the (7)…………. . difficulties they encountered, many foreign workers did manage to (8)
………….to their new conditions, settling in their new adopted country and prospering. Their contribution
had the effect not only of speeding up the (9)………….of economic change in the post-war period, it also
(10)…………. Western Europe into a multiracial society.
1. A. switch B. change C. modification D. variation
2. A. from B. to C. in D. at
3. A. turned B. converted C. switched D. moved
4. A. little B. small C. short D. low
5. A. occasions B. examples C. ways D. cases
6. A. broke B. carried C. came D. started
7. A. several B. high C. numerous D. heavy
8. A. fit B. adjust C. match D. suit
9. A. growth B. motion C. pace D. step
10. A. transformed B. transferred C. modified D. shifted
III. Complete the following passage by filling in each blank with ONE suitable word.
THE COMPUTER MOUSE

54
When is a mouse not a mouse? When it sits on your desk and controls your on-screen cursor. The
computer mouse is a pointing device ! (1)……………has a flat bottom, a shaped top with buttons on and -
perhaps - a cable connecting the mouse to the computer. The mouse moves (2)……………the surface of the
desk, and the cursor copies this movement on the screen. The first computer mouse had wheels that made
contact 1(3)……………the working surface. The name ‘mouse’ (4)……………adopted because all earlier
models had a long cord, and this, together with the shape, made it look something (5)……………the furry
little animal of the same name.
The person who invented the mouse, and so was responsible (6)…………… changing the way
computers worked, was an American (7)…………… Douglas Engelbart. In fact, he invented many devices,
including ones attached (8)……………the chin or nose, to point at the screen. However, his computer mouse
soon proved to be much more convenient and far simpler to use. His first attempt (9)…………… producing a
mouse was in 1964, but it was large, heavy and difficult to move. A few years later, in 1968, his improved
mouse (10)…………… its first public appearance and rapidly became a huge success.
PART D: WRITING (2.5 points)
I. Complete each of the following sentences in such a way that it means exactly the same as the one
printed before it. The word provided (if any) must not be altered in any way.
1. Celine has always been fond of classical music. (TASTE)
→ Celine……………………………………………………………………………………….
2. They cannot watch this film because they are not adults yet. (UNDER)
→ Since………………………………………………………………………………………..
3. It’s a pity you didn’t ask us to spend more time with you.
→ If only……………………………………………………………………………………….
4. She hasn’t spoken much since the day she met the President.
→ Little…………………………………………………………………………………………
5. People think that someone started the fire deliberately.
→ The fire………………………………………………………………………………………
II. Essay writing
Vietnamese educators are divided on whether the current system of high schools for the gifted should be
maintained. Whilst some argue that the system may promote unhealthy competition or unnecessary stress
among students, and therefore should be abolished, many others have praised the merits of those schools and
voiced strong approval of their existence.
What advantages can schools for the gifted offer to their students and to the society?
Write 250-300 words.
…………………………………………………………………………………………………………….
…………………………………………………………………………………………………………….
…………………………………………………………………………………………………………….
…………………………………………………………………………………………………………….
…………………………………………………………………………………………………………….

ĐỀ SỐ 13
(Đề thi vào lớp 10 THPT Chuyên Nguyễn Tất Thành - Kon Tum)
Thời gian làm bài: 120 phút
(Học sinh làm bài trên đề thi)
SECTION ONE: LISTENING - TRACK 2 (2.0 pt)
Question 1. You will hear a radio announcer giving details about a contest that is being held. Fill in the
missing information. Write your answers in the numbered box below.

55
PHOTOGRAPHY CONTESTS
Contest is being held by North Counties:…………..Animal Rescue…………..
You should submit photos of (1.)……………………….
Send submissions by email to the address: (2.)……………………….
Photos must be sent by (3.)……………………….
Winners will receive a £ 25 (4.)……………………….
Also the winning photos will be used in a (5.)……………………….

Write your answers here:


Example 1. 2. 3. 4. 5.
Animal Rescue

Question 2. You will hear there people (Wendy, Mrs. Turner and Adrian) discussing a film they have
seen at the cinema. Choose the best answer (A, B, or C) to answer the questions. Write your answers in
the numbered box below.
Example: What does Wendy's mum think of the film?
A. It was not her favorite film.
B. She loved the film.
C. She absolutely hated it.
6. Adrian mentions computer graphics in order to show that……………
A. he wants to change the subject.
B. Mrs. Turner is wrong about the dinosaurs.
C. he knows well what he is talking about.
7. Adrian feels that watching a film at home……………..
A. is always better than going to the cinema.
B. is not something he wants to do again.
C. has advantages and disadvantages.
8. When Adrian suggests that Wendy was frightened, she…………………
A. admits that she felt scared.
B. denies that she felt scared.
C. complains about the length of the film.
9. What is suggested that they do now when the film has ended?
A. go straight home B. go for a drink C. go for dinner
10. Adrian doesn ’t let Mrs. Turner drive because……………….
A. he enjoys driving. B. she will drink alcohol. C. they could get in trouble.
Write your answers here:
Example 6. 7. 8. 9. 10.
A

SECTION TWO: USE OF ENGLISH (3.0 pts)


Question 1. Choose the best answer (A, B, C or D) to finish each of the following sentences. Write your
answers in the numbered box below. (1.0 pts)
11. They received…………….advice from their parents that they became successful.!
A. so good B. such a good C. so good an D. such good
12. She asked me how long……………..
A. I have been waiting B. have I been waiting
C. had I been waiting D. I had been waiting
13. …………….you pay in advance, you must give us a 10 percent deposit.

56
A. Unless B. Provided C. Supposing D. If
14. She never let you cook the meals,…………….?
A. did she B. doesn’t she C. didn’t she D. does she
15. When I have some free time, I tend to do nothing…………….laze about.
A. only B. and C. but D. like
16. Most of the courses at the banquet were completely consumed, but there……………food
still remaining.
A. were few B. was little C. were a few D. was a little
17. The four-storey house…………….on that hill is still new.
A. be built B. building C. built D. being building
18. You should pay ……………to what the instructor is saying.
A. attendance B. attention C. convention D. intention
19. We met our cousin last night. We…………….each other for years.
A. don’t see B. haven’t seen C. hadn’t seen D. didn’t see
20. As a great politician, he always needs to protect his……………..
A. reputation B. publicity C. notoriety D. rumour
Write your answers here:
11. 12. 13. 14. 15. 16. 17. 18. 19. 20.

Question 2. Replace the words in brackets in the following sentences with a suitable phrasal verbs. Put
the verb into the correct tense. (0.5 pts)
doze off cut down on call off
take after go down with show off

Example: What a lovely baby. He certainly……………his father, does he?


21. “I came first in all my exams.” - “Stop……………! You are such a big head” (boast)
22. My father’s tried to …………… the number of cigarettes he smokes but it’s is far too difficult. (reduce)
23. Bill can’t come with us tonight after all. It seems he’s……………a cold. (catch)
24. Because of heavy downpour, the proposed open - air concert had to be……………(cancel)
25. After a heavy meal and a glass or two of wine, I……………in front of the TV. (fall asleep)
Write your answers here:
Example 21. 22. 23. 24. 25.
takes after

Question 3. Read the text below and look carefully at each line. Some of the lines are correct and some
have a word which should NOT be there. If a line is correct, put a tick (√) by the number. If a line has
a word which should NOT be there, write the word in the corresponding numbered boxes. There are 2
examples at the beginning (0 and 00) (1.0 pt)
A LATE NIGHT
Thank you for the book which you had sent me for my birthday last week. I am certain 0. …had…
I will really enjoy reading it as the book is one I have been intending to buy ever since 00…√….
it was being published about two and a half years ago. On Thursday, instead of buying 26………..
for me a present, my sister took me out for a meal in a restaurant which had been 27………..
highly recommended her in a local newspaper. The restaurant, which had been open 28………..
for only two months, was about 25 kilometres away and we went to there in my sister's 29………..
car. The meal was excellent and we thoroughly enjoyed ourselves the evening. 30………..
Unfortunately, after we left the restaurant, we had a problem with the car as that it 31………..

57
would not start. We could not find out what the trouble was and decided to leave it 32………..
behind in the car park until the following day. We asked of a waiter to call a taxi but 33………..
had to wait over an hour for it to arrive, and so we did not get back home until very 34………..
late. I certainly did not feel at all like getting up and going to work next morning. 35………..

Question 4. Give the correct form of the words in brackets. Write your answer irt the numbered box.
(0.5 pt)
0. Thank you for your …………………(GENEROUS)
36. David has been a bit…………………today. (TROUBLE)
37. I have told you on…………………occasions not to leave the safe unlocked. (NUMBER)
38. She was presented with the gift in …………………for her long service. (GRATEFUL)
39. Jack’s parents keep complaining about him because he’s very………………… (OBEY)
40. My boss was so angry that he was absolutely…………………. (SPEECH)
Write your answers here:
Example 36. 37. 38. 39. 40.
generosity

SECTION THREE: READING (3.0 pts).


Question 1: Read the text below and decide which answer (A, B, C, or D) best fits each numbered gap.
Write your answers in the numbered box below. (1.0 pt)
HOW DO YOU CHOOSE THE RIGHT JOB?
A few people decide early in life what they want to do and then follow a prescribed of training to
reach their (41.)………... More often, a young person takes the best job available, and that leads to a life time
in particular field. Of course, both who you know and what you know are important. Frequently, an
individual finds a job through a friend, relative or other (42.)……….. person. However, a university degree,
the minimum requirement for many jobs and advanced training and qualifications is a (43.) ……….. in most
professional fields. What’s more, what you know can be the (44.) ………..to a good job.
There is probably no sing right job for anybody. Out of the (45.)………..20,000 types of occupations
available in modern society, there are probably hundreds that you could perform well and find rewarding.
Although circumstances (46.)………..one’s choices, there are still more possibilities than people realise.
Chance always (47.)………..a part in finding a job you like, but vocational counsellors believe that a person
can improve the (48.)………..by analysing his or her ambitions, interests and (49.)………... One person may
want to earn a lot of money. Others may give priority to being active, helping people or having a low risk of
becoming unemployed. Personal interests, such as love the outdoors or a fascination with computers, may
point the (50.)……….. to a job a person enjoys and respect.
41. A. plan B. goal C. end D. object
42. A. valuable B. beneficial C. helpful D. advantaged
43. A. need B. must C. want D. demand
44. A. ticket B. admission C. direction D. certificate
45. A. expected B. assessed C. estimated D. calculated
46. A. shorten B. press C. shrink D. narrow
47. A. takes B. shorten C. puts D. plays
48. A. odds B. difference C. angles D. favour
49. A. faculties B. aptitudes C. inclinations D. tendencies
50. A. course B. way C. passage D. progress
Write your answers here:
Example 41. 42. 43. 44. 45. 46. 47. 48. 49. 50.
A

58
Question 2: Read the text below and think of ONE word which best fits each gap. Write your answers
in the box numbered below. (1.0 pt)
Regular exercise such as jogging or swimming is good for your heart. It can also give you more
energy to enjoy life. As a (51)…………..of regular exercise, your body gets better at using oxygen. It
becomes easier for your heart to pump blood (52) ………….. your body. After a while, the heart doesn’t
need to work quite as hard. Exercise is often thought to be an easy way of losing weight. But in (53)………,
exercise tends to increase your appetite. Many people discover they lose no weight with exercise alone. (54)
…………..diet and exercise are needed to achieve this. Some people exercise because they think it will help
them to live longer. If that is your reason for exercising, then you (55) ………….. avoid short, intensive
exercise. Squash, for example, (56)…………..is a fast game, may be harmful (57)…………..you’re unfit or
middle-aged. Other sports can be dangerous (58)…………... Although both rugby and football are popular
sports, a rugby player is three (59).………….. more likely to be injured than a tennis player. It is advisable,
(60)…………, to choose a sport that suits you and not one that is going to harm you.
Write your answers here:
51. 52. 53. 54. 55.

56. 57. 58. 59. 60.

Question 3. Read the text and choose the best answer (A, B, C or D) to each of the following questions.
Write your answers in the numbered box below. (1.0 pt)
INTERNET JOBS
Contrary to popular belief, one does not have to be a trained programmer to work online. Of course,
there are plenty of jobs available for people with high- tech computer skills, but the growth of new media has
opened up a wide range of Internet career opportunities requiring only a minimal level of technical expertise.
Probably one of the most well-known online job opportunities is the job of webmaster. However, it is hard to
define one basic job description for this position. The qualifications and responsibilities depend on what tasks
a particular organisation needs a webmaster to perform.
To specify the job description of a webmaster, one needs to identify the hardware and software that
the website the webmasters will manage is running on. Different types of hardware and software require
different skill sets to manage them. Another key factor is whether the website will be running internally (at
the firm itself) or externally (renting shared space on the company servers). Finally, the responsibilities of a
webmaster also depend on whether he or she will be working independently, or whether the firm will provide
people to help. All of these factors need to be considered before one can create an accurate webmaster job
description.
Webmaster is one type of Internet career requiring in-depth knowledge of the latest computer
applications. However, there are also online jobs available for which traditional skills remain in high
demand. Content jobs require excellent writing skills and a good sense of the web as a "new media."
The term "new media" is difficult to define because it encompasses a constantly growing set of new
technologies and skills, specifically, it includes websites, email, Internet technology, CD-ROM, DVD,
streaming audio and video, interactive multimedia presentations, e-books, digital music, computer
illustration, video games, virtual reality, and computer artistry.
Additionally, many of today's Internet careers are becoming paid-by-the job professions. With many
companies having to downsize in tough economic items, the outsourcing and contracting of freelance
workers online has become common business practice. The Internet provides an infinite pool of buyers from
around the world with whom freelancers can contract their services. An added benefit to such online jobs is
that freelancers are able to work on projects with companies outside their own country of residence. How
much can a person make in these kinds of careers? As with many questions related to today's evolving

59
technology, there is no simple answer. There are many companies willing to pay people with technical
Internet skills salaries well above $70,000 a year. Generally, webmasters start at about $30,000 per year, but
salaries can vary greatly. Freelance writers working online have been known to make between $40,000 and
$70,000 per year.
61. The word "identify" in paragraph 2 is closest in meaning to
A. name B. estimate C. discount D. encounter
62. The word "them" in paragraph 2 refers to
A. companies B. new job opportunities
C. hardware and software D. webmasters
63. What does "it" in paragraph 4 refers to?
A. Modern technology B. New media
C. The Internet D. A webmaster's career
64. According to the passage, which of the following is TRUE of webmasters?
A. They never work independently.
B. They require a minimal level of expertise.
C. The duties they perform depend on the organisation they work for.
D. They do not support software products.
65. According to the passage, all of the following are true EXCEPT
A. There are online job available for workers with minimal computer skills.
B. Webmaster must have knowledge of the latest computer applications.
C. Online workers cannot free themselves from the office.
D. "New media" is not easy to define.
66. Which of the following is not mentioned as part of"new media”?
A. websites B. Internet technology C. writing skills D. video games
67. It can be inferred from the passage that
A. online workers can work full-time online
B. only skilled workers make good money
C. it is easy to become a webmaster
D. workers with limited computer skills cannot work online
68. The word "vary " in paragraph 5 could best be replaced by which of the following?
A. change B. decrease C. increase D. differ
69. What is the purpose of the passage?
A. To inform people about the tasks and role of a webmaster
B. To inform people about the computer industry
C. To inform people about employment related to the Internet
D. To explain why webmasters make a lot of money
70. Which of the following is TRUE of the job offreelance writers?
A. They may work with others at the company.
B. They require in-depth knowledge of applications.
C. They manage hardware and software.
D. It is considered a 'content'job.
Write your answers here:
61. 62. 63. 64. 65. 66. 67. 68. 69. 70.

SECTION FOUR: WRITING (2.0 pts)


Question 1. Finish each of the following sentences in such a way that it has a similar meaning to the
original one.0 has been done for you as an example. (0.5 pt)
71. They believe that Barack Obama deeply studied Vietnamese history and culture before the visit to our

60
country.
- Barack Obama……………………………………………………………………………………..
72. In my opinion, it’s better to walk than take a taxi.
- I prefer……………………………………………………………………………………………..
73. It was such lovely party that all of them didn’t want to leave until midnight.
- The party…………………………………………………………………………………………..
74. Linh is the only real friend that my daughter has.
- Except …………………………………………………………………………………………….
75. Susan didn’t say anything to the police until she met her lawyer.
- It was not until……………………………………………………………………………………..
Question 2. Complete the second sentence so that it has a similar meaning to the first one, using the
word given in capital. DO NOT CHANGE THE WORD GIVEN. You must use between TWO and SIX
words, including the word given (0.5 pt)
76. Jim didn’t intend to go to the party at first. (INTENTION)
- Jim………………………………………………………………to the party at first.
77. This old iPad was so badly damaged that it wasn’t worth keeping. (POINT)
- This old iPad was so badly damaged that …………………………………………………keeping it.
78. The police said Jonathan had robbed 2 billion dollars from the bank (ACCUSED)
- The police………………………………………………………………2 billion dollars from the bank.
79. Driving in Hanoi Old Quarter at night is prohibited. (SUPPOSED)
- You ………………………………………………………………in Hanoi Old Quarter at night.
80. When we arrive isn’t really important, as long as we get there. (MATTER)
- It really ………………………………………………………………time we arrive, as long as we get there.
Question 3. In the future, students may have the choice of studying at home using technology with
electronic device like computers, smartphones. They may also choose to go to traditional school. Which
do you prefer? Write a passage of about 180 words, using reasons and examples to support your
position. (1.0 pt)
……………………………………………………………………………………………………………
……………………………………………………………………………………………………………
……………………………………………………………………………………………………………
……………………………………………………………………………………………………………
……………………………………………………………………………………………………………
……………………………………………………………………………………………………………
……………………………………………………………………………………………………………
……………………………………………………………………………………………………………
……………………………………………………………………………………………………………
……………………………………………………………………………………………………………
……………………………………………………………………………………………………………

ĐỀ SỐ 14
(Đề thi vào lớp 10 THPT Chuyên chung Hà Nam)
Thời gian làm bài: 60 phút
Part I. Phonetics (lpoint)
Circle the word whose underlined part is pronounced differently from that of the rest (0.6 point)
1. A. miles B. sleeps C. laughs D. unlocks
2. A. missed B decided C. stopped D. walked
3. A. chair B. children C. machine D. reach

61
Circle the word that has a different stress pattern from the others in the group (0.4point)
4. A.mother B. worker C. invite D. lazy
5. A. along B. thirteen C. enter D. hotel
Your answer:
1. 2. 3. 4. 5.

Part II. Choose the word or phrase which is the best to complete each sentence (2points)
1. Mary visited you last summer,……………?
A. did she B. didn't she C. was she D. wasn't she
2. "I promise I will study harder next semester." - "……………”
A. I hope so B. Good idea C. No, thanks D. I'm sorry I can't
3. William suggested that……………at home for a change.
A. staying B. stay C. they stay D. they stayed
4. We are……………in watching cartoons on TV.
A. interest B. interested C. interesting D. interestingly
5. Thank you for looking……………my children while I was out.
A. for B. up C. after D. at
6. ……………his poor health, he works hard all day.
A. Because B. Although C. In spite of D. Despite
7. Pompeii,……………is an ancient city of Italy, was completely destroyed in A.D.79 by an eruption of
Mount Vesvius.
A. which B. who C. where D. that
8. Last week, I ……………my children to the biggest zoo in town.
A. got B. brought C. fetch D. took
9 Their……………made the party atmosphere warmer.
A. friendly B. friendliness C. unfriendly D. unfriendliness
10. There is……………in my bedroom.
A. an old square wooden table B. a square wooden old table
C. a wooden old square table D. an old wooden square table
Your answer:
1. 2. 3. 4. 5. 6. 7. 8. 9. 10.

Part III. Give the correct tense or form of the verbs in brackets (1 point)
a. You (1. ever/ talk) to a foreigner before? - Yes, I (2. have) a chance to talk with some during my last
vacation in Nha Trang.
b. When I came, he (3.watch) TV.
c. He advised me (4. take) part in this contest.
d. We are going to have our house (5. repaint) next week.
Your answer:
1. 2. 3. 4. 5.

Part IV. Identify the underlined word or phrase that must be changed to make the sentence correct. (1
point)
1. She is so a good student that all her teachers like her.
A B C D
2. I am looking forward to see my closest friend after a long time.
A B C D
3. Jim is more intelligent than his brother does.
A B C D

62
4. Lan wishes that she can go to Thailand to visit her pen pal.
A B C D
5. They made their living by catching fish in ocean every day.
A B C D
Your answer:
1. 2. 3. 4. 5.

Part V. Read the passage and then circle the correct answer each question. (1 point)
English is my mother tongue. Besides, I can speak French and Spanish. I studied the two languages when I
was at high school. Now, I am still learning Spanish at the University. As for me, mastering a foreign
language is not easy. After studying a language, practice is very necessary and useful. Travelling to the
country where the target language is spoken is very helpful, but if you cannot speak the language well
enough you will certainly have troubles. I also frequently go to the movies, watch television, listen to the
radio in the language I am trying to learn. Reading is another good way to learn. Books are good, but I
personally think newspapers and magazines are better. However, getting some knowledge of the language is
the most important thing. Grammar and vocabulary should be mastered first.
1. How many languages can the writer speak?
A. 1 B.2 C. 3 D. 4
2. The writer has learnt Spanish…………...
A. in Spain B. at high school C. at university D. B and C
3. Travelling may cause troubles if…………....
A. you cannot speak the language well enough.
B. you can speak the language well enough.
C. you can speak the language badly enough.
D. you can communicate in the target language.
4. Some useful ways to practice your target language are…………...
A. listening to the radio and watching TV in the language.
B. reading books in the language.
C. seeing films in the language.
D. all are correct.
5. The most important thing is…………...
A. mastering grammar and vocabulary
B. practicing speaking
C. getting some knowledge of the language
D. reading books and newspapers
Part VI. Read the passage and choose the best word to fill in each blank. (2points)
amount burning because pollutes who
of rubbish and green should

One of the reasons, which (1)…………….our environment, is that each household produces a large
amount of (2)…………….every day. In order to reduce the pollution, we need to increase the (3) …………
of rubbish that is recycled (4)…………….we cannot carry on burying and (5)……………. rubbish forever.
This means that we (6)…………….reduce the amount of rubbish we produce and increase the amount we
reuse and recycle. We also need to have a strict punishment for those (7)…………….throw rubbish on the
streets, in the ponds, rivers (8)…………….seas. Besides, we ourselves should be aware (9)…………….
keeping our environment (10)……………., clean and beautiful.
Your asnwer:

63
1. 3. 5. 7. 9.
2. 4. 6. 8. 10.

Part VII. Finish each of the following sentences in such a way that it means exactly the same as the
sentence printed before it. (1 point)
1. Nga said: "I am going to visit my grandmother next weekend.”
→ Nga said that…………………………………………………………………………………..
2. The doctor is working at Bach Mai hospital. You met him yesterday.
→ The doctor whom……………………………………………………………………………….
3. Someone has stolen the picture.
→ The picture ………………………………………………………………………………………
4. In spite of her sickness, Mary insisted on going to work.
→ Although Mary………………………………………………………………………………….
5. He had scarcely put the phone down when the doorbell rang.
→ Scarcely………………………………………………………………………………………….
Part VIII. Complete each of the following sentences, using the words given (1 point)
1. My /brother/ enjoy / play/ football.
……………………………………………………………………………………………………..
2. I/ know/ him/ since/ we/ be/ primary students/.
……………………………………………………………………………………………………..
3. If today/ Sunday, I/ go / fish/ brother/.
……………………………………………………………………………………………………..
4. It/ take/ me/ three hours/ get/ house/ tidy/ yesterday/.
……………………………………………………………………………………………………..
5. You/ should/ apologize/ her/ not/ attend/ wedding/.
……………………………………………………………………………………………………..

ĐỀ SỐ 15
(Đề thi vào lớp 10 THPT Chuyên Lê Hồng Phong - Nam Định)
Thời gian làm bài: 120 phút
PART A. LISTENING - TRACK 3 (2.0 POINTS)
Hướng dẫn phần thi nghe hiểu:
- Nội dung phần nghe gồm 03 phần, mỗi thí sinh được nghe 2 lần, đĩa CD tự chạy 2 lần.
- Mở đầu và kết thúc phần nghe có tín hiệu nhạc.
- Hướng dẫn làm bài chi tiết cho thí sinh (bằng Tiếng Anh) đã có trong bài nghe.
I. PART 1: You are going to hear two women talking about a holiday in France. Listen and choose the
best option A, B, or C to complete the statements about the recording. Put a tick (√) next to the correct
answer. There is an example at the beginning (0). You will hear the conversation TWICE. (0.5 p)
- 0. Paula is talking to…………………
√A. Meg B. Anna C. Peter
1. Meg says that
A. she has been ill.
B. Paula doesn’t look very well.
C. she’s pleased to see Paula.
2. Before the trip, Paula…………………
A. was enthusiastic about it.

64
B. wanted to go to the Lake District.
C. didn’t tell anybody she was going.
3. Before Mark and Paula went to Paris,…………………
A. Mark’s boss didn’t want him to go.
B. Paula arranged for somebody to look after the hamster.
C. Paula’s sister promised to look after the children.
4. The journey across the Channel…………………
A. was very smooth.
B. was unpleasant for Paula.
C. lasted eight hours.
5. The only thing Paula liked about Paris was…………………
A. the weather. B. the accommodation. C. the shops.
II. PART 2: You will hear a woman talking about a sports center. Listen and answer the following
questions with NO MORE THAN TWO WORDS AND/ OR A NUMBER. You will hear the recording
TWICE. (0.5 p)
1. What time does the center close?
……………………………………………………………………………………………………..
2. What kind of sports equipment does it sell?
……………………………………………………………………………………………………..
3. How long do you have to return faulty items?
……………………………………………………………………………………………………..
4. Who can give you advice?
……………………………………………………………………………………………………..
5. When is there a late night opening?
……………………………………………………………………………………………………..
III. PART 3: You are going to hear two separate recordings about the uses of plastics. (1.0p)
Questions 1-5. Complete the sentences below. Write NO MORE THAN THREE WORDS OR A
NUMBER for each answer. You will hear the recording TWICE.
1. According to the speaker, the three factors contributing to the success of plastics today are safety,
…………….. and……………..
2. Meat and other food products are wrapped in plastic to protect them from……………..
3. The use of plastics in major appliances ensures that they will …………….. corrosion.
4. Experts have calculated that a kilo of plastic packaging can lead to a reduction of……………..kilos in
wasted food.
5. Plastic parts have made ……………..as much as fifty per cent more efficient since the 1970s.
Questions 6-10. Complete the notes. Write NO MORE THAN THREE WORDS OR A NUMBER for
each answer. You will hear the recording TWICE.
A history of the plastic bag
1957: The launch of (6)……………….
1966: Around one third of packaging in (7)……………….consists of plastic bags.
(8)………………. 'New York City Experiment': garbage collected in plastic bags.
1973: (9)……………….first manufactured commercially.
2002: New Irish tax of (10)……………….cents per carrier bag.

PART B. GRAMMAR AND VOCABULARY (2.0 POINTS)


I. PART 1: Choose the best word or phrase to complete the following sentences. Put a tick (√) next to
the correct answer A, B, C or D. There is an example at the beginning (0). (1.0 p)
0. Peter tried to avoid………………my questions.
A. answer B. to answer C. answering D. answered

65
1. For the last few months, the south-central coastal provinces ………………significantly less precipitation
and prolonged drought.
A. have experienced B. experience
C. are experiencing D. have been experienced
2………………my friends knew I was going to study overseas.
A. Not much of B. Not many of C. Not a lot of D. Not many
3. There was a terrible accident last night. When I arrived at the scene,………………injured had been taken
away.
A. the B. an C. one D. some
4. Nobody wants to be the first to make a sacrifice,………………?
A. does she or he B. does anybody C. do we D. do they
5. He always………………the crossword in the newspaper before breakfast.
A. writes B. makes C. does D. works
6. I know her………………sight, but I’ve never talked to her.
A. in B. at C. on D. by
7. "How about going fishing this Sunday?" - "………………”
A. That's a good idea. B. Never mind.
C. Yes, 1 am fishing. D. That’s my pleasure.
8. ………………is more interested in rhythm than in melody is apparent from his compositions.
A. Philip Glass B. Philip Glass, who
C. That Philip Glass D. Because Philip Glass
9.I don’t see any………………in arriving early at the theatre.
A. cause B. point C. reason D. aim
10. You will have to………………your holiday if you are too ill to travel.
A. put out B. put off C. put down D. put up
II. PART 2: Each of the following sentences has ONE mistake. Underline the mistake and write the correction in the
provided blank on the right. There is an example at the beginning (0). (0.5 p)
Sentences Corrections
0. He collided with a car because he was driving too fastly. fast
1. You really must be more careful when you do your exercises because you did six
mistakes in this one.
2. Instead of being exciting about the news, she was indifferent to it.
3. The exhibition was the centre of attraction like it was of historical interest.
4. Each of the cars in the showroom was quickly sold to their new owner.
5. I gave to Susan the address so that she could contact me.

III. PART 3: Give the correct form of the words in capital letters. Write your answers in the blanks.
There is an example at the beginning (0). (0.5p)
0. If you get a burn, cool it (IMMEDIATE)….immediately….
1. Robbers broke into the art gallery and took away many (VALUE)………………paintings.
2. George and I have been friends since (CHILD)………………. He used to live next door.
3. John lost his job because he often behaved (POLITE)………………towards his customers.
4. Peter fell off the ladder, but his………………(INJURE) were not very serious.
5. Hoang Anh was………………(COURAGE) to apply for the job.
PART C. READING (3.0 POINTS)
I. PART 1: Read the following passage and answer the questions. For question 1-4, put a tick (√) next
to the correct answer A, B, C or D. For question 5-8, write your answers in the spaces provided (you
can give short but enough information answers). (0.8p)
In the world today, particularly in the two most industrialized areas, North America and Europe,

66
recycling is the big news. People are talking about it, practicing it, and discovering new ways to be sensitive
to the environment. Recycling means finding ways to use products a second time. The motto of the recycling
movement is “Reduce, Reuse, Recycle”.
The first step is to reduce garbage. In stores, a shopper has to buy products in blister packs, boxes and
expensive plastic wrappings. A hamburger from a fast food restaurant comes in lots of packaging: usually
paper, a box and a bag. All that packaging is wasted resources. People should try to buy things that are
wrapped simply, and to reuse cups and utensils. Another way to reduce waste is to buy high-quality products.
When low-quality appliances break, many customers throw them away and buy new ones - a loss of more
resources and more energy. For example, if a customer buys a high-quality appliance that can be easily
repaired, the manufacturer receives an important message. In the same way, if a customer chooses a product
with less packaging, that customer sends an important message to the manufacturers. To reduce garbage, the
throw-away must stop.
The second step is to reuse. It is better to buy juices and soft drinks in returnable bottles. After
customers empty the bottles, they return them to the stores. The manufacturers of the drinks collect bottles,
wash them, and then fill them again. The energy that is necessary to make new bottles is saved. In some parts
of the world, returning bottles for money is a common practice. In those places, the garbage dumps have
relatively little glass and plastic from throw-away bottles.
The third step being environmentally sensitive is to recycle. Spent motor oil can be cleaned and used
again. Aluminum cans are expensive to make. It takes the same amount of energy to make one aluminum can
as it does to run a color TV set for three hours. When people collect and recycle aluminum (for new cans),
they help save one of the world’s precious resources.
1. What is the topic of the passage?
A. How to live sensitively to the environment.
B. How to reduce garbage disposal.
C. What is involved in the recycling movement.
D. What people understand the term “recycle”.
2. What does the word “sensitive” means?
A. cautious B. logical C. responding D. friendly
3. What best describe the process of reuse?
A. The bottles are filled again after being returned, collected and washed.
B. The bottles are collected, washed, returned and filled again.
C. The bottles are washed, returned filled again and collected.
D. The bottles are collected, returned, filled again and washed.
4. The word “practice” is closest in meaning to…………..
A. training B. exercise C. deed D. belief
5. What are the two most industrialized areas in the world today?
……………………………………………………………………………………………………..
6. What kind of products should people buy to reduce waste?
……………………………………………………………………………………………………..
7. What happens to the garbage dumps in the areas where each returned bottle is paid?
……………………………………………………………………………………………………..
8. What are the two things mentioned as examples of recycling?
……………………………………………………………………………………………………..
II. PART 2: Read the text below and decide which answer (A, B, C or D) best fits each space. Put a tick
(√) next to the correct answer. There is an example at the beginning (0). (0.8 p)

There can be no (0)……………at all that the Internet has made a huge difference to our lives. Parents are
worried that children spend too much time playing on the Internet, hardly (1)……………doing anything else
in their spare time. Naturally, parents are (2)……………to find out why the Internet is so attractive and they

67
want to know if it can be harmful for their children. Should parents worry if their children are spending that
much time (3)……………their computers?
Obviously, if children are bent over their computers for hours, (4)……………in some game, instead of doing
their homework, then something is wrong. Parents and children could decide how much use the child should
(5)……………of the Internet, and the child should give his or her word that it won’t interfere with
homework. If the child is not holding to this arrangement, the parent can take more drastic (6)…………….
Dealing with a child’s use of the Internet is not much different from negotiating any other sort of bargain
about behavior.
Any parent who is seriously alarmed about a child’s behavior should make an appointment to (7)……………
the matter with a teacher. Spending time in front of the screen does not necessarily affect a child’s
performance at school. Even if a child is (8)…………… crazy about using the Internet, he or she is probably
just going through a phase, and in a few months there will be something else to worry about!
0. A. reason B. doubt C. purpose D. motive
1. A. always B. rarely C. never D. ever
2. A. worried B. concerned C. curious D. hopeful
3. A. staring at B. stare at C. stared at D. to stare at
4. A. supposed B. occupied C. excited D. absorbed
5. A. do B. have C. make D. create
6. A. rules B. procedures C. regulations D. steps
7. A. speak B. discuss C. talk D. say
8. A. absolutely B. more C. less D. few
III. PART 3: Read the following passage and do the tasks that follow. (0.8 p)
A. Famous for its beautiful setting, San Francisco is built on a series of steep hills located on the northern tip
of a peninsula at the entrance to San Francisco Bay. The bay and its extensions, constitute one of the great
natural harbors of the world, embracing nearly 1,200 sq km of water. Because of this, San Francisco was
once the major Pacific Coast seaport of the United States. Today the city is an important center for finance,
technology, tourism, and culture.
B. San Franciscans, and in some cases their counterparts in the Bay Area, have successfully undertaken
mammoth construction projects such as the San Francisco-Oakland Bay Bridge, the Golden Gate Bridge, and
the Bay Area Rapid Transit system. Since at least the 1950s, San Franciscans have also earned a reputation
for tolerance of and respect for diversity.
C. The most serious social problems facing the city are not unique to San Francisco, but some have taken on
greater dimensions in the city than they have elsewhere. One such problem is homelessness. From 1988 to
1992, the plaza in front of city hall became an encampment for homeless people, rendering other use
impossible and raising public health concerns. The problem of homelessness persists despite the efforts of
city agencies and private charities to provide shelter, health care, and drug, alcohol, and mental health
treatment. In the mid-and late 1990s mayors Frank Jordan and Willie Brown both sought to discourage
homeless people from living in public space in the downtown area and, in Brown’s case, in Golden Gate
Park. However, residents of other areas complained that because of these projects, the displaced homeless
had moved into their neighborhoods.
D. In other areas the city has made some progress toward addressing social problems. As was true across
much of the nation, the crime rate in San Francisco dropped in the 1990s, as did the rate of drug-related
violence. In addition, some public housing projects in San Francisco that were especially prone to violence
and drug-related activity were razed and rebuilt with designs considered less likely to encourage those
activities. Other public housing projects received stepped up security patrols.
E. Some social critics have pointed to an increasing economic and social polarization of San Francisco’s
population. Those who work in finance or high- tech fields are increasingly affluent, pushing rents and home
prices to among the highest levels in the nation. At the same time, people who labor in the service sector
often work for the minimum wage, cannot share the affluent lifestyles around them, and are hard-pressed to

68
afford rising rents. This economic polarization coincides in part with ethnic and educational patterns.
Workers in the low-wage end of the service sector are likely to have limited English proficiency and a high-
school education or less; many workers in those areas are also disproportionately African American and
Hispanic. By contrast, those people who work in the finance and high-tech sectors are more likely to be white
or Asian American and to have one or more college degrees.
Question 1-5: The five paragraphs of the Reading Passage are lettered A-E.
Choose the most suitable headings for paragraphs A-E from the list of headings below.
NB There are more headings than paragraphs, so you will not use them all.
1. Paragraph A…………..List of headings
2. Paragraph B…………..i. The problems in San Francisco.
3. Paragraph C…………..ii. Geographic characteristics of San Francisco.
4. Paragraph D…………..iii. Ethnic and education patterns in San Francisco
5. Paragraph E…………..iv. San Francisco’s accomplishments.
v. The problem of homelessness in San Francisco.
vi. The beauty of San Francisco
vii. Public security improvements in San Francisco.
viii. The problem of social economic polarization.

Questions 6-8: Do the following statements agree with the information given in the Reading Passage?
In the spaces provided, write:
TRUE if the statement agrees with the information.
FALSE if the statement contradicts the information.
NOT GIVEN if there is no information on this information.
6………..San Francisco has achieved tremendous successes in construction
7………..City agencies and private charities have succeeded in addressing the problem of homelessness.
8………..The increasing economic polarization in San Francisco has no relation to ethnic and educational
patterns.
IV. PART 4: Fill each of the following numbered blanks with ONE suitable word and write your
answers in the corresponding numbered boxes. There is an example at the beginning (0) (0.6p)
Everyone wants (0)….to….reduce pollution. But the pollution problem is as complicated as it is serious. It is
complicated because much pollution (1)…………..caused by things that benefit people. For example,exhaust
(2)………….. automobiles causes a large percentage of all air pollution. But the automobiles provide
transportation for millions of people. Factories (3)…………..much of the material which pollutes air and
water, but factories give employment to a large number of people.
Thus, to end or greatly reduce pollution immediately, people would have to stop using many things (4)……..
benefit them. Most of the people do not want to do that, of course. But pollution can be gradually reduced in
several ways. Scientists and engineers can work to find ways to lessen the (5)………..of pollution that such
things as automobiles and factories cause. Governments can pass and enforce laws that require businesses
and individuals to stop, or cut (6) ………….. on certain polluting activities.
PART D. WRITING (3.0 POINTS)
I. PART 1:
Question 1-5: Finish each second sentence so that it has similar meaning to the first sentence, using the
clues at the beginning of each sentence. There's an example at the beginning (0). (0.5p)
0. It takes him at least three hours a day to chat on facebook.
→ Everyday he spends at least three hours chatting on facebook .
1. As soon as we had finished dinner, the thunderstorm broke.
→ Hardly ……………………………………………………………………………………….
2. It’s thought that he was staying in London at the time of the assault.
→ He is………………………………………………………………………………………….

69
3. President Obama was surprised when he saw the hospitability of Vietnam government and people.
→ It came……………………………………………………………………………………….
4. He always has his nose in a book and never pays attention to what I say.
→ If he didn't …………………………………………………………………………………..
5. Martin may not be very well but he still manages to enjoy life.
→ Martin's poor…………………………………………………………………………………
Question 6-10: Complete the second sentence using the word given so that it has a similar meaning to
the following sentence. Write between two and five words in the space provided on the answer sheet.
Do NOT change the word given in brackets in any way. There’s an example at the beginning (00).
(0.5p)
00. David praised her exceptionally good choice of venue for the party. (CONGRATULATED)
→ David……congratulated her on…… a good venue for the party.
6. Lisa was just about to leave the house when the phone rang. (POINT)
→ Lisa was………………………………………………….the house when the phone rang.
7. A number of sporting events had to be cancelled owing to the bad weather. (LED)
→ Bad weather…………………………………………………of a number of sporting events.
8. When it comes to computer games, Gareth is a real expert. (CONCERNED)
→ As far as …………………………………………………, Gareth is a real expert.
9. I’ve been greatly impressed by the way Jasper deals with problems. (MADE)
→ Jasper’s way of dealing with problems has………………………………………………me.
10. “Is Peter likely to change his mind?” Rob asked. (CHANCE)
→ “Is there ……………………………………………………changing his mind?” Rob asked.
Question 11-15: Use the given word to write a new sentence as similar as possible in meaning to the
original sentence. Do NOT change the word given in brackets in any way. There’s an example at the
beginning (000). (0.5p) 000. Turning down that job was very foolish of you. (FOOL)
→ You were a fool to turn down that job.
11. This mix-up is not my fault. (BLAME)
→…………………………………………………………………………………………………
12. My brother is not feeling terribly well these days. (WEATHER)
→…………………………………………………………………………………………………
13. I'd like to express my thanks for everything you’ve done for me (THANKFUL)
→…………………………………………………………………………………………………
14. On no account will I lend you $500. (QUESTION)
→…………………………………………………………………………………………………
15. His smooth manner didn’t deceive us. (TAKEN)
→…………………………………………………………………………………………………
II. PART 2: Writing an argument (1.5 pts)
Nowadays, most students take extra classes. Do you think this is a good idea? Or would it be better for
students to invest all that time in self-study?
Write a paragraph of about 150 words. Support your points with examples and relevant evidence.
…………………………………………………………………………………………………….
……………………………………………………………………………………………………
…………………………………………………………………………………………………….
…………………………………………………………………………………………………….
…………………………………………………………………………………………………….
…………………………………………………………………………………………………….
…………………………………………………………………………………………………….
…………………………………………………………………………………………………….
…………………………………………………………………………………………………….

70
D. ĐỀ THI NĂM 2017 - 2018
ĐỀ SỐ 16
(Đề thi vào lớp 10 THPT Chuyên Quốc Học Huế)
Thời gian làm bài: 120 phút
I. LISTENING - TRACK 4 (20 points)
The listening test has TWO parts. You will hear each part TWICE. At the beginning of each part, you
will have time to read the questions and at the end of each part, you will have time to complete your
answers.
Part 1: (10 points)
Questions 1-4
Listen to the recording of three students talking to their tutor about the presentation they are
planning. Circle the best letter among A, B, or C.
1. The topic of the presentation is
A. how mobile phones are designed.
B. the risks caused by mobile phones.
C. how mobile phones are used.
2. The introduction explains the
A. dangers of mobile phones.
B. importance of mobile phones.
C. importance of understanding the dangers of mobile phones.
3. On the second slide, the students are planning to
A. point out some different kinds of risks.
B. explain why mobile phones are dangerous.
C. mention ways to avoid the risks.
4. The tutor suggests
A. not discussing the dangers of mobile phones.
B. discussing the benefits of mobile phones.
C. having an argument.
Questions 5-7
Listen to the next part of the recording and complete the information for pnch slide. Write ONE
WORD ONLY.
Slide 1 Slide 2 Slide 3
Introduction Health (7)…………….
(5)…………….of using mobile (6)……………. for avoiding dangers
phones Security

Questions 8-10
Listen to the last part of the recording and complete the sentences. Write NO MORE THAN TWO
WORDS AND/OR A NUMBER.
8. The actual talk will last for …………….
9. Each student will speak for…………….
10. The slides must all have the…………….
Part 2: (10 points)
You will hear a woman called Yvonne on a TV programme giving her opinion about children being
punished at school. For questions 1 - 10, complete the sentences with NO MORE THAN THREE

71
WORDS.
The strap was a long piece of leather made especially for (1)…………….children’s palms.
Today, children who misbehave at school seldom even get a (2)…………….
In the fifties, Yvonne was strapped for coming to school in (3)…………….
Yvonne thought the way she was disciplined at school was (4)……………..and unfair.
The members of the organization P.O.P.P.I. all had (5)…………….
In 1979, because of P.O.P.P.I., (6)…………….made the strap illegal.
Yvonne describes her children as (7) ……………. and irresponsible.
Yvonne does not think her children understand (8) …………….they are.
She is now sorry that the government (9) …………….
She believes that there would be less (10)…………….if the strap was still used.
II. VOCABULARY AND GRAMMAR (10 points)
For each of the following questions, circle the best answer (A, B, C or D) to fill the blank.
1. The noise got……………as the car disappeared into the distance.
A. smaller B. slighter C. weaker D. fainter
2. She……………her neighbor’s children for the broken window.
A. accused B. blamed C. denied D. complained
3. To a large extent, slavery was……………during the past century.
A. prevented B. abolished C. uprooted D. removed
4. Despite the storm, the plane……………down at Heathrow Airport as planned.
A. landed B. took C. touched D. flew
5. Could you……………this parcel to my parents when you come to Rome?
A. bring B. take C. fetch D. carry
6. He looked……………with such a big scar on his face.
A. frightening B. frightened C. frightenedly D. frighteningly
7. Before you start cooking, you should gather together all the necessary
A. ingredients B. factors C. substances D. elements
8. The actor got his moustache shaved so as to avoid……………
A. being recognized B. to be recognized C. recognizing D. be recognized
9. My classmate,……………father has been in hospital for a month, looked depressed this morning.
A. which B. who C. of whom D. whose
10. We all have to follow the rules, and none of us is……………the law.
A. beyond B. over C. above D. onto
11. This is……………the most difficult job I’ve ever had to do.
A. by heart B. by chance C. by far D. by myself
12. They asked me a lot of questions,……………I couldn’t answer.
A. neither of them B. most of them C. neither of which D. most of which
13. Be patient; ……………you won’t succeed.
A. otherwise B. or C. because D. unless
14. Peter is a good teacher who taught me……………Maths and……………Physics at secondary school.
A. the/the B. a/the C. the/a D. Ø/Ø
15. Let your younger brother talk first,……………?
A. will you B. would you C. shall we D. do you
16. It is essential that she……………her English for the course she is going to do at Queensland University
of Technology.
A. improves B. improve C. improving D. improved
17. Why don’t we emigrate? If we……………in Australia, at least the weather…………… better.
A. lived/would be B. didn’t live/is C. live/would be D. live/will be
18. “It is said that these herbs are good for you.” has the same meaning as……………

72
A. “These herbs are said to do very good.”
B. “These herbs is said to do you good.”
C. “These herbs are said are good for you.”
D. “These herbs are said to do you good.”
19. After graduating from university in Vietnam, she went on……………for a Ph.D. course at an Australian
University.
A. applying B. apply C. Applied D. to apply
20. Jeff is calling Alice.
- Jeff: “Hello? Alice? This is Jeff. How are you?”
& - Alice: “Jeff? What a coincidence! I……………about you when the phone rang.
A. have just been thinking B. just thought C. was just thinking D. was just thought
III. READING (25 points)
Part 1: Read the following passage, choose and circle the best answer (A, B, C or D) to fill in each of
the blanks. (10 points)
PROOF THAT SILENCE IS GOLDEN FOR STUDYING
The combination of music and study has long been a source of disagreement between adults and children.
Parents and teachers alike maintain that silence is important when learning, (1)……………youngsters insist
that their favorite sounds help them concentrate.
Now a study shows that the grown-ups have been (2)……………all along. Psychologists in Florida tested
how fast students wrote essay with and without music in the (3)……………. They found that the sounds
slowed progress down by about sixty words per hour. “This demonstrates clearly that it is difficult to (4)
……………with listening and writing at the same time”, said Dr. Sarah Randall. She also (5)……………to
the conclusion that it is a myth that instrumental music is less distracting than vocals. “All types of music (6)
……………the same effect”, she said in her report. “One’s ability to pay attention and write fluently is likely
to be (7)……………by both vocal and instrumental music”, she added.
Dr. Randall claimed the research (8)……………that the idea that music could improve performance was
wrong. “Writing an essay is a complex (9)……………You are recalling information and putting it in (10)
……………. An additional stimulus in the form of music is bound to distract. But music is not the only
distractor. What is particularly worrying is that more and more teenagers are studying in front of the
television”.
1. A. whereas B. Unlike C. besides D. despite
2. A. precise B. Right C. valid D. frue
3. A. setting B. background C. surrounding D. circumstances
4. A. manage B. support C. cope D. stand
5. A. reached B. drew C. arrived D. came
6. A. made B. had C. brought D. kept
7. A. disturbed B. interfered C. bothered D. shocked
8. A. pointed B. displayed C. demonstrated D. presented
9. A. project B. concern C. scheme D. task
10. A. order B. arrangement C. line D. pattern
Part 2: Read the text below and think of the word which best fits each space. Use only ONE word in
each space. (10 points)
KEEPING FIT
Bodies are made to move! They are not designed for sitting around in front of the television or reading
magazines. Keeping fit doesn’t (1)…………..you have to be a super-athlete, and even a (2) …………..
exercise can give a lot of fun. When you are fit and healthy, you will find you look better and feel better.
You’ll have more energy and self-confidence.
Every time you move you are exercising. The human body is designed to blend, stretch, run, jump and climb.
The (3)…………..it does, the stronger and fitter it will become. Best of (4) ………….., exercise is fun. It’s

73
(5) …………..your body likes doing most - keeping on the move.
Physical exercise is not only good for your body. People who take regular exercise are usually happier, more
relaxed and more alert (6) …………..people who sit around all day. Try an experiment - next time you’re (7)
…………..a bad mood, go for a walk or play a ball game in the park. See how (8) …………..better you feel
after an hour.
A sense of achievement is yet (9) …………..benefit of exercise. People feel good about themselves when
they know they have improved their fitness. People who exercise regularly will (10) …………..you that they
find they have more energy to enjoy life. So have a go; you’ll soon see and feel the benefits!
Part 3: You are going to read a newspaper article about the problems caused by acid rain. Six
sentences have been removed from the article. Choose from the sentences A-G the one which fits each
gap (0-5). Number (0) has been done for you. There is one extra sentence which you do not need to use.
(5 points)
RUINING THE RUINS
Acid rain is now a familiar problem in the industrialized countries in Europe. Gases like sulphur dioxide and
nitrogen oxide are produced by power stations and cars. (0)….E….
Acid rain is also capable of dissolving some rocks, and buildings made of soft rock, such as limestone, are
particularly badly affected. The acid rain attacks the rocks, and so carvings and statues are eroded more
quickly.
(1)…………... According to a report in the New Scientist, acid rain is being blamed for the rapid decay of
ancient ruins in Mexico. The old limestone buildings in places like Chichen Itza and Palenque are wearing
away very quickly indeed. These sites are the remains of the buildings built by the Mayas between
250 BC and AD 900, and the spectacular ruins of Mayan civilization are visited by thousands of tourists
every year. But those ruins are in danger of being seriously damaged by pollution. At many sites the stone
has been covered with a layer of black substance like tar. (2) …………...
Scientists estimate that about one millimeter of stone is worn away every twelve years. (3)…………... The
acid rain is said to be caused by pollution from oil wells in the Gulf of Mexico. Car exhaust gases are also a
problem. Local volcanic eruptions make the problem even worse. Nevertheless, with enough money and
effort, researchers say that many of the problems could be solved and the rate of erosion reduced. (4) ………
Mexico’s current lack of funds is also partly due to oil. The country has rich oil fields and a few years ago,
when oil was expensive, Mexico was selling large quantities of oil to the USA and earning a lot of money.
(5) …………..However, the price of oil then dropped, and Mexico has been left owing enormous sums of
money and with not enough income from oil sales to pay back the loans. So unless the price of oil rises, it is
unlikely that Mexico will be able to afford to clean up the pollution and save its Mayan ruins from
destruction.
A. At others the painted surfaces inside temples are lifting and flaking off and the stone is being eaten away.
B. That is enough to have caused some of the ancient carvings to become seriously damaged already.
C. These measures would reduce the pollution, but would not stop it completely.
D. The government was therefore able to borrow huge sums of money from banks around the world, thinking
they would have no problem repaying their debts.
E. They dissolve in rainwater and this makes acid rain, which damages trees, river and streams.
F. The problem, however, is not just a European one.
G. However, the Mexican government does not have enough money to do the work, and needs to spend what
money it has on the Mexican people.
IV. WRITING (25 points)
Part 1: For each question, complete the second sentence in such a way that it is as similar in meaning
as possible to the one printed before it. (5 points)
1. The number of road accidents has soared in recent months.
→There………………………………………………………………………………………………..
2. People no longer smoke so many cigarettes as they used to.

74
→ The smoking……………………………………………………………………………………….
3. I don't intend to apologize to either of them.
→ I have.………………………………………………………………………………………………
4. Nobody expected her to lose, but she did.
→Against………………………………………………………………………………………………..
5. I'm more interested in the people than the job.
→It's not the …………………………………………………………………………………………...
Part 2: For each question, complete the second sentence in such a way that it has the closest meaning to
the original one, using the WORD given. DO NOT change this word. You must use between THREE
AND FIVE WORDS, including the word given. (5 points)
1. I found it difficult to follow the instructions. TROUBLE
→ I………………………………………………………………………………the instructions.
2. The neighbors’ all-night parties were too much for me, so I moved. PUT
→ I moved because I…………………………………………………the neighbors’ all-night parties.
3. Nigel should leave soon if he’s catching the 8.30 train. HAD
→ Nigel…………………………………………………………………if he’s catching the 8.30 train.
4. Things are much more expensive here than we thought they would be. EXPECTED
→ We had………………………………………………………………………………cheaper here.
5. The news was such a shock to them that they all sat there without saying a word. SILENCE
→ They all sat there…………………………………………………………so shocked by the news.
Part 3: Within from 200 to 250 words, write a short composition on the following topic. (15 points)
Should teenagers spend more time on such cultural activities as enjoying music or theatre, etc and less
time on sports? Why or why not?
…………………………………………………………………………………………………………….
…………………………………………………………………………………………………………….
…………………………………………………………………………………………………………….

ĐỀ SỐ 17
(Đề thi vào lớp 10 THPT Chuyên Vĩnh Phúc)
Thời gian làm bài: 120 phút
PART A. LEXICO-GRAMMAR
I. Write the letter A, B, C or D on your answer sheet to indicate the correct answer to each of the
following questions.
1. Some people show…………… attitude toward the misery of others, totally untouched by their suffering.
A. passionate B. dispassionate C. passion D. passionately
2. I enjoy walking to school, but on rainy days I…………… to going by bus.
A. would rather B. resort C. commit D. prefer
3. This is a photograph of the school I……………when I lived in Swindow.
A. went B. studied C. attended D. joined
4. The train was……………by a heavy snowfall.
A. held out B. held off C. held up D. held back
5. The people in my class,……………are very friendly.
A. most of international students B. the most international students
C. almost international students D. mostly international students
6. Doctors advise people who are deficient……………Vitamin C to eat more fruit and vegetables.
A. from B. for C. of D. in
7. The teacher made a difficult question, but at last, Joe ……………a good answer.

75
A. came up against B. came up with C. came up to D. came up for
8.……………at his lessons, still he couldn't catch up with his classmates.
A. Hard as he was B. Hard as he does
C. Hardly as he worked D. Hard as he worked
9. He refused to give up work,……………he had won a million dollars.
A. despite B. even though C. however D. as though
10. Jane wasn't in when I arrived. I suppose she……………I was coming.
A. may forget B. can't have forgotten
C. must forget D. must have forgotten
11. Not for a moment……………the truth of the case.
A. he doubted B. was he doubting C. he did doubt D. did he doubt
12. The student……………the highest score will be awarded a scholarship.
A. receives B. who is receiving C. who receive D. receiving
13. On leaving prison, Vic decided to turn over a new……………and to give up his old life of crime.
A. leaf B. book C. chapter D. page
14. He’s been very sick. His doctor insisted that he……………in bed this week.
A. stays B. stay C. would stay D. will stay
15. According to some historians, if Napoleon had not invaded Russia, he……………the rest of the world.
A. would have conquered B. conquered C. had conquered D. would conquer
16. He was so mean that he could not bear to……………the smallest sum of money for the charity appeal.
A. part with B. let out C. give in D. pay off
17. We……………for three hours and are very tired.
A. have been walking B. are walking
C. were walking D. had been walking
18. ……………weather! We can't go out for a walk now.
A. What a terrible B. How a terrible C. What terrible D. How terrible
19. Susan: "How much do you earn, Joe?"
Joe: “I'd……………”
A. rather don't say B. rather not say C. better not to say D. prefer not say
20. Anna: "You don't have to go to school this afternoon, do you?"
Bob: “……………”
A. No, I needn't B. What does that mean?
C. Yes, I don't have to D. That’s OK
II. Write the letter A, B, C, or D on your answer sheet to indicate the underlined part that needs
correcting.
1. Nobody had known before the presentation that Sue and her sister will receive the awards for outstanding
scholarship.
A. had known B. the C. will receive D. scholarship
2. Many of the characters portrayed by writer Joyce Carol Oates is mentally ill.
A. Many of B. portrayed C. is D. mentally
3. The extent to which an individual is a product of either heredity or environment cannot proven, but several
theories have been proposed.
A. to which B. cannot proven C. several theories D. have been
4. An octopus has three hearts to pump blood throughout their body.
A. An B. to pump C. throughout D. their
5. Since vitamins are contained in a wide variety of foods, people seldom lack of most of them.
A. are B. in a C. variety of D. lack of
III. Complete the following passage with the correct form of the word given in CAPITALS.
MY FAVORITE PLACE

76
This may be a surprising choice as it’s not comfortable or obviously (1. APPEAL)…………….. I’m sure
many people gravitate towards the (2.GRAND) ……………..of Venice or the beauty of Sydney but the place
that does it for me is a remote valley in Iceland. Far from anything man-made, it was created by a
(3. VIOLENCE) …………….catastrophe. It would have been a challenging place to live under any
circumstances but thousands of years ago a volcanic (4. ERUPT) ……………..under a glacier caused a flood
that carved out a huge canyon. (5. TOWER) …………….. walls of rock on either side protect the valley
from the (6. STRONG) ……………..of the ferocious Arctic winds. Here a forest has grown up in a (7.
SHELTER) ……………..area of calm. I find that it has its own (8. ATMOSPHERE) ……………..identity,
which some may find (9. PLEASE) ……………..or even threatening. However, it draws me back time after
time. I stay in the campsite and it gives me new perspective on my everyday life. It makes me appreciate the
formidable power and (10.ESCAPE) ……………..force of nature.

I. Read the following passage and write the letter A, B, C or D on your answer sheet to indicate the
correct answer to each of the following questions.
In 1900, the United States had only three cities with more than a million residents- New York, Chicago, and
Philadelphia. By 1930, it had ten giant metropolises. The newer ones experienced remarkable growth, which
reflected basic changes in the economy. The population of Los Angeles (114,000 in 1900) rose spectacularly
in the early decades of the twentieth century, increasing a dramatic 1,400 percent from 1900 to 1930.
A number of circumstances contributed to the meteoric rise of Los Angeles. The agricultural potential of the
area was enormous if water for irrigation could be found, and the city founders had the vision and dating to
obtain it by constructing a 225-mile aqueduct, completed in 1913, to tap the water of the Owens River. The
city had a superb natural harbor, as well as excellent rail connections. The climate made it possible to shoot
motion pictures year-round; hence Hollywood. Hollywood not only supplied jobs; it disseminated an image
of the good life in Southern California on screens all across the nation. The most important single industry
powering the growth of Los Angeles, however, was directly linked to the automobile. The demand for
petroleum to fuel gasoline engines led to the opening of the Southern California oil fields, and made Los
Angeles North America's greatest refining center.
Los Angeles was a product of the auto age in another sense as well: its distinctive spatial organization
depended on widespread private ownership of automobiles. Los Angeles was a decentralized metropolis,
sprawling across the desert landscape over an area of 400 square miles. It was a city without a real center.
The downtown business district did not grow apace with the city as a whole, and the rapid transit system
designed to link the center with outlying areas withered away from disuse. Approximately 800,000 cars were
registered in Los Angeles County in 1930, one per 2.7 residents. Some visitors from the east coast were
dismayed at the endless urban sprawl and dismissed Los Angeles as a mere collection of suburbs in search of
a city. But the freedom and mobility of a city built on wheels attracted floods of migrants to the city.
1. According to the passage, the initial success of Hollywood's motion picture industry was due largely to
the……………
A. beauty of the countryside
B. region's reputation for luxurious lifestyles
C. region's climate and good weather
D. availability of many skilled workers
2. What is the passage mainly about?
A. The development of the Southern California oil fields
B. Factors contributing to the growth of Los Angeles
C. The growth of cities in the United States in the early 1900's
D. Industry and city planning in Los Angeles
3. The author characterizes the growth of new large cities in the United States after 1900 as resulting
primarily from.……………
A. new economic conditions B. images of cities shown in movies

77
C. a large migrant population D. new agricultural techniques
4. The word "it" in paragraph 2 refers to…………….
A. vision B. agricultural potential
C. aqueduct D. water
5. According to the passage, the Southern California oil fields were initially exploited due to……………..
A. an increase in the use of gasoline engines in North America
B. a desire to put unproductive desert land to good use
C. the fuel requirements of Los Angeles' rail system
D. innovative planning on the part of the city founders
6. The phrase "apace with" in paragraph 3 is closest in meaning to…………….
A. apart from B. a new with C. as fast as D. at the middle of
7. The word "meteoric" in paragraph 2 is closest in meaning to…………….
A. controversial B. famous C. methodical D. rapid
8. The visitors from the east coast mentioned in the passage thought that Los Angeles……………..
A. was not really a single city
B. lacked good suburban areas in which to live
C. had an excessively large population
D. was not accurately portrayed by Hollywood images
9. It can be inferred from the passage that the spatial organization of Los Angeles contributed to the relative
decline there of……………..
A. suburban neighborhoods B. oil fields
C. industrial areas D. public transportation
10. According to the passage, the most important factor in the development of agriculture around Los
Angeles was the……………..
A. development of new connections to the city's natural harbor
B. construction of an aqueduct
C. influx of new residents to agricultural areas near the city
D. expansion of transportation facilities
II. You are going to read an article in which four crime writers talk about other authors they like. For
questions 1-10, choose from the crime writers (A-D). The writers may be chosen more than once.
Which writer
1…………… enjoys the slow pace of the book?
2…………… feels overshadowed by this author?
3…………… appreciates the author’s precise style of writing?
4…………… mentions a detective with an unusual background?
5…………… believes a previous job influences the author’s work?
6…………… had not expected to enjoy the books?
7…………… likes the way the books reflect what’s going on in the world?
8…………… admires the way the author is not influenced by what is expected?
9…………… prefers books that show the feelings of the author?
10…………… is impressed by the consistent high standard of an author’s books?

A. John Harvey on Peter Temple


I started reading Temple’s books about 10 years ago, and at the time it was fairly straightforward crime
fiction set in Australia, a lot of it with a horse-racing background. The books were OK, but not exceptional.
Then suddenly with The Broken Shore, he just hit something quite different and moved up a level. He is
writing the kind of books I’ve spent 20 years trying to write, and writing them better than I do. He’s using
crime fiction not just to tell a story, but to say something about today’s society. He really gets the

78
connections between politics, I the police and the media right. I love the writing - it‘s really taut and a lesson
in how to say a lot without being too wordy.
B. Sara Paretsky on Liza Cody
There is a lot of good, stylish writing out there that puts me off because it’s unfelt. I’m looking for someone
who has a gift for storytelling, and who is willing to do the hard work of digging into the emotional aspect of
life. Liza Cody does that, and she takes a lot of risks. She doesn’t just do the easy thing to be recognized in
the marketplace; she goes where the story and emotion takes her. For instance, she had to self-publish her
most recent book, Ballad of A Dead Nobody, because no one in the industry was willing to. These days,
you’re told you have to create a brand or a series, with a recognizable link. She is willing to turn her back on
that and focus on what her inner voice is telling her.
C. Mark Billingham on Michael Connelly
The first Connelly book I read was The Poet, his first standalone novel and a big breakthrough for him. By
then, he had written four books in his Harry Bosch series, which I went back to read. He has maintained a
level of quality through a long-running series - an incredibly difficult thing to achieve. People talk about how
you create suspense, tricks such as cliffhangers and reveals, but actually the real secret is to create characters
the readers care about. Bosch, a detective in the Los Angeles Police Department, is a character who has
grown and changed, who you come to know and care about. All that aside, he is a fantastic storyteller,
probably the best in modem mystery fiction. I think his days as a journalist - he was a crime reporter on the
Los Angeles Times - stood him in good stead because he never forgets the story.
D. Ann Cleeves on Johan Theorin
I read Theorin’s first novel, Echoes from the Dead, after my Swedish editor had said: ‘There’s this Swedish
author writing four books set on an island in the Baltic - I wonder if he stole the idea from you?’ because I’ve
written novels set in the Shetlands. I was a bit annoyed - although I’m sure it was coincidental - so I wasn’t
prepared to like it. But I really loved it. He has this interesting detective who is elderly and lives in a
sheltered housing complex, so he can’t be out there doing car chases. One of the books is about this
crumbling house by the shore and you’re not sure if there’s a supernatural element. It has the most wonderful
climax. Theorin is prepared to take his time over the telling of the story, which is unusual these days. If you
look at the bestsellers, they usually move quite rapidly. There is nothing wrong with that, but Theorin writes
in a much deeper, literary way.
III. Read the article below about the illegal international trade of whale meat. Six sentences have been
removed from the article. Choose from the sentences A - F the one which fits each gap 1-5. There is one
extra sentence which you do not need to use.
EATING ENDANGERED SPECIES?
The International Whaling Commission was established in 1946 to manage dwindling stocks of whales.
Quotas were set to limit the number of whales that could be killed each year for commercial use, but these
were often ignored and whale numbers continued to decrease. In 1975, the Convention on International Trade
in Endangered Species of Wild Flora and Fauna (CITES) gave full protection to several species including the
blue, grey, humpback and right whales. International pressure on the IWC continued and in 1986 it finally
put a limit on commercial whaling.
(1)………….. This is achieved by issuing scientific research permits, as killing whales for research is not
forbidden. The stated aim of the Japanese research programme is to establish sustainable whaling in the
Antarctic Ocean. Both Japan and South Korea are also permitted to trade, within their own countries, whale
meat from animals killed as an incidental result of other fishing. (2)……………Also, in an attempt to control
hunting, individual whales are logged on a DNA register so that they can be identified.
The value of “lethal sampling”, that is, the practice of killing whales in order to study them, is highly
contentious issue. (3)…………... On the other hand, opponents say this information is not strictly necessary,
and moreover, there are better ways to get it. The selling of whale meat from the lethal sampling to fish
markets is purportedly to help fund the research. This claim, however, is disrupted by opponents as being a
cover for illegal whaling. Now there is evidence to support their views.

79
A team of scientists, led by Professor Scott Baker, have used DNA to analyse samples of sushi from
restaurants in Los Angeles and Seoul. The sushi was found to be made from the illegally-traded meat of
protected whale species. The results of the study were handed over to local and national authorities and have
since resulted in criminal proceedings against the Los Angeles restaurant.
The researcher used DNA sequencing to identify the species of whale and then used DNA profiling - the
same technique used to identify human individuals in criminal forensics - to identify the source of the meat.
(4)…………..In addition, some of the meat purchased in Seoul came from Antarctic minke whales, a species
which is not local to South Korea and must have therefore also been traded illegally.
Although Japanese authorities keep a DNA register of each whale destined to be sold commercially, this
information is not available for monitoring purposes, (5)…………... As the authors state, “The illegal trade
of products from protected species of whales, presumably taken under a national permit for scientific
research, is a timely reminder of the need for independent, transparent and robust monitoring of any future
whaling”.
A. The DNA results showed that the whale meat in the Los Angeles restaurant had almost certainly
originated in Japanese “scientific” whale hunts.
B. However, the export of any whale meat from these countries to the U.S.A, is strictly prohibited.
C. The researchers suggest that urgent action is needed in making this information available to scientists to
further monitoring and analysis of commercially available whale meat can take place.
D. Despite the 1986 ban, Japan has continued to hunt whales legally.
E. Some claim that it is required in order to learn about the eating habits and lifespan of whales.
F. Since the 1986 international moratorium, it has been assumed that there is no international trade in whale
products, but this does not seem to be the case.
PART C. WRITING
I. Complete the second sentence so that it has a similar meaning to the first sentence, using the word
given. Do not change the word given. You must use between two and five words, including the word given.
I. They offered her a job but she refused. (TURNED)
→ She……………………………………………………………………………………………of a job.
2. I find driving on the left in England very strange. (ACCUSTOMED)
→ I………………………………………………………………………driving on the left in England.
3. I expect you were very tired by the end of the week. (MUST)
→ By the end of the week……………………………………………………………………exhausted.
4. Being late is inexcusable. (EXCUSE)
→ There is………………………………………………………………………………… late.
5. Matthew decided not to do his homework and went to play football. (INSTEAD)
→ Matthew went to play football……………………………………………………………homework.
II. Essay writing
Every year an increasing number of students are doing their higher education abroad.
Do you think the benefits of this development outweigh the problems associated with it?
Write 250-300 words.
……………………………………………………………………………………………………………..
……………………………………………………………………………………………………………..
……………………………………………………………………………………………………………..
……………………………………………………………………………………………………………..

80
ĐỀ SỐ 18
(Đề thi vào lớp 10 THPT Chuyên Lê Hồng Phong - Nam Định)
Thời gian làm bài: 120 phút
PART A. GRAMMAR AND VOCABULARY (2.0 POINTS)
I. Circle the letter A, B, C or D to complete each of the following sentences. (l.0p)
1. What a lovely baby…………!
A. do they have B. have they C. they have D. they don’t have
2. Why don’t you buy oranges when they are much…………than apples?
A. less expensive B. not expensive C. as expensive D. lesser expensive
3. My mother…………for an hour and she hasn’t finished it yet.
A. has cooked B. has been cooking
C. had cooked D. had been cooking
4. …………I’d like to help you out, I’m afraid I just haven’t got any spare money at the moment.
A. Much as B. Try as C. However D. Despite
5. I bought some new shoes. They felt a bit strange…………because I wasn’t used to them.
A. first B. at first C. firstly D. first of all
6. There should be no discrimination on the…………of sex, race or religion.
A. grounds B. places C. areas D. fields
7. I can’t find my purse anywhere. I must…………it at the cinema.
A. leave B. have left
C. be leaving D. have been leaving
8. We don’t sell newspapers because there is no…………for them.
A. demand B. requirement C. claim D. request
9. A policeman suddenly appeared, so the thief quickly…………on a motorbike.
A. took in B. made off C. handed in D. took over
10. She insisted that the reporter…………her as his source of information.
A. didn’t mention B. doesn’t mention C. hadn’t mentioned D. not mention
II. Each of the following sentences has ONE mistake. Underline the mistake and write the correction in the provided
blank on the right. There is an example at the beginning (0). (0.5 p)
Sentences Corrections
0. People tend to make a lot of shopping at Christmas. do
1. Although a number of police officers was guarding the invaluable treasures in the
museum, the director worried that someone would try to steal them.
2. Laser treatment isn’t always effective and chemotherapy isn’t, neither.
3. All of mammals, dolphins are undoubtedly among the friendliest to humans.
4. Dams are used to control flooding, provide water for irrigation, and generating
electricity for the surrounding areas.
5. I made every effort to contact with John two weeks ago but so far I haven’t received
his reply.

III. Give the correct form of the words in capital letters. Write your answers in the blanks. (0.5p)
1. She didn’t want to give him the ………………of seeing her cry. (SATISFY)
2. The church was damaged in the war but it has now been………………(BUILD)
3. We can’t tell which team will win. The result is so ………………(PREDICT)
4. The young man was………………for possession of drugs for five years. (PRISON)
5. Should developing countries be given more………………by the West? (ASSIST)

81
PART B. READING (3.0 POINTS)
I. Read the text and circle A, B, C or D that best fits each space to complete the text. (1.0 p)
HISTORY OF WRITING
The development of writing (1)…………….a huge difference to the world and we might see it as the
beginning of the media. Pieces of pottery with marks on that are probably numbers have been (2) …………in
China that date from around 4000 BC. Hieroglyphics and other forms of “picture writing” developed in the
(3) …………….around Mesopatamia (modern-day Iraq), where the (4) …………….Sumerian civilization
was based, from around 3300 BC onwards. However, the first (5) …………….alphabet was used by the
Phoenicians around 1050 BC. Their alphabet had 22 letters and it is (6) …………….that it lasted for 1000
years. The first two signs were called “aleph”, which in Greek became “alpha” and “beta”, which gave us the
(7) ……………word “alphabet”.
The modern European alphabet is based on the Greek and (8) …………….to other European
countries under the Romans. A number of changes took (9) …………….as time passed. The Romans added
the letter G, and the letters J and V were (10) …………….to people in Shakespeare’s time.
1. A. made B. took C. did D. had
2. A. invented B. displayed C. appeared D. discovered
3. A. earth B. length C. area D. distance
4. A. ancient B. old-fashioned C. antique D. dated
5. A. precise B. true C. exact D. accurate
6. A. observed B. measured C. estimated D. counted
7. A. modern B. trendy C. new D. fashionable
8. A. was B.spread C. appeared D. occurred
9. A. control B. part C. place D. account
10. A. hidden B. unpopular C. infamous D. unknown
II. Read the text and choose the correct heading from the list i-ix for each paragraph (1-5) below. Write your answers
in the boxes provided. There is an example at the beginning (0) (1.0p)
i The plans for the future of films vi Too true to life?
ii The history of cinema vii Uncertainty about financial advantages
iii Integrating other events into cinema viii Indecision about a name
iv Current problems with distribution ix The contrasts of cinema today
v An unexpected advantage

The end of silver screen?


Cinema technology has remained much the same for a century, so when will it go digital? Kevin Hilton
views the projections.
Paragraph 0. Cinema is full of contradictions. It is high-tech and old-fashioned at the same time. Today’s
films are full of digital sound and computer-generated special effects. Yet they are still stored on celluloid
film, the basis of which is more than 100 years old. They are also displayed with projectors and screens that
seem to belong to our great grandparents’ generation.
Paragraph 1. Now that we are in the second century of cinema, there are moves to bring the medium right up
to date. This will involve revolutionizing not just how films are made but also how they are distributed and
presented. The aim is not only to produce and prepare films digitally, but to be able to send them to movie
theatres by digital, electronic means. High-resolution digital projectors would then show the film. Supporters
say this will make considerable savings at all stages of this chain, particularly for distribution.
Paragraph 2. With such a major technological revolution on the horizon, it seems strange that the industry is
still not sure what to call itself. This may appear a minor point, but the choices, ‘digital’ cinema and
‘electronic’ cinema (e-cinema), suggest different approaches to, and aspects of, the business. Digital cinema
refers to the physical capture of images; e-cinema covers the whole chain, from production through post-
production (editing, addition of special effects and construction of soundtrack) to distribution and projection.

82
Paragraph 3. And what about the effects of the new medium? The main selling point of digital cinema is the
high resolution and sharpness of the final image. But those who support the old-fashioned approach to film
point to the celluloid medium’s quality of warmth. A recurring criticism of video is that it may be too good:
uncomfortably real, rather like looking through an open window. In 1989, the director of the first full-length
American digital high-definition movie admitted that the picture had a ‘stark, strange reality to it’.
Paragraph 4. Even the money-saving aspect of e-cinema is doubted. One expert says that existing cinemas
will have to show the new material and not all of them will readily or rapidly furnish themselves with the
right equipment. ‘E-cinema is seen as a way of saving money, because print costs a lot,’ he says. ‘But for that
to: work, cinemas have to be showing the films because cinemas are the engine that drives the film industry.’
Paragraph 5. This view has promoted some pro-digital entrepreneurs to take a slightly different approach. HD
Thames is looking at reinventing the existing cinema market, moving towards e-theatre, which would use digital video
and projection to present plays, musicals and some sporting events to the public. This is not that different from the
large-screen TV system that was set up in New York in 1930, and John Logie Baird’s experiments with TV in the late
1920s and early 30s.
Paragraph 0 1 2 3 4 5
Heading ix
III. Read the text below and fill in each numbered blank with ONE suitable word. (1.0 p)
Our classes take place for three hours every morning from Monday to
Friday. The (1)………….class size is twelve and the average is ten. We use
modern methods of teaching and learning, and the school has (2) ………….
language laboratory, a video camera and recorders. You will only be successful in improving your English,
however, if you work hard and practise (3) ………….English as much as you can. You will take a short
(4)……………in English as soon as you arrive. In this way, we can put you in a(5)…………. at the most
suitable level.
There are two classes at the Elementary level: one is for complete (6)……………and the other is for students
who know only a little English. In both classes you will practise simple conversations. In the class at the
Intermediate level, you will have a lot of practice in communication in real-life (7) ………….because we
help you to use the English you have previously (8) ………….in your own country. You will also have the
chance to improve your knowledge of English grammar and to build (9) ………….your vocabulary.
The emphasis is on oral communication practice in a wide (10) ………….of situations at the
advanced knowledge. You will learn how to use language correctly and appropriately when you talk to native
speakers. In addition, you will develop such study skills as reading efficiently, writing articles and reports,
and note-taking from books and lectures.
PART C. WRITING (3.0 POINTS)
I. Finish the second sentence so that it has a similar meaning to the first one, using the clues at the
beginning of each sentence. (0.5p)
1. She felt sorry she hadn’t gone to his birthday party.
- She regretted …………………………………………………………………………………………….
2. You missed the class yesterday so you can’t do this exercise now.
- Had …………………………………………………………………………………………………….
3. She was so physically attractive that every boy ran after her.
- Such……………………………………………………………………………………………………...
4. I am sure the man took the money on purpose.
- The man can’t……………………………………………………………………………………………
5. The number of people out of work has fallen dramatically this year.
- There has…………………………………………………………………………………………………
II. Complete the second sentence so that it has a similar meaning to the first sentence, using the word
given in brackets. Do not change the word given (1.0p)
1. I don’t intend to take a part-time job this summer. (INTENTION)

83
- I ………………………………………………………………………………………………………….
2. What a pity I told John about my plan. (ONLY)
- If.…………………………………………………………………………………………………………
3. There is no way that young man can pass the coming driving test. (BOUND)
- That young man………………………………………………………………………………………….
4. He was so scared that he couldn’t tell me what he really thought. (COURAGE)
- He………………………………………………………………………………………………………..
5. Do you think that people’s personalities are strongly affected by climate? I (INFLUENCE)
- Do you think that climate………………………………………………………………………………..
6. It was the heavy rain yesterday that made me ill. (FOR)
-If.………………………………………………………………………………………………………….
7. He did not get married until he was 40 years old. (UNTIL)
-It…………………………………………………………………………………………………………..
8. The events of that day were described in detail by my daughter. (DETAILED)
- My daughter……………………………………………………………………………………………...
9. The phone rang again as soon as I put it down. (HAD)
- Hardly…………………………………………………………………………………………………….
10. My grandfather completely forgot that he had phoned me. (RECOLLECTION)
- My grandfather didn’t have………………………………………………………………………………
III. Write a paragraph. (1.5p)
It is obvious that there are more and more people going on holiday all over the country. What, in your
opinion, are the reasons for this? Write a paragraph of about 150 words to support your answer, using
examples and relevant evidence.

ĐỀ SỐ 19
(Đề thi vào lớp 10 THPT Chuyên An Giang)
Thời gian làm bài: 120 phút
SECTION A. LISTENING - TRACK 5 (2 points)
Part 1. Listen and fill in the missing information. (1 point)
MIDBURY DRAMA CLUB
Background
- club started in 1957
- prize recently won by youth section
- usually performs (1)………………plays
Meetings
- next auditions will be on Tuesday, (2)………………
- help is needed with publicity and lights
- rehearsals take place in the (3)………………hall
- nearest car park for rehearsals in Ashburton Road opposite the sports centre
Costs
- annual membership fee is £ 180
- extra payment for (4)………………
Contact
- secretary’s name is Sarah Sawdicott
- secretary’s phone number is (5)………………
Your answers:
1……………… 4………………
2……………… 5………………

84
3………………
Part 2. You will hear a news reporter called Angela Bond, talking on the radio about her job. Listen
and choose the best answer. (1 point)
6. Where has Angela just returned from?
A. Britain B. the USA C. Asia
7. One thing Angela dislikes about her job is that she………………
A. loves being in dangerous situations
B. never knows where she'll go next
C. enjoys watching important events happen
8. What did Angela bring home from Hong Kong?
A. pictures B. carpets C. furniture
9. Where did Angela meet her boyfriend?
A. at her sister's house B. at university C. in Thailand
10. What does Angela do to relax?
A. She cooks a meal. B. She goes sailing. C. She goes shopping.
Your answers:
6 7 8 9 10

SECTION B. LANGUAGE USE (2 points)


Part 1. Choose the word or phrase which best complete each sentence. (1 point)
11. My brother got an excellent grade in his examination ............ the fact that he had not worked particularly
hard.
A. on account of B. because of C. in spite of D. although
12. Jack’s shoes are very dirty. They need ..............
A. cleaning B. cleaned C. to clean D. to be cleaning
13. ............ tasteful furniture you have bought!
A. What a B. What C. So D. How
14. We will not pass the exam .................... working harder.
A. unless B. without C. if D. although
15. It was raining very....................., so I had to wear my raincoat.
A. wet B. bad C. hard D. heavy
16. It was said the man ......................35 the following week.
A. was B. is C. will be D. would be
17. Remember to turn off the......................after using to save water.
A. lights B. televisions C. faucets D. gas stoves
18. We had better get......................to check the wiring before we start decorating.
A. a plumper B. an electrician C. an architect D. a consumer
19. If you want to save money, you should......................the amount of water your family use.
A. reduce B. increase C. repair D. adapt
20. If you are not satisfied......................your essay, I suggest you rewrite it.
A. to B. in C. with D. at
Your answers:
11 12 13 14 15 16 17 18 19 20

Part 2. Choose the option A, B, C or D to indicate the word(s) SIMILAR in meaning to the underlined
word(s) in each of the following sentences. (0.5 point)
21. She didn’t stop working when she got married.

85
A. give up B. go on C. get into. D. set in .
22. Who will take care of your children when you go away?
A. look for B. take after C. put off D. look after
23. Tet is a festival which occurs in late January or early February.
A. begins B. causes C. happens D. carries
24. Maths and English are compulsory subjects in this course.
A. effective B. required C. difficult D. separated
25. Ultimately, you will have to make the decision yourself.
A. Luckily B. Basically C. Finally D. Generally
Your answers:
21 22 23 24 25

Part 3. Give the correct form o f the words in capital letters. (0.5 point)
26. She didn’t want to give him the ............................. of seeing her cry (SATISFY)
27. The church was damaged in the war, but it has now been ................... (BUILD)
28. We can’t tell which team will win. The result is so .......................... (PREDICT)
29. He is an ............................. storyteller. Everyone likes listening to his stories. (AMUSE)
30.............................., I lost my keys on the way home. (FORTUNE)
Your answers:
26............................. 29.............................
27............................. 30.............................
28.............................

SECTION C. READING (3 points)


Part 1. Fill in each numbered blank with ONE suitable word. (1 point)
Before the start of their season, most sports teams, regardless of whether they are professional,
college, or high school teams, hold a training camp. The training camp can last anywhere from two months
(31) ..................... three months. Training camps serve several purposes. Obviously, they are intended to (32)
..................... the players and improve their skills, but they also help coaches make (33) ..................... about
the team. As they watch their players in the training camps, coaches can see what players they will keep
(34) ............................the team and who will be dropped. They can also see what game strategies will work
(35) ..................... for their team. Training camp generally starts with about two weeks of conditioning. The
intention of this phase of training camp is (36) .....................to get the players into good physical
shape. It consists mostly of exercise, and players may (37) ..................... even play their actual sport. There
are two types of conditioning: endurance conditioning and strength training. Endurance conditioning
basically involves a lot of running and exercises to make (38) .....................heart stronger. Strength training
involves exercise to strengthen specific muscles. Most athletes agree that this is the most physically
(39) .....................part of training camp, and this is (40) .....................the majority of training camp injuries
occurs.
Your answers:
31............................. 36.............................
32............................. 37.............................
33............................. 38.............................
34............................. 39.............................
35............................. 40.............................
Part 2 Read the article on “Spyware ” and write questions A - E in the correct place 41-45. (1 point)

86
A.What is Spyware?
B. How do I get rid of it?
C. What can it do?
D. How does it get on to my computer?
E. Is Spyware a common problem?

Nobody likes a computer virus, but at least you can get rid of it. And there is a lot of anti-virus
software these days. It can find a virus, fix it and you can forget about it. Unfortunately the same isn’t true
for spyware.
41. Spyware is software that hides somewhere on your computer. It collects information about what you do
on the Internet and passes this information to companies without your permission. If you shop on the Internet
and use your credit card, you should know that some spyware can record this information!
42. Your computer can catch spyware in lots of ways. If you open the wrong mail, or even visit the wrong
website, spyware can download itself onto your computer. And spy ware often comes with free software.

43. Most spyware just collects information about your surfing habits for advertising reasons. But some
spyware can be more powerful and will often make your computer slower. Adware is a type of spyware
which is advertising software, h makes ‘pop-up’ advertisements appear while you are connected to the
Internet. Not dangerous ... but very annoying.
44. If this is the first time you’ve heard of spyware, you’ve probably got some on your PC. Surveys hav
found that 90% of computers have several pieces of spyware on them. And spyware will stay on your
computer for a long time, quietly collecting information and sending it back to its authors.
45. There is some anti-spyware software that will remove most of your problems. However, some anti-
spyware is spyware itself, so be careful! For a complete list of software, go to spywareyvarrior.com.
Your answers:
41 42 43 44 45

Part 3. Read the passage carefully and choose the correct answer. (1 point)
Design is the act of making something better. Everything, no matter how ordinary, has been designed.
That some objects give us no special pleasure or are not fashionable does not alter the fact that somebody
decided what they would look like, what they would do and how they would be used.
Every time you buy a new kettle or toaster, the quality of the design is influential, encouraging you to
choose one kettle or toaster over the others. Good design works well. Excellent design works well and gives
pleasure. Look at it the other way round. Some objects look very good but do not work well. Take the Alessi
kettle, with its curved handle and two-tone whistle. It looks very exciting but the handle can get too hot to
touch. Compare this with the familiar Russell Hobbs automatic electric kettle. It has been in production since
the late 1950s, works perfectly and looks good.
Poor designs are easy to find. If you cannot see what is at the back of the kitchen cupboard without
getting down on your hands and knees, that is bad design. If you catch your sleeve on a door handle, that is
bad design. If you cannot understand how to use the controls on your cooker without searching for the
instruction book - and if, when you find the book, you still cannot work the timing switch, that is
unpardonably bad design.
The question is: how, when these kinds of faults are so obvious, have some designs ever reached our
homes? The answer is that in most cases, bad designs emerge because not enough energy and time is given to
thinking through all the different questions that should be asked about the product. Kitchen cupboard makers
will say that they are making cupboards as economically as possible. This kind of “cheapness” is one of the

87
main reasons for the absence of good design in our homes. To make a cupboard where the shelves swing out
to display the contents when the door is opened is more expensive.
46. What does the passage say that good designers think about?
A. How things will be used. B. What people are used to.
C. What is fashionable. D. What will influence people.
47. Things which are excellently designed……………
A. work perfectly. B. last a long time.
C. always get chosen by shoppers. D. both work well and good look.
48. What was wrong with Alessi kettle?
A. It was too round. B. It was unreliable.
C. The design was impractical. D. The design was old-fashioned.
49. In what ways are some cookers badly designed?
A. The handles stick out too far.
B. It is difficult to find the controls.
C. Using the timing switch is confusing process.
D. The instruction books have no diagrams.
50. Why do badly designed things get made and sold?
A. They are quicker and cost less to make.
B. Manufacturers pay low wages to designers.
C. Designers do not know enough about manufacturing process.
D. There are too few food designers.
Your answers:
46 47 48 49 50

SECTION D. WRITING (3 points)


Part 1. Complete the second sentence, using the word given in bold. Do not change the word given. You
must use BETWEEN THREE AND SIX WORDS, including the word given. (1 point)
51. We are proud of our work. (TAKE)
→We………………………………………………………………………………..our work.
52. There were five participants in the discussion. (TOOK)
→Five people………………………………………………………………………the discussion.
53. Everyone was surprised to see John leave the meeting early. (SURPRISE)
→To……………………………………………………………………………..the meeting early.
54. Peter was ready to leave the office when his boss asked him to type up a report. (POINT)
→Peter………………………………………the office when his boss asked him to type up a report.
55. The task is too complex for students in the class to understand. (ENOUGH)
→The task is………………………………………………for students in the class to understand.
Part 2. Finish the second sentence in each pair in such a way that it means the same as the sentence
before it. (0.5 point)
56. They have recycled waste paper into newsprint.
→Waste paper………………………………………………………………………………
57. It’s a two-hour flight from Can Tho to Hanoi.
→It takes……………………………………………………………………………………
58. “You’d better do more morning exercise, Minh,” I said.
→I advised………………………………………………………………………………….
59. The boys prefer playing soccer to staying indoors.
→The boys would rather……………………………………………………………………
60. Their dog was so fierce that nobody would visit them.

88
→They had…………………………………………………………………………………..
Part 3. Write a paragraph of about 120 words about the benefits of studying at a high school for the
gift (1.5 points)

ĐỀ SỐ 20
(Đề thi vào lớp 10 THPT Chuyên Bình Dương)
Thời gian làm bài: 120 phút
I. Communication. (1 point)
Choose the statements (A-H) to complete the conversation between Lily and the receptionist .
A. as soon as possible Lily: Hi, I’d like to see the doctor.
B. Do you have an appointment? Receptionist: (1)……………
C. Here you are. Lily: No.
D. I’ll take care of that in a moment. Receptionist: When would you like an appointment?
E. It’s too cold outside today. Lily: Is today possible?
F. Of course not. Receptionist: Yes. Today is fine. What time would you like?
G. Tomorrow at 8 o’clock. Lily: (2) ……………My stomach really hurts.
H. Would you mind waiting little longer? Receptionist: Please wait a moment. I’ll see if the doctor is
available.
Lily: OK.
Receptionist: Sorry, he’s with a patient right now. It’s probably
going to be about 30 minutes.
(3) ……………
Lily: No problem.
Receptionist: May I see your insurance card, please?
Lily : (4) ……………
Receptionist: Thank you.
Lily: Would you please turn on the heat? It’s really cold in here.
Receptionist: (5) ……………Please have a seat and fill out
these forms.

II. Phonetics. (0.4 points.)


Choose the word whose underlined part differs from the other three in pronunciation in each of the
following questions.
6. A. missed B.canned C. cracked D. dripped
7. A. treasure B. weather C. earthquake D. healthy
Choose the word which differs from the other three in the position of primary stress in each of the
following questions.
8. A. invent B. reduce C. pollute D. surface
9. A. prohibit B. compliment C. minimize D. innovate
III. Lexico-Grammar. (1.6 pts) Choose the best option A, B, C, or D.
10. Donald gets up at six every morning. Then he…………to his office at seven-thirty.
A. goes B. went C. is going D. has gone
11. At this time last year. Mr. and Mrs. Smith’s son…………for WHO.
A. have worked B. has worked C. was working D. were working
12. I think you should buy a Moto 360 smartwatch,…………is the prettiest watch at present.
A. who B. that C. which D. whose
13. You have free time,…………?
A. have you B. haven’t you C. do you D. don’t you

89
14. Should you be late, the train…………
A. won’t wait B. don’t wait C. didn’t wait D. not wait
15. Taylor Swift was born…………December 13, 1989.
A. on B. in C. at D. since
16. They all wish the Second World War…………
A. didn’t happen B. hadn’t happened
C. wouldn’t happen D. shouldn’t happen
17. She was walking home when the wind…………her hat off.
A. blows B. blew C. was blown D. has blown
18. Of the two girls, Helen and Alice, Helen is the…………
A. funny B. funnier C. most funny D. funniest
19. He has…………given up playing on-line games.
A. success B. succeeded C. successful D. successfully
20. My sister can play the…………very well.
A. yoga B. tennis C. judo D. violin
21. Robots are used…………dangerous jobs in places where people cannot safely work.
A. do B. to do C. doing D. to doing
22. She was wearing…………dress.
A. a unique white wedding striped silk B. a unique white silk striped wedding
C. a unique white striped silk wedding D. a unique white wedding silk striped
23. Some scientists have predicted that healthy adults and children may one day take drugs to improve their
intelligence. The synonym of ‘predicted’ is…………
A. believed B. thought C. forecasted D. imagined
24. Please decrease the volume. I’m doing my homework. The synonym of ‘decrease’ is…………
A. turn up B. turn down C. turn on D. turn off
25. Find the mistake in one of the four underlined parts of this sentence.
The narrow road to our house needs to be widening soon.
A. The B. to C. needs D. be widening
IV. Reading. (2.5 points)
Read the passage below carefully and fill in each blank with the best option, A, B, C, or D.
Earth Day is the largest, most widely celebrated international environmental event. Celebrating Earth Day
can inspire awareness of and appreciation for the earth’s environment. Earth Day was first celebrated in
April, 1970 in (26) ……………when 20 million people and thousands of local schools and communities
participated. Earth Day’s success helped influence the government to create stronger laws to(27)…………
the environment. Taking care of the earth may sound (28) ……………, but there are many simple steps that
each of us can take to save energy and reduce our impact on the planet. The earths (29) ……………us with
all good gifts: food, water, minerals, fresh air, fire, beauty. When we remember this, we are more careful
about our attitudes and actions. We choose food that is healthful for us and environmentally friendly for the
earth. We pick up (30) ……………to show our respect for our surroundings.
26. A. United States B. a United States
C. an United States D. the United States
27. A. impress B. protect C. worship D. distinguish
28. A. complicated B. considerate C. compulsory D. environmental
29. A. provide B. help C. gives D. supplies
30. A. evidence B. refreshment C. litter D. deforestation
Read the passage below carefully and fill in each blank with the best option, A, B, C, or D.
The organisation City Of Trees have shown the public a plan to plant three million trees in Greater
Manchester over the next quarter of a century, a plant that aims to use the power of trees to refresh and
redevelop the area. Of course planting more trees in built up urban areas has not only beauty effects but also

90
other good consequences. As well as reducing stress, encouraging people to spend more time in shopping
areas and improving air quality, a lot of trees can actually reduce flooding in urban areas.
Tony Hothersall, the director of City Of Trees , explained that the scheme had three main aims. He
said: "One is to plant three million trees which means a tree for every man, woman and child in the next
quarter of a century. Next, we focus on managing existing woodlands because there is no point in planting
new woodlands if you can't manage what you've got already. Finally, we want to interest people a lot more in
their natural environment - in planting trees, in managing areas, in understanding more about the benefits that
trees and woodlands bring to our society. They can do great things in terms of air pollution and can help to
protect us from noise pollution. What is really important is that it is about the right tree in the right place."
And the plan is off to a storming start. Since its launch in 2015, about one hundred thousand trees have been
planted around Greta Manchester.
(Adapted from http://wvvvv.bbc.com)
Questions 31 to 34. Match the following synonyms from the article.
Example 0. G.
0. areas 31. effects 32. aims 33. woodlands 34. benefits

A. advantages B. signals C. disaters D. forests E. grasslands


F. purposes G. regions H. results I. trees J. wetlands
35. According to the passage, what is NOT the benefit of trees?
A. Lowering stress levels B. Bettering air quality
C. Heating shopping areas D. Refreshing the area
36. What do trees help to decrease?
A. Floods B. Storms C. Beauty D. Woodlands
37. According to Tony Hothersall, what will the project attract people to?
A. City of Trees B. Their society
C. Urban areas D. Natural environment
38 What does the word “they” in the last paragraph refer to?
A. trees B. benefits C. areas D. people
39 When will they complete planting 3,000,000 trees?
A. In 2015 B. 2030 C. 2040 D. In 2115
40. Which of the following can be the best title for the passage?
A. Benefits of trees and woodlands B. Scheme to grow a city of trees
C. The biggest area launch in 2015 D. Billions of trees in Manchester
Read the passage below carefully and fill in each blank with the best option, A, B, C, D, E, F, G, H, I,
or J.
German sportwear bans Adidas is helping to clean up the earth’s oceans by using the waste floating
around the world to make shoes. The sportwear company has teamed up with Parley for the Oceans, an
environmental group that raises awareness of pollution in the ocean, to produce pairs of trainers made from
recycled ocean waste. Only 7,000 pairs of the UltraBOOST Uncaged Parley were made in 2016, but the
company says it plans to make one million pairs of shoes using Parley Ocean Plastic in 2017. “Nobody can
save the oceans alone. Each of us can play a role in the solution. We are extremely proud that Adidas is
joining us in this mission to show that it is possible to turn ocean plastic into something cool,’ said Cyrill
Gutsch, the founder of Parley for the Oceans.
Priced at €2,000, the shoes contain 11 plastic bottles. The upper part of the shoe is made from
recycled polyester and waste plastic taken from the waters around the Mandives, and most of the rest of the
trainer was made from recycled material.
Plastic in the world’s odeans has become am increasing problem in recent years. A report in January
said oceans would contain more plastic than fish by 2050 unless the world stopped rubbish leaking into the

91
seas. According to UNESCO, in 2006, there were 30,000 pieces of floating plastic for every squre kilometre
of oceans. Plastic kills over a million seabirds and 100,000 marine animals evey year.
Questions 41-44. Match the following synonyms from the article.
41. awareness 42. trainer 43. role 44. rubbish

A. atmosphere B. garbage C. instructor D. kindness E. part


F. reduction G. shoe H. understanding I. variety J. wastebasket
45. How many pairs of trainers that use Parley Ocean Plastic will be made this year?
A. 7, 000 B. 30,000 C. 100,000 D. 1,000,000
46. What is the UltraBOOST Uncaged Parley made from?
A. Adidas and Parley for the Oceans B. Recycled material and plastic
C. The waters around the Mandives D. Floating seabirds and marine animals
47. According to Cyrill Gutsch, it is possible to turn ocean plastic into…………….
A. Something wet B. Something cold C. Something calm D. Something great
48. Where is floating plastic found?
A. On the surface of waters B. In UltraBOOST Uncaged
C. At the bottom of the ocean D. Only in Mandives’s waters
49. According to the passage, how many seabirds are killed every year?
A. About 7, 000 B. About 30,000 C. About 100,000 D.About 1,000,000
50. Why are the trainers made from floating ocean rubbish?
A. To raise pollution in the waters B. To decrease plastic in the ocean
C. To recycle rubbish in the Mandives D. To make something cool to sell
V. Writing.
51. Health is more important than money.
→ Money………………………………………………………………………………….
52. They said: “Why don’t we build a library in our village?”
→ They suggested building……………………………………………………………….
53. The last time she rode a bike was three years ago.
→ She hasn’t………………………………………………………………………………
54. It took Michael thirty minutes to take his dog for a walk.
→ Michael spent half.……………………………………………………………………..
55. Lindsey regretted telling them her secret.
→ Lindsey wishes she……………………………………………………………………..
Paragraph writing. (2pts)
Would you like to live in the big city or in the countryside? Write a paragraph (from 80-120 words) about the
place you would like to live in and some benefits of living in that place.

ĐỀ SỐ 21
(Đề thi vào lớp 10 THPT Chuyên Đại Học Sư Phạm Hà Nội)
Thời gian làm bài: 120 phút
Choose the word that is CLOSEST im meaning to the underlined word or phrase in each sentence.
Write your answer (A, B, C, or D) in the corresponding numbered boxes on the answer sheet.
1. Gerry didn't go on the expedition - he made up that part of the story.
A. narrated B. unfolded C. recounted D. invented
2. What seems certain for any language is that new words will form, meanings will migrate, and obsolete
words will die out.

92
A. outdated B. borrowed C. printed D. loaded
3. Flats which are both comfortable and reasonably priced are few and far between in the current context of
economics crisis.
A. non-standard B. non-existent C. unusual D. uncommon
Choose the word whose primary stress is placed differentlt from that of the others. Write A, B, C, or D
in the corresponding numbered boxes on the answer sheet.
4. A. preservative B. political C. artificial D. compulsoiy
5. A. eplore B. swallow C. forbid D. exploit
Choose the word whose underlined part is pronounced differently from that of the others. Write A, B,
C, or D in the corresponding numbered boxes on the answer sheet.
6. A. develop B. respond C. devastate D. preserve
7. A. churches B. clothes C. colleges D. exercises
8. A. essence B. passage C. cassette D. mission
Read the passage and decide which answer best fits each gap. Write A, B, C, or D in the corresponding
numbered boxes on the answer sheet. The first one has been done as an example.
WHAT’S IN IT FOR ME?
Students and jobseekers keen to get on to the course or into the place of their (0)……………..hope that
voluntary work will help them (9) ……………..from the crowd. This chance to (10) ……………..experience
- personally and professionally - is (11) ……………..on the wish-list of young people.
A survey carried out last year revealed that young and old (12) ……………..said volunteering had
improved their lives, particularly those (13) ……………..in conservation and heritage work.
Businesses recognize its importance and get to (14) ……………..their profile in the community,
while staff get a break from their daily routine to develop "soft skills", such as initiative and decision-
making. One volunteering organization is (15) ……………..another survey to find out if volunteering does
make a difference in the workplace, or if it is something businesses do simply to improve their (16) ………
Not (17) ……………..are business-sponsored placements becoming more common, the government
is also investing money and aiming to (18) ……………..volunteers. The push is clearly on to make
volunteering as attractive as possible to everyone.
And the more people who participate, the more the act fulfils its aim of making the world a better
place.
Example: 0. B
0. A. alternative B. choice C. option D. selection
9. A. point out B. stand out C. pick out D. lift out
10. A. win B. coolect C. achieve D. gain
11. A. high B. strong C. sharp D. extreme
12. A. alike B. also C. similar D. same
13. A. associated B. committed C. connected D. involved
14. A. raise B. motivate C. increase D. arouse
15. A. conducting B. directing C. guiding D. governing
16. A. figure B. image C. look D. representation
17. A. merely B. only C. simply D. just
18. A. bring B. recruit C. claim D. enter
Choose the best answer to complete each of the following sentences. Write A, B, C, or D in the
corresponding numbered boxes on the answer sheet.
19. The office was closed for a week for refurbishment and now the staff have to deal with the………..that
built up during their absence.
A. backlash B. backdrop C. backdog D. backstage
20. The board proposes that the majority of this year's profits………..in new product development.
A. is invested B. to be invested C. be invested D. will be invested

93
21. Julie was so engrossed………..the book that she didn't hear sister come in.
A. to B. in C. on D. with
22.. As they travelled across the………..landscape, each of them wondered how it was possible to grow
anything there.
A. bustling B. grimy C. barren D. mundane
23……….. modelling is actually hard work.
A. However glamourous it may seem
B. Even if it may seem glamourous
C. Glamourous although it may seem
D. So glamourous may it seem
24. It's sensible to take our travel insurance.............our luggage is lost or damaged.
A. otherwisw B. as long as C. unless D. in case
25. Now I associate public transport with one of the worst experiences of my life and the.............of it is that I
will never catch a bus again.
A. thick and thin B. ups and downs
C. hard and fast D. long and short
26. Jane was a…………...woman.
A. young beautiful thin tall B. young thin tall beautiful
C. beautiful young tall thin D. beautiful tall thin young
27. Mary: Did you enjoy the movie? Tom: No. I would rather………….home.
A. to stay B. stayed C. to have stayed D. have stayed
28. The scientists are on the………….of making an incredible discovery abut our past.
A. edge B. limit C. verge D. extent
29. Mary: Did the teacher mention anything about a test in last week's class?
Tom: I don't know. I had to leave early because I…………..to the dentist.
A. was going B. would go C. went D. had gone
30. In no way…………..that people will be prevented from organizing peaceful protests.
A. this law means B. means this law
C. does this law mean D. this law does mean
Read the following article about how to be environmentally friendly and decide in which paragraph
(A-E) the following are mentioned. Write your answer (A, B, C, or D) in the corresponding numbered
boxes on the answer sheet. Write one letter for each answer. The paragraph may be chosen more than
once.
A. FAIR TRADE
Farmers in developing countries are some of the most vunerable people on earth, prey to world
commodity markets, middle men and the weather. So-called “fair trade" arrangements guarantee co-operative
groups a price above the world market ans a bonus on top. The growing fair-trade market has distributed
hundreds of millions of pounds to more than 50 million people worldwide. But critics say that fair trade will
never lift a country out of poverty; indeed, it may keep it there, because the money generated from sales goes
almost in its entirely rich countries which promote the products. As a simple guide, only a bout 5% of the
sale price of a fair-trade chocolate bar may actually go to the poor country.
B. ORGANIC FOOD
For food to be organic it must be free of added chemicals, both in the growing of the food and in the
killing of the pests that might damage the crop. In a world where many manufactures chemicals have never
been properly tested for safety, this is a very big selling point. Parents are thus prepared to pay a premium for
organic food, especially when chemicals suspected of causing a variety of problems have been found, albeit
in tiny quantity, in most children's blood. The problem is that many farmers have not switched to organic in
sufficient numbers to satisfy this growing market. As a result, supermarkets are often forced to fly vegetables
as they can label "organic" halfway round the wolrd, at a great cost to the planet in extra greenhouse gases.

94
Environmentalists are now urging shoppers to buy locally produced vegetables, even if they are not organic
and have been sprayed with pesticides.
C. RECYCLING
A great shift has taken place in the way we think about rubbish. Where once we were happy to put it
in landfills or dump it at sea, we are now urged by national and local governments to recycle it and think of
waste as a resource. The wheeliebin culture is being replaced by a series of kerbside collections for paper,
metal, plastic, bottles, clothes and compost. The idea is to cut landfill as well as saving the planet. It is,
however, having some unexpected consequences. Most of Britain's plastic and paper is now being sent for
recycling in China or in India, which creates more greenhouse gases just to get it there, plus workers then
have to separate it. Meanwhile, some paper and bottles carefully sorted out by householders end up being
dumped in landfills after all, because the demand for recycled materials constantly fluctuates.
D. BEING CARBON NEUTRAL
If you want to make yourself feel better about the planet, there are lots of ways to ease your
conscience by becoming "carbon neutral". One of the most appealing methods is to pay for someone to plant
frees, preferably creating or regeneratingnew forests. The theory is that trees grow by absorbing carbon
dioxide and giving out oxygen, storing the carbon in their trunks. But woods and forests create their own
mini-climate, which collects and stores water and creates rainclouds. Added to this, there is the potentila
problem that planting frees often releases carbon stored in the soil - and what happens if the forests catch fire,
or are chopped down and harvested for timber? Another and perhaps better solution might be to invest in
small-scale hydro-electric schemes, so that people who live in the Himalayas, for example, and currently do
not have electricity, can develop a 21st-century lifestyle without polluting the planet.
E. ECO-TOURISM
The idea of "green" tourism is to persuade local people not to chop down forests, shoot elephants or wipe
out tigers, but to prepare them so rich tourists visit and peer at the wildlife through binoculars. Unfortunately, the
best money is made from reintroducing animal for trophy hunting by the very rich - an idea which does not always
meet with approval and has caused much debate. While tourists may help sustain some national parks, they often
create as many problems as they solve. One is that they tend to demand all mod cons in their hotels, such as a great
deal of water for showers: a luxury sometimes not available for locals. Eco- tourism, when properly managed, can
offer the locals and the animals a brighter future. Sometimes, though, the only winners are a few business people
who own hotels.
In which section is the following mentioned?
a controversial pastime that raises considerable money 31………….
an action which creates a different weather pattern 32………….
an undesirable result of necessary global transportation 33…………. 34…………….

(NB. You must provide two different options)


inadequate research into harmful substances 35………….
a continual change in what is required or needed 36.…………
people at the greatest risk from factors beyond their control 37………….
a far-reaching change in official attitude 38………….
a benefit for those the scheme was not originally 39…………. 40………….
intended for
(NB. You must provide two different options)
the bringing of a source of energy to remote areas 41………….
a failure to adapt in order to meet increasing demands 42………….
Use the word given in brackets to forn a word that fits in the gap. Write the word in the corresponding
numbered boxes on the answer sheet. The first one has been done as an example (0).
Example: (0). PROJECTIONS/projections
Today, population largely means urban population growth. United Nations (UN) (0. PROJECT)…….

95
show the world’s rural population has already stopped growing, but the world can expect to add close to 1.5
billion (43. URBAN)……………….in the next 15 years, and 3 billion by 2050. How the world meets the
challenge of sustainable development will be (44. INTIMATE)……………….tied to this process.
For many people, cities represent a world of new opportunities, including jobs. There is a powerful
link between urbanization and economic growth. Around the world, towns and cities are responsible for over
80 per cent of gross national product. While urban poverty is growing around the world, tis is largely because
many people - including the poor - are moving to urban areas.
The opportunities there extend beyond just jobs. Cities also offer greater opportunities for (45.
SOCIETY)……………….mobilization and women's empowerment. Many young people, especially young
women, regard the move to cities as an opportunity to escape traditional partiarchy and experience new
freedoms. Urban areas also offer greater access to education and health services, including sexual and
reproductive health care, further (46. PROMOTE)……………….women's empowerment and the realization
of their reproductive rights. This contributes to significantly reduced (47. FERTILE)……………….in urban
areas, changing the trajectory of overall population growth.
This process, which is particularly (48. PRONUNCIATION)……………….in Africa and Asia,
where much of the world's population growth is taking place, is also an enormous opportunity for
sustainibility, if the right policies are put in place. Urban living has the potential to use resources more (49.
EFFICIENCY)………………. To create more sustainable land use and to protect the (50. DIVERSE)……..
of natural ecosystems.
Still, the face of (51. EQUAL)……………….is increasingly an urban one. Too many urban
residents grapple with extreme poverty, (52. EXCLUDE)………………., vulnerability and marginalization.
(Source: http:/www.unfpa.org/urbanization)
Read the text below and complete each blank with ONE suitable word. Write the word in the
corresponding numbered boxes on the answer sheet. The first one has been done as an example (0).
Example: (0) in
COMPUTER WISE
From the day we feel our children are capable of understanding, we begin educating them about the
perils ever-present (0)……………our modern world. We teach them that they must never cross the road (53)
……………looking both ways, accept lifts from strangers, or take sweets from people (54)…………… don't
know; the list (55)……………on and on.
In (56)…………… of those warnings, isn't it ironic that we still ignore one of the biggest threats of
all....the Internet? In this age of the PC - (57)……………every house has one - we as parents often fail (58)
……………our duty to protect our children from this potential source of danger.
Just as we supervise what our little ones - and (59)……………so little ones watch on television, we
should also be (60)……………of waht they're doing while they're surfing the Net. Of course, we can't be
forever looking over their shoulders when they're online, but there are ways of increasing Internet safety.
Using software to filter and block what young users can access, warning children (61)……………giving out
any personal information, and talking,openly about cyber hazards, (62)……………a few of the really
imporatnt first steps.
Fill in each blank with a suitable preposition given in the box. There are more prepositions than
needed. Write your answer in the corresponding numbered boxes on the answer sheet.
off - for - down - around - on - up - out - over

63.Suddenly the lights went ut, and she wasn’t able to make……………where the exit was.
64. Her father decided to hand……………his entire estate to her.
65. I have a headache and I certainly don't feel……………to going to watch a concert.
66. While the office computer system was being fixed, there was nothing to do so we just sat…………
67. She wasn't interested in his money. She fell……………his sense of humour.
Find the only ONE word which can be used appropriately in each set of three sentences. Write your

96
answer in the corresponding numbered boxes on the answer sheet. The first one has been done as an
example.
Example:
(0) I learn a great……………on the course.
How should I……………with angry customers?
The two companies have signed a two-year business……………
Answer: (0). deal
68. When speaking in front of a large audience, you should bear in……………that everybody should be able
to both hear and see you.
It's strange how its walys seems as if Emily can read my…………….
It's getting to be a cliche now, but i still believe that travel can broaden the……………
69. The event was……………in an old castle by the sea.
Many of the team members were……………up by a last-minute flight cancellation.
After I finished playing, she told me that I hadn't…………… the instrument correctly.
70. There has been a……………in the number of Internet users this year.
It must be difficult to cope with a sudden……………to fame.
Employees were given a 10% pay……………at the beginning of the financial year.
Complete the second sentence in such a way that it means exactly the same as the sentence printed
before it. Use the word given in capital letters at the end of the original sentence. DO NOT change the
given word. Use between three and six words, including the word given.
71. I was amazed because there were no problems throughout the holiday. (WENT)
To………………………………………………………………………........wrong throughout the holiday.
72. No matter what happens. Jane will never forgive Mark for what he did. (EVER)
Under……………………………………………………………………….......Mark for what he did.
73.. Can anyone slove this problem? (COME)
Can anyone……………………………………………………………………….......this problem?
74. You probably won't find a better deal for a holiday. (CHANCES)
The………………………………………………………………………......find a better deal for a holiday.
75. Apparently the enntire warehouse was destroyed in a fire last month. (FLAMES)
Apparently the enntire warehouse…………………………………………………last month.
Complete the second sentence in such a way that it means exactly the same as the sentence printed
before it.
76. People think that heavy traffic caused the subsidence.
The subsidence……………………………………………………………………….......
77. She was so popular that everyone voted for her.
Such..………………………………………………………………………......................
78. We got out of the building safely only because the smoke alarms went off.
But for ……………………………………………………………………….....................
79.I don’t like the way that Jane gets angry so often with other staff.
I wish……………………………………………………………………….......................
80. Evan is doing very well at school, especially if you consider that he’s younger than the others in his class.
Given……………………………………………………………………….......................
The passage below contains 5 errors. For questions 81-85, indicate the line number, the errors and
correction in the space provided on the answer sheet. The first one has been done as an example (0).
Example: (0). line 1: a place => place
WORLD BOOK DAY
1 This year's World Book Day (WBD), which is taking a place on March 2., hoping to encourage everyone,
and especial children, to discover the joy of reading.
Schools and libraries are getting involved, with a packed schedule of events designed to bring books

97
to life. There will be writers popping into schools to read from their books and answer 5 questions, and
telling-story events. Children will also be able to take part in reading so that they really have a chance to
engage with the books.
As long as hoping to encourage children to cath the reading bug, WBD also hopes to get reluctant
adults hooked on books. So, for the first time, WBD will also have an adult focus, with the launch of Quick
Reads, a selection of short, fast-paced stories by well-known authors. The first set of 10 Quick Reads will be
published on WBD, with a further collection of books being released latter
in the summer.
Write a paragrpah about the most essential skill a successful 21st-century learner should acquire. You
should write about 150 words.
……………………………………………………………………….................................................
……………………………………………………………………….................................................
……………………………………………………………………….................................................
……………………………………………………………………….................................................

ĐỀ THI NĂM 2018-2019


ĐỀ SỐ 22
(Đề thi vào lớp 10 THPT Chuyên TP. Hồ Chí Minh)
Thời gian làm bài: 150 phút
I. USE OF ENGLISH.
PART A. CHOOSE THE CORRECT ANSWER TO FILL IN THE BLANKS.
1. Every Christmas of my childhood was the same. My father…………late for lunch, weighed down with
presents for the family.
A. would arrive B. could arrive
C. was arriving D. got used to arriving
2. If I…………you a free ticket, would you go to Florence with me?
A would offer B. were to offer C. had offered D. were offered
3. "You…………things. Can't you be more careful?" sad Tom's mother.
A. always lose B. are losmg
C. always lost D. are always losing
4…………in the next room, her voice is likeof a boy.
A. Hearing - the sound B. Hearing - the one
C. Heard - that D. Having been heard - that one
5. I'd say let's meet on Saturday but I’m none…………sure what’s happening at the weekend.
A. so B. very C. that D. too
6. When the exam is over. I'll go fishing…………I haven't done for weeks.
A anything B. something C. things D. everything
7. We're going to visit the Great Walls. Everybody says this is…………
A. a too good chance for being lost B. too good a chance to be lost
C. too good a chance for being lost D. a too good chance to lose
8.…………, her suggestion is of greater value than her friend's.
A. All things considered B. All things considering
C. Considering all things D. Considered all things
9. As far as I am concerned, education is about learning and the more you learn,…………
A the more for life are you equipped B. the more life you equip for

98
C. the more equipped for life you are D. the more Me you are equipped for
10. He wants to give his daughters…………he can afford.
A. all for the best B. the very best C. at best D. what best
11…………Sam had was gone when she heard that nearly all her classmates had failed to answer the
teacher’s question.
A. A little confidence B. The little confidence
C. Little confidence D. Little of confidence
12…………the difficulty of the task, I shall be lucky to have completed it by June.
A. Regarding B. Presuming C. Given D. Encountered
13. Diana took a course in shorthand and typing…………applying for a secretary job.
A. on account of B. with a view to C. with a reason for D. for fear of
14. Jessica has bungled every task her boss has given her so far…………, he’s prepared to give her one last
chance.
A. Notwithstanding B. Instead C. Fiathermore D. Nevertheless
15. The restaurant is popular with film stars and the…………
A same B. like C. such D. similar
16. Tom's decision to leave university after a year is one he now…………regrets.
A. painfully B. harshly C. heavily D. deeply
17. Poltutants in this river are increasing………… - something must be done about it immediately.
A. chillingly B. utterly C. rigorously D. alarmingly
18. Although his paintings are abstract, the artist…………inspiration from the natural world.
A. draws B. pulls C. makes D. has
19. She…………with pleasure at the unexpected compliment.
A. grinned B. glared C. beamed D. laughed
20. As Jane lay in her cabin, she heard the gentle sound of waves…………against the hull.
A pelting B. splashing C. gushing D. lapping
21. David was so…………by the beauty of Ha Long Bay that he instantly decided to move there.
A. captured B. captivated C. accumulated D. beckoned
22. Many species of fish have…………….ways of protecting their eggs from predators.
A. ingenious B. indicative C. inspiring D. ingenuous
23. No one knows how the rival company got……………of the plans for thee new marketing campaign.
A. wind B. breere C. voice D. ear
24. Family relationships later……………a great significance in his life.
A. built up B. kept on C. took on D. made up
25. For non-native speakers, it is not unusual for understanding to……………when listening to others’
conversations in English.
A. fall away B. break down C. give up D. set out
26. Since we had only one day left, we decided to make an……………effort to finish the run in record time.
A all-in B. all-out C. over-all D. all-around
27. Jukan is one of the many young,……………artists to be taken on recently by an important gallery.
A. bottom-up B. up-and-coming C. top-downm D. out-and-out
28. Jimmy's not interested in the……………He just wants to know the plan facts.
A. cut and thrust B. hue and cry C. ifs and buts D. part and parcel
29. ~ Mai: “……………”
Ian: 'I'm pretty busy right now. I'm doing my homework because I have an exam tomorrow."
A. How is your day going? B. How are you doing?
C. What do you do? D. What are you doing here?
30. Student 1: ’’Excuse me! Could you show me the way to the library?
Student 2: ’’Sorry, I'm new here. Student 1: “……………”

99
A. Not at all. B. Bad luck.
C. No problem. D. Thank you all the same.
PART B. CHOOSE THE WORD OR PHRASE THAT FITS EACH SPACE IN THE FOLLOING
PASSAGE.
In recent years, ready-made meals have (1)…………….Britain’s eating (2) ……………. Britons
now spend four times as much as the Italians on ready-made meals and six times more than the Spanish. (3)
……………. for instant meats has increased across Europe as a (4) …………….but why has Britain
become the (5) …………….European capital of ready-made food, second only in the world to America?
Convenience is (6) …………….of the attraction. A recent survey (7) …………….that 77 percent
of purchasers said they only bought ready meals when they did not have time to cook. Dr Susan Jebb, head of
nutrition at the Medical Research Council, said: "People in the UK work the longest hours, we are very time-
poor, and we don't have a strong (8) …………...history of cooking.'
The ready-made meal boom also reflects changing social (9) …………….in Britain. More people live
alone and so are less likely to be (10) …………….to cook. And with families eating together less often,
ready mealsh allow people to eat what they want when they want. Juliaa Michna, of Marks and Spencer, says
that ready meals also reflect changing (11) …………….infood. 'Britain's multiculturalism has brought a
(12) …………….range of restaurants than other European countries, and ethnic cuisines, which people are
often scared of cooking from (13) …………….are (14) …………….more popular. One quarter of (15)
…………….meals are Indian, and nearly one in Five are Chinese.
1. A. amended B. adjusted C. transferred D. transformed
2. A. ways B. forms C. habits D. manners
3. A. Request B. Order C. Demand D. Charge
4. A. conclusion B. total C.resur D. whole
5. A. unclaimed B. unclassified C. undefeated D. undisputed
6. A. element B. piece C. part D. share
7. A. found B. made C. put D. gave
8.A. tradtonai B. cultural C. modern D. customary
9. A trends B. temptations C. drifts D. movements
10. A. offended B. bothered C. worried D. disturbed
11. A. desires B. likings C. tastes D. washes
12. A. longer B. deeper C. harder D. wider
13. A. scratch B. beginning C. memory D. nowhere
14. A very B. lot C. far D. such
15. A. chilled B. decent C. meager D. junk
II. READING.
PART A. READ THE PASSAGE AND CHOOSE THE BEST ANSWERS
TO THE QUESTIONS.
The radical change in the land's surface that results when rural areas are transformed into cities is a
significant cause of the rise in temperature in cities that is known as urban heat island.
First, the tall buildings and the concrete and asphalt of the city absorb and store greater quantities of
solar radiation than do the vegetation and soil typical of rural areas.
In addition, because the concrete and asphalt are impermeable, the runoff of water following a rain is
rapid, resulting in a severe reduction in the evaporation rate. So heat that once would have been used convert
liquid water to a gas goes instead to increase the surface temperature further.
At night, although both city and countryside cool through radiation losses, the stone-like surface of
the city gradually releases the additional heat accumulated during the day, keeping the urban air warmer than
that of the outlying areas.
Part of the urban temperature rise must also be attributed to waste heat from such sources as home
heating and air conditioning, power generation, industry, and transportation. Many studies have shown that

100
the magnitude of human-made energy in metropolitan areas is equal to a significant percentage of the energy
received from the Sun at the surface.
Investigations in Sheffield, England, and Berlin showed that the annual heat production in these cities
was equal to approximately one-third of that received from solar radiation. Another study of the densely
built-up Manhattan section of New York City revealed that during the winter, the quantity of heat produced
from combustion alone was two and one-half times greater than the amount of solar energy reaching the
ground. In summer, the figure dropped to one-sixth.
It is interesting to note that during the summer there is a mutual reinforcement between the higher
nighttime temperatures of the city and the human-made heat that helped create them. That is, the higher
temperatures result in the increased use of air-conditioners, which, in turn, use energy and further increase
the amount of urban heat. During the winter the nighttime warmth of urban areas, produced in large part by
heavy energy consumption, is beneficial because less energy needed to heat buildings.
1. What does the passage mainly discuss?
A. The loss of farmland to urban development
B. The causes of increased heat in cities
C. Waste heat generated by home heating and air conditioning
D. How seasonal change affects the temperature of cities
2. All of the following contribute to the urban heat island effect EXCEPT......
A. absorption of heat from the Sun
B. storage of heat from the Sun
C. an increased rate of evaporation after a rainfall
D. the release of heat at night from city surfaces
3. The word "convert" in the passage is closest in meaning to…………
A. reverse B. transform C. reduce D. compare
4. The word "that" in the passage refers to…………..
A. city B. heat C. day D. air
5. In which of the following locations would the rate of evaporation probably be highest?
A. A rural area B. A small town C. A medium-sized city D. A big city
6. The word "magnitude” in the passage is closest in meaning to................
A. calculation B. comprehension C. extent D. formation
7. The author mentions Manhattan to order to demonstrate that………….
A. heat in urban areas can be reduced
B. the conclusions of the investigation in Sheffield were wrong
C. its heat production is smaller than that of Berlin
D. human-made heat can exceed the solar energy that reaches the ground inwinter
8. According to the passage, on important consequence of the use of air- conddroners at night is...............
A. greater energy costs
B. higher levels of urban heat
C. senous problems with the energy supply
D. less need for air conditioning in the morning
9. The word "beneficial" in the passage is closest in meaning to.................
A. predictable B. powerful C. hazardous D. advantageous
10. Which of the following is true about cities at night in the winter?
A. Solar energy has an increased impact on the urban heat island.
B. They tend to be colder than rural areas.
C. Less energy is required to heat buildings
D. Human-made energy created a larger area of total heat than solar energy.
PART B: REARRANGE THE FOLLOWING SENTENCES SO THAT THEY MAKE A
MEANINGFUL REVIEW OF A COMPUTER GAME.

101
A. A selection of viewpoints, including a breakneck-'biker's eye view are offered. This game will push your
skills and patience to the limit.
B. There are 10 tracks, of which three are available at the outset. Only by scoring gold in both classes on all
of these can you gain access to the next three, and so on.
C. Wonder Racer succeeds in bringing the body breaking speed of time-trial biking to the PC, but its
difficulty may leave you shaking. The approach is simple and unsophisticated.
D. The courses are a fictitious mix of country lanes, exotic beaches and snowy mountain passes. The 3-0 js
excellent in its speed, smoothness and level of detail.
E. There are only three controls, far fewer than in many other modem games. Players start by selecting one of
sixteen riders, from a set of teams.
PART C. SUPPLY EACH BLANK WITH ONE SUITABLE WORD. PASSAGE I
It is forecast that we can look forward to working (1)……………hours in the future, but it is
necessary for health and tranquility to work a certain (2) ……………of hours per week, ideally doing a
variety of jobs - something schools have always known. It may be that house building will meet this need. It
is a very basic human instinct. Gardening is a related activity. It is already (3) ……………to cultivate many
hurts and vegetables than to buy them in the shops and the house of the next decade should take this into (4)
……………
(5) …………… important question is that of energy conservation. The proportion of income (6)
……………on keeping warm is steadily going up, and, with the cost of energy likely to double in real terms
during the next ten years or (7) ……………many large bady-insulated old houses will become extremely
expensive to use. The demand will be (8) ……………small, well-insulated homes located in warm protected
areas and making the best (9) …………… of the sun's warmth. Efficient heating units will be of prime
importance. At (10) ……………, we waste a lot of space in planning rooms which are awkward to use.
PASSAGE 2
We live surrounded by objects and systems that we take for (1) ……………,but which profoundly
affect the way we behave, think, work, play, and in general lead our (2) …………… Look, for example, at
the place in which you are reading this now, and see how much of (3) ……………surrounds you is
understandable, how much of it you could actually build yourself or repair (4) ……………it cease to
function. When we start the car or press the (5) ……………in the elevator, or buy food in the
supermarket, we gave no (6) …………… to the complex devices or systems that make the car move, or the
elevator rise, or the food appear on the shelves.
Throughout this century we have become increasingly dependent on the products of (7) ……………
They have already changed our lives: at the simplest (8) …………… , the availability of transport has made
us physically less fit than our ancestors. Many people are alive only because they have been given (9) ……..
to disease through drugs. The vast majority of the world's population relies on the abiliy of technology to
provide and transport food. We are unable to feed and clothe or keep (10) …………warm without
technology.
ERROR CORRECTION
IDENTIFY THE FIVE (5) MISTAKES IN THE FOLLOWING PASSAGE AND CORRECT THEM.
LINE
NUMBERS
1 An ecosystem is a group of animals and plants living in a specific region and
2 interact with one another and with their physical environment. Ecosystems
3 include physical and chemical components, such as soils, water, and nutritions
4 that support the organisms living there. These organisms may range from large
5 animals to microscope bacteria. Ecosystems also can be thought of as the
6 interactions among all organisms in a given habitat; for instance, one species may
7 serve as food for other. People are part of the ecosystems where they live and
8 work. Hunan activities cah harm or destroy local ecosystems unless actions such

102
9 as land development for housing or businesses are careful planned to conserve
and sustain the ecology of the area. An important part of ecosystem management
involves findng ways to protect and enhance economical and social well-being
while protecting local ecosystems.

IV. WORD FORMS.


SUULY THE APPROPROATE FORMS OF WORDS IN THE BRACKETS.
1. His tour was cut…………..short due to his illness. [appoint]
2. Since the city center became a (n )…………..pedestrian aiea, shopping has been a more pleasant
experience. [traffic]
3. I've…………..the CNN homepage as I use it regularly to get the latest news. [mark]
4. The…………..at the temple are bringing heart for peace and prosperity. [devotion]
5. A (n )…………..life, lack of physical activity, stressful jobs, and bad habits can influence one's heath in a
very bad way. [pace]
6. Officials urged caution in anticipation of catastrophic and…………..flooding in the days to come. [threat]
7. The store manager explained…………..that only certan items were sold at a discount. [apology]
8. Women are traditionally supposed to be good at…………..[task]
9. The recent events…………..the need for a better understandng of the environmental impact of
biotechnology. [score]
10 …………..fines may be issued for careless driving and other offenses. [spot]
V. WRITING.
REWRITE THE FOLLOWING SENTENCES WITHOUT CHANGING THE MEANING. YOU
HAVE TO USE THE EXACT WORD GIVEN IN BRACCKETS FOR EACH SENTENCE.
1. I think you should get someone to fix the computer. [fixed]
I would prefer it………………………………………………………………………………….
2. Jack was so nervous that his mind couldn't function properly. [straight]
Such………………………………………………………………………………………………
3. Suddenly, the management said it was important for us to wear dark suits to the meeting. [once]
The management insisted…………………………………………………………………………
4. To say briefly, this school regulation cannot be abolished right away. [do]
In a………………………………………………………………………………………………..
5. Trying to persuade someone to agree with you can be a formidable task. [view]
Trying to bring……………………………………………………………………………………
6. Anna inherited a fortune when her father died. [into]
After the…………………………………………………………………………………………..
7. We were not late for school because you took us in your car. [time]
If it hadn’t been for……………………………………………………………………………….
8. I managed to finish the task, but it was difficult. [succeed]
Only with…………………………………………………………………………………………
9. Although she didn’t agree with the management's decision, Chloe was forced to accept it. [choice]
Much……………………………………………………………………………………………..
10. Fred didn’t tell Sopite his news until she had finished her homework. [telling]
Fred waited...................................................................................................................................

103
ĐỀ SỐ 23
(Đề thi vào lớp 10 THPT Chuyên Hà Nam)
Thời gian làm bài: 120 phút
SECTION A. PHONETICS (10 points)
part I. Choose the word whose underlined part is pronounced differently from that of the rest (5
points)
1. A. dosage B. massage C. carriage D.voyage
2. A. architect B. parachute C. choir D. psychology
3. A. walked B. naked C. sacred D. wicked
4. A. religion B. logical C. hamburger D. teenager
5. A. waste B. dangerous C. ancient D. natural

Your answers:
1. 2. 3. 4. 5.

Part II. Choose the word that has a different stress pattern from the others in the group (5 points)
1. A. cartoon B. westen C. teacher D. theatre
2. A. politics B. advantage C. predator D.energy
3. A. photography B. financial C. ambitious D. politician
4. A. undoubtedly B. economic C. entertainment D. agricultural
5. A. responsibility B. environmentally
C. accommodation D. mischievousness

Your answers:
1. 2. 3. 4. 5.

SECTION B. LEXICO AND GRAMMAR (25 points)


Part I. Choose the word, phrase or expression which best completes each sentence (5 points)
1…………..the phone rang later that night did Anna remember the appointment.
A. Just before B. Not until C. Only D. No sooner
2. Looking down at the coral reef, we saw …………..of tiny, multi-coloured fish.
A. teams B. flocks C. schools D. swarms
3. She insisted that the reporter…………..her as his source of information.
A. didn’t mention B. doesn’t mention C. hadn’t mentioned D. not mention
4. You will have to…………..your holiday if you are too ill to travel.
A. put out B. put off C. put down D. put up
5. She had butterflies in her…………..before the interview.
A. face B. heart C. palms D. stomach
6. There was hardly…………..money left in my bank account.
A. more B. no C. some D. any
7. Give me your telephone number…………..need your help.
A. in case B. so that C. unless D. whether
8. Do you like the language center…………..in the women’s Magazine?
A. advertiser B. advertised C. advertising D. advertisement

104
9. Thomas has a garden which is…………..mine.
A. double as large B. semi-larger than
C. twice as large as D. as two-time as large
10. ~ A: “…………..” ~ B: “Well, what I mean is, I think it’s a good idea.”
A. Do you mean it? B. What’s the meaning of this word?
C. Is it what you mean? D. I don't get what you mean.
Your answers:
1. 2. 3. 4. 5. 6. 7. 8. 9. 10.

Part II. Give the correct form of the given words in brackets (10 points)
RUNNING FOR HEALTH
If you want to improve your overall level of fitness, running is one of the best sports to choose. It can
increase the strength of your bones, it is good for your heart, and it can help with weight (1.LOSE) ………...
You will soon begin to see a significant (2. IMPROVE) ……………... in your general health and if you are
the sort of person who enjoys a challenge, then you could consider making the decision to run a marathon-a
race of (3. APPROXIMATE) ……………...42 kilometres.
(4. EXPERIENCE) ……………...runners and sports instructors say you should make a point of
having a physical (5. CHECK) ……………...with your doctor before you start training. Another (6.
RECOMMEND) ……………... is that anyone who has an old back, knee or ankle injury should take extra
care. You should never use running shoes which hurt your feet or in which you feel (7. COMFORT)
……………... after a long run. It is a good idea to start by running slowly for about twenty minutes three
times a week and (8.GRADUAL) ……………...increase the number of kilometres you run. You should find
running long (9.DISTANT) ……………...gets progressively easier, and after a time, you may feel that even
a marathon will be (10. POSSIBILITY) ……………...!
Your answers:
1………………….. 6.…………………..
2.………………….. 7.…………………..
3…………………... 8…………………...
4.………………….. 9.…………………..
5.………………….. 10.…………………..
Part III. There is one mistake in each numbered line in the following passage.
Find, underline and correct them (5 points)
People are destroying the Earth. The seas and rivers are so dirty to swim in. There 1………………..
is so much smoke in the air that it is healthy to live in many of the world’s cities. 2………………..
In one well-known city, for example, poison gases from cars pollute the air so 3………………..
much that policeman have to wear oxygen masks. We have cut on so many trees 4………………..
that there are now vast areas of wasteland all over the world. As a result, so many 5………………..
farmers in parts of Africacan donot enough grow to eat. In some countries in 6………………..
Asian there is so little rice. Moreover, we do not take enough care of the 7………………..
countryside wild animals are quick diappearing. For instance, tigers are rare in 8………………..
India now because we have killed too much for 10 them to survive. However, it 9………………..
isn’t so simple to talk about a problem. We must act now before it is too late to do 10………………..
anything
I about it. Join us now!

Part IV. Give the correct tense or form of the verbs in brackets (5 points)
1. She left her home last week and (not see)………………..ever since.
2. My teacher encouraged us (take) ………………..part in the English contest.
3. My brother often fell asleep while he (do)………………..his homework.

105
4. I'm very exhausted now because I (run)………………...
5. It was the second time we (visit)………………..Ha Long Bay.

Your answers: 3.
1. 4.
2. 5.

SECTION C. READING COMPREHENSION (25 points)


Part I. Fill in each gap with ONE suitable word to complete the following paragraph (5 points)
Solar energy is a long lasting source of energy which can be used (1) ……………...anywhere. To
generate solar energy, we only need solar cells and the sun! Solar cells can easily (2) ……………...installed
on house roofs, so no new space is needed and each user can quietly generate their (3) ……………... energy.
Compared to other renewable sources, they also possess many advantages. Wind and water power rely on
turbines (4) ……………...are noisy, expensive and easy to break down. Solar cells are totally silent and non-
polluting. As they have no moving parts, they require little maintenance and have (5) ……………...long
lifetime.
However, solar energy also has some disadvantages. We can only general solar energy during (6)
……………...because the system depends on sunlight. Beside, solar cells require (7) ……………...area to
work effectively. The main disadvantage of solar energy is that it (8) ……………...about twice as much as
traditional sources such as coal, oil, and gas. This is because solar cells are (9) …………….... Scientists are
hoping that the costs of solar cells will reduce as more and more people see th advantages of this
environmentally (10) ……………...source of energy.

Your answers:
1. 2. 3. 4. 5.
6. 7. 8. 9. 10.

Part II. Choose the best word A, B, C or D to fill in spaces in the following passage (10 points)
Reality television is a genre of television programming which, it is claimed, presents unscripted
dramatic or humorous situations, documents (1) ……………...events, and features ordinary people rather
than professional actors. It could be described as a form of artificial or "heightened" documentary. (2)
……………... the genre has existed in some form or another since the (3) ……………... years of television,
the current explosion of popularity dates from (4) ……………...2000.
Reality television (5) ……………...a wide range of television programming formats, from game or
quiz shows which resemble the frantic, often demeaning programmes (6) ……………... in Japan in the
1980s and 1990s (a modern example is Gaki no tsukai), to surveillance- or voyeurism-focused productions
(7) ……………...Big Brother.
Critics say that the (8) ……………... "reality television" is somewhat of a misnomer and that such
shows frequently portray a modified and highly influenced form of reality, (9) ……………... participants put
in exotic locations or abnormal situations, sometimes coached to act in (10) ……………...ways by off-screen
handlers, and with events on screen manipulated through editing and other post-production techniques.

1. A. factual B. actual C. real D. interesting


2. A. Because B. If C. Despite D. Although
3. A. early B. earliest C. earlier D. more early
4. A. in B. during C. around D. with
5. A. covers B. spreads C. stretches D.expands
6. A.which produced B. produced C. which was produced D. producing

106
7. A. such B.such as C. as D. from
8. A. saying B. words C. term D.
definition
9. A. for B. into C. with D. with
10. A. other B. special C. own D. certain

Your answers:
1. 2. 3. 4. 5. 6. 7. 8. 9. 10.

Part III. Choose the most suitable heading from the list A - F for each paragraph from 1 - 5 of the article. Write them in
the numdered blanks (5 points)
A. An easy life
B. Less money; less waste
C. Home ’s like people
D. Not all good news
E. Starting to take control
F. Everything under control

INTELLIGENT HOUSING
1
Have you ever thought about how your body works? Everyday, you make your body do lots of
different actions, such as when you walk, talk, or dance. But at the same time, your body automatically does
many other things that are essential for everyday life. For instance, when you blink or yawn or breathe, you
don’t have to think about what you are doing; your body simply carries out these task for you. You might
think this is strange, but the latest houses are like this, too.
2
Now, thanks to the work of some far- sighted architects and designers, houses have become “more
intelligent”. It isn’t quite true to say that houses are starting to think for themselves, but some modem homes
are now using sophisticated technology to control many of the routine jobs that up to now have always been
our responsibility.
3
For years, architects and builders have been trying to find ways to make houses more economical and
comfortable to live in, as well as trying to reduce the amount of damage that houses do to the environment.
“Intelligent” houses seem to be the ideal solution. Clever use of building materials and techniques has
brought us houses that are easier and cheaper to keep warm than traditional housing. Modern water systems,
together with the recycling of washing water from bathrooms and kitchens, means that “intelligent” houses
use 30% less water than convetional homes.
4
“Intelligent” homes might look a little unusual from the outside. This is because the builders use a lot
of glass and wood to make the walls. Some of the houses even have a grass roof because it is kinder to the
atmosphere! But, inside, they are comfortable and bright. The technology, which is similar to that of your TV
remote control, makes sure of this. There are always enough lights on in every room and the windows close
when it starts to rain. The water is always at the correct temperature whenever you want to have a shower and
the house doesn’t allow you to use more water than you need. You can even watch TV in every room, or use
the close- circuit cameras to see what’s happening in the garden.
5
Until recently, we have always had to think about everything we want our houses to do. If we have
wanted it to be warmer or cooler, lighter or darker, we had to turn the heating or lighting on or off. However,
for some families already live in “intelligent”houses, daily life has changed in many ways. They don’t worry

107
about turning off the light or locking the door before they go out. The house does all this for them. Families
in “intelligent” houses don’t have to do as much work to keep their home clean and comfortable- and the hi-
tech kitchen means it isn’t difficult to prepare family meals.
Part IV. Read the passage below, then choose the correct answer A, B, C or D to each of the following
questions (5 points)
In addition to providing energy, fats have several other functions in the body. The fat-soluble
vitamins, A, D, E and K, are dissolved in fats, as their name implies. Good sources of these vitamins have
high oil or fat content, and the vitamins are stored in the body’s fatty tissues. In the diet, fats cause food to
remain longer in the stomach, this increasing the feeling of fullness for some time after a meal is eaten. Fats
add variety, taste, and texture to foods, which accounts for the popularity of fried foods. Fatty deposits in the
body have an insulating and protective value. The curves of the human female body are due mostly to
strategically located fat deposits.
Whether a certain amount of fat in the diet is essential to human health is not definitely known. When
rats are fed a fat - free diet, their growth eventually ceases, their skin becomes inflamed and scaly, and their
reproductive systems are damaged. Two fatty acids, linoleic and arachdonic acids, prevent these
abnormalities and hence are called essential fatty acids. They also are required by a number of other animals
but their roles in human beings are debatable. Most nutritionists consider linoleic fatty acid an essential
nutrient for humans.
1. This passage probably appeared in which of the followings?
A. A diet book B. A book on basic nutrition
C. A cookbook D. A popular women’s magazine
2. According to the passage, which vitamin is not stored in the body’s fatty tissue?
A. Vitamin A B. Vitamin D C. Vitamin B D. Vitamin E
3. The author states that fats serve all the following body function EXCEPT…………..
A. promote a feeling of fullness B. insulate and protect the body
C. provide energy D. control weight gain
4. The word “essential” in line 8 is closest in meaning to…………..
A. required for B. desired for C. detrimental to D. beneficial to
5. According to the author, which of the following is true for rats when they are fed a fat free diet?
A. They stop growing B. They have more babies
C. They lose body hair D. They require less care
Your answers:
1. 2. 3. 4. 5.

SECTION D. WRITING (20 points)


Part I. Finish each of the following sentences in such a way that it means exactly the same as the
sentence printed before it (5 points)
1. It’s thought that he is staying in London at the time of the accident.
=> He is………………………………………………………………………………………………
2. The train left before he got to the station.
=>By the time ……………………………………………………………………………………….
3. If Cathy didn’t help me, I wouldn’t finish my work on time.
=> But .………………………………………………………………………………………………
4. Linda was sorry that she didn’t say goodbye to her relatives at the airport.
=> Linda regretted…………………………………………………………………………………..
5. He never suspected that the money had been stolen.
=> At no time……………………………………………………………………………………….
Part II. Rewrite each of the following sentences so that it means exactly the same as the given one. Use
the given word in CAPITAL letters. Do not change the word (5 points)

108
1.I can’t believe that he passed the exam.
FIND………………………………………………………………………………………………..
2. This traditional craft village is often transferred from generation to generation.
DOWN………………………………………………………………………………………………
3. His coming to the party last night was unexpected
BLUE………………………………………………………………………………………………..
4. Could you guard my handbag while I go to the shop?
EYE………………………………………………………………………………………………….
5. He studies hard, as the result, he gets good marks.
THE………………………………………………………………………………………………….
Part III. Writing a paragraph (10 points)
Recently, there have been some changes in the roles of teachers and students.
Write a paragraph of about 180 words about the changes and their effects.
……………………………………………………………………………………………………….
……………………………………………………………………………………………………….
……………………………………………………………………………………………………….
……………………………………………………………………………………………………….

ĐỀ SỐ 24
(Đề thi vào lớp 10 THPT Chuyên Bình Phước)
Thời gian làm bài: 120 phút
HƯỚNG DẪN PHẦN THI NGHE HIỂU
- Trước khi bắt đầu mỗi phần thí sinh có 20 giây để đọc đề.
- Sau mỗi phần thí sinh có 1 phút để kiểm tra đáp án.
- Bài nghe gồm 2 phần, mỗi phần nghe 2 lần.
A. LISTENING - TRACK 6 (1 pt)
Part 1: You will hear an interview with a teenager called Amy Manero, who is talking about her
interest in music, particularly jazz piano. For each question, choose the correct answer A, B or C.
1. When did Amy have lessons with a piano teacher?
A. at the age of four.
B. before she started secondary school.
C. after getting advice from her school music teacher.
2. How did Amy feel at her first big public event?
A. excited because she was wearing a wonderful costume.
B. nervous because it was her first time on stage.
C. surprised because the audience was very big.
3. What does Amy say about playing jazz with the school band?
A. She was better at it than the rest of the players.
B. She put lots of effort into learning about it.
C. She enjoyed it as soon as she started.
4. Amy gives piano lessons so that she can………...
A. help people who couldn’t normally afford them .
B. encourage lots of young people to play the piano.
C. improve her own playing technique.
5. Why does Amy write a blog on the internet?

109
A. to get some experience for becoming a journalist.
B. to tell people about her daily life as a musician.
C. to recommend events for people to go to.
Part 2. You will bear a boy called Adam telling his class about a hot air balloon flight he went on. For
each question, fill in the missing information in the numbered space.
ADAM’S HOT AIR BALLOON FLIGHT
6. The flight in the balloon lasted for……………………
7. Adam could see as far as the……………………from the balloon.
8. Adam didn’t recognize a local……………………from the air. Adam was surprised that it was warm in the
balloon basket.
9. Adam didn't like having to……………………in the basket.
10. The balloon finally landed on a……………………
I. TRẮC NGHIỆM NGÔN NGỮ (3,5 điểm)
PART 1: PHONETICS (0.5 pts - 0.1/ each)
Task 1: Choose the word (A, B, C or D) whose underlined part is pronounced differently from that of
the rest in each o f the following questions.
11. A. imagine B. valentine C. discipline D. magazine
12. A. dictation B. station C. repetition D. question
Task 2: Choose the word (A, B, C or D) whose main stress pattern is different from that of the rest in
each of the following questions.
13. A. minority B. priority C. necessary D. facility
14. A. delicious B. generous C. confidence D. energy
15. A. unidentified B. imagination C. disappointed D. interactive
PART 2: LANGUAGE FUNCTION (0.5 pts - 0.1/ each)
Choose the word or phrase (A, B, C or D) that best completes each of the following exchanges.
16. Daisy: “What an attractive hair style you have got, Mary!” - Mary: “……………”
A. Thank you very much! I am afraid. B. You are telling a lie.
C. Thank you for your compliment! D. I don't like your sayings.
17. Dick: “Sorry, Brian is not here.” - Peter: “……………”
A. Would you like to leave a message?
B. Can I take a message then?
C. Can I speak to Brian, please?
D. Can I leave a message then?
18. Susan: “Tom says he doesn’t like you!” - Lucy: “……………”
A. It makes nothing B. So do I C. I know he isn’t D. I don’t care
19. Tom: “May I smoke?” Jerry: “……………”
A. What suits you? B. You are free
C. Accommodate yourself! D. Go ahead!
20. Laura: “What a lovely house you have!” - Maria: “……………”
A. Of course not, it‘s not costly
B. Thank you. Hope you will drop in
C. I think so
D. No problem
PART 3: SYNONYM AND ANTONYM (0.5 pts - 0.1/ each)
Task 1: Choose the word or phrase (A, B, C or D) that is CLOSEST in meaning to the underlined part
in each of the following sentences.
21. The most important thing is to keep yourself occupied.
A. busy B. comfortable C. free D. relaxed
22. My uncle, who is an accomplished guitarist, taught me how to play.

110
A. skillful B. famous C. perfect D. modest
23. After many year of unsuccessfully endeavoring to form his own orchestras, Glenn Miller finally
achieved world fame in 1939 as a big band leader.
A. requesting B. trying C. offering D. deciding
Task 2: Choose the word or phrase (A, B, C or D) that is OPPOSITE in meaning to the underlined
part in each of the following sentences.
24. We offer a speedy and secure service of transferring money in less than 24 hours.
A. uninterested B. unsure C. open D. slow
25. The Red Cross is an international humanitarian agency dedicated to reducing the sufferings of wounded
soldiers, civilians and prisoners of war.
A. happiness B. worry and sadness
C. pain and sorrow D. loss
PART 4: LEXICO - GRAMMAR (2.0 pts - 0.1/ each)
Choose the word or phrase (A, B, C or D) that best completes each of the following sentences.
26. He is a specialist……………modern Vietnamese literature.
A. of B. at C. for D. in
27. David has not seen Linda for fifteen years and has……………of her address.
A. no idea B. nothing C. no thought D. no mind
28. The population of the world is growing at an alarming…………….
A. rate B. measure C. step D. cost
29. After Freddie……………school, he joined the army.
A. finishing B. finishes C. had finished D. has finished
30. Don't go too fast! I can't……………up with you.
A. go B. walk C. run D. keep
31. It is imperative……………what to do when there is a fire.
A. he must know about B. that everyone know
C. we knew D. that he knew
32. Our industrial output……………from $2 million in 2002 to $4 million this year.
A. was rising B. Rises C. Rose D. has risen
33. That hotel is so expensive. They……………you sixty pounds for bed and breakfast.
A. charge B. fine C. take D. cost
34. Under no circumstances……………in public places.
A. should we allow to be smoked B. should smoking be allowed
C. we should allow smoking D. smoking should be allowed
35. The carefully nurtured gardens with a wide……………of flowers and fruit trees have added elegance to
this place and made it a major tourist attraction.
A. amount B. number C. Species D. variety
36. Visitors to the local museum are mostly attracted by……………table.
A. an old wooden Chinese beautiful
B. an old beautiful wooden Chinese
C. a wooden old beautiful Chinese
D. a beautiful old Chinese wooden
37. While I was looking through my old albums the other day, I……………this photograph of my parents’
wedding.
A. saw about B. looked down C. came across D. made up
38. We've already bought the house but won't……………it until May, when the present occupants have
moved out.
A. take possession of B. keep track of
C. catch sight of D. gain recognition of

111
39. With……………, it is obvious that our approach to the problem was completely wrong.
A. retrospect B. hindsight C. afterthought D. review
40. She……………me a very charming compliment on my painting.
A. made B. showed C. look D. paid
41.I was immensely……………to hear that none of my relatives was killed in the bus accident.
A. shocked B. relieved C. enlightened D. healed
42. Some data surveyed by the researchers have already proved that there is no ……………for mother’s
milk.
A. alternative B. exchange C. substitute D. equivalent
43.I get the……………impression that you don't like her very much.
A. distinct B. distinction C. distinctive D. distinguishable
44. She does what she wants to do, showing little……………for the feelings of others.
A. regard B. sensitivity C. awareness D. perception
45. After she had made several disastrous decisions, people began to……………her judgment.
A. disbelieve B. inspect C. wonder D. question
II. Tự LUẬN KIẾN THỨC NGÔN NGỮ (1,5 điểm)
PART 1: WORD FORM (0.5 pts - 0.05/ each)
Give the correct form of the words in brackets to complete the following sentences.
46. In the exam we had to write one……………essay. DESCRIBE
47. The reports are treated as strictly…………… CONFIDENCE
48. They knew that a vaccine for the virus was……………possible. THEORY
48. The streets were……………with strings of coloured lights. LUMINOUS
49. Children normally feel a lot of……………about their first day at school. ANXIOUS
50. Safety tests on old cars have been……………throughout Europe. STANDARD
51. We eventually began the……………task of sorting through his papers. LABOUR
52. The engineering sector achieved significant……………last year. GROW
53. There is (an)……………simple explanation of what happened. SUPPOSE
54. The magazine offers tips on cutting your house……………costs. MAINTAIN
55. Over 30,000……………will run in the New York marathon. COMPETE
PART 2: ERROR CORRECTION (0.5 pts - 0.1/ each)
There are 05 errors in the following passage. Identify the errors, write the line number and correct
them. Write your answers in the numbered spaces below. Number (00.) is done as an example .
Line Passage
1 KEEPING OUR TEETH HEALTHY
2 It’s very important to have health teeth. Good teeth help us to chew our food. They also
3 help us to look nicely. How does a tooth go bad? The decay begins in a little crack in the enamel
4 covering of the tooth. This happens after germs and bits of food have collected there. Then the
5 decay slowly spreads inside the tooth. Eventually, poisonous goes into the blood, and we may
6 feel quite ill. How can we keep our teeth healthy? First, we ought to visit our dentist twice a
7 year. He can fill the small holes in our teeth before they destroy the teeth. He can examine our
8 teeth to check that they are growing in the right way. Fortunately, many people wait until they
9 have toothache before they see a dentist. Secondly, we should brush our teeth with a toothbrush
10 and fluoride toothpaste at least twice a day once after breakfast and once before we go to bed.
11 We can also use wood toothpicks to clean between our teeth after a meal. Thirdly, we should eat
12 food that is good for our teeth and our body: milk, cheese, fish, brown bread, potatoes, red rice,
13 raw vegetables and fresh fruit. Chocolate, sweets, biscuits and cakes are bad, especially when
14 we eat it between meals. They are harmful because they stick to our teeth and cause decay.

Line Error Correction

112
1 00. health →healthy
56. →
57. →
58. →
59. →
60. →

Part 3: PREPOSITIONS AND PHRASAL VERBS (0.5 pts - 0.05/ each)


Supply each space with a suitable preposition or adverbial particle to complete each of the following
sentences.
61. The certificates can be exchanged……………goods in any of our stores.
62. Their stated aim was to free women……………domestic slavery.
63. I think they’ve gone to the airport to see their boss……………
64. Her injuries are consistent……………having fallen from the building.
65. He made a good impression……………his first day at work.
66. Why should you be displeased……………the fun of the children?
67. Be temperate……………speech as well as in eating and drinking.
68. Rewards should be proportionate……………merit.
69. Where is Anna? She should be here……………now.
70. ……………heat from the sun, there would be no life on earth.
KỸ NĂNG NGÔN NGỮ
PART 1: READING (2 pts - 0.1/ each)
Task 1: GUIDED CLOZE TEST
Choose the word or phrase (A, B, C or D) that best fits each blank in the following passage.
Why is it that many teenagers have the energy to play computer games until late at night but can’t find the
energy to get out of bed (71)……………….for school? According to a new report, today’s generation of
children are in danger of getting so (72) ……………….sleep that they are putting their mental and physical
health at (73) ……………….. Adults can easily survive on seven to eight hours’ sleep a night,(74)……….
teenagers require nine or ten hours. According to medical experts, one in five youngsters (75) …………….
anything between two and five hours’ sleep a night less than their parents did at their age.
This (76) ……………….serious questions about whether lack of sleep is affecting children’s ability to
concentrate at school. The connection between sleep deprivation and lapses in memory, impaired reaction
time and poor concentration is well (77) ……………….. Research has shown that losing as little as half an
hour’s sleep a night can have profound effects (78) ……………….how children perform the next day. A
good night’s sleep is also crucial for teenagers because it is while they are asleep (79) ……………….they
release a hormone that is essential for their ‘growth spurt’ (the period during teenage years when the body
grows at a rapid rate). It’s true that they can, to some (80) ………………., catch up on sleep at weekends, but
that won’t help them when they are dropping off to sleep in class on a Friday afternoon.
71. A. behind time B. about time C. in time D. at time
72. A. few B. less C. much D. little
73. A. jeopardy B. threat C. risk D. danger
74. A. or B. because C. whereas D. so
75. A. puts B. gets C. brings D. makes
76. A. raises B. rises C. results D. comes
77. A. organized B. arranged C. established D. acquired
78. A. in B. on C. to D. at
79. A. at which B. which C. where D. that
80. A. rate B. extent C. level D. point

113
Task 2: READING COMPREHENSION
Read the passage and choose the best answer for each question below.
The ability to conduct electricity is one of the key properties of a metal. Other solid material such as
silicon can conduct electricity but only effectively at certain temperatures. Also, some substances such as salt
(sodium chloride) can conduct when molten or when dissolved in water. The ability of metals to conduct
electricity is due to how their atoms bond together. In order to bond together the metal atoms lose at least one
of their outermost electrons. This leaves the metal atoms with a positive charge and they are now strictly
ions. The lost electrons are free to move in what are known as a sea of electrons. Since the electrons are
negatively charged they attract the ions and this is what keeps the structure together.
An electric current is a flow of charge and since the electrons in the sea of electrons are free to move
they can be made to flow in one direction when a Source of electrical energy such as a battery is connected to
the metal. Hence we have an electric current flowing through the wire, and this is what makes metals such
good conductors of electricity. The only other common solid conducting material that pencil users are likely
to encounter is graphite (what the ‘lead’ of a pencil is made from). Graphite is a form of carbon and again the
carbon atoms bond in such a way that there is a sea of electrons that can be made to flow as an electric
current. Likewise, if we have an ionic substance like salt we can make the electrically charged ions flow to
create a current but only when those ions are free to move, either when the substance is a liquid or dissolved
in water. In its solid state an ionic substance like salt cannot conduct electricity as its charged ions cannot
flow.
Electrical insulators are substances that cannot conduct electricity well either, because they contain no
charged particles or any charged particles they might contain do not flow easily. Water itself is a poor
conductor or electricity as it does not contain a significant amount of fully charged particles (the ends of a
water molecule are partly charged but overall the molecule is neutral). However, most water we encounter
does contain dissolved charged particles, so it will be more conductive than pure water. Many of the
problems that occur when touching electrical devices with wet hands result from the ever-present salt that is
left on our skin through perspiration and it dissolves in the water to make it more conductive.
81. Electrical conductivity is
A. one of the most important properties of metals
B. one of the key properties of most solid materials
C. impossible for any substance when it is dissolved in water
D. completely impossible for silicon
82. According to the passage, a metal can conduct electricity due to
A. the absence of free electrons B. its atoms with a positive charge
C. the way its atoms bond together D. the loss of one electron in the core of its atoms
83. The word “outermost” in paragraph 1 mostly means
A. the lightest. B. nearest to the inside.
C. furthest from the inside. D. the heaviest.
84. The atoms of a metal can bond together because
A. the lost electrons cannot move freely in the sea of electrons
B. electrons can flow in a single direction
C. they lose all of electrons
D. negatively charged electrons attract positive tons
85. Salt in its solid state is not able to conduct electricity because
A. it has free electrons B. Its charged torts can flow easily
C. it cannot create any charge ions D. it charged tons are not free to move
86. The word “they” in paragraph 3 refers to
A. charged ions B. electric currents
C. charged particles D. electrical insulators
87. Water is a poor conductor because it contains

114
A. no positive or negative electric charge B. only a small amount of fully charged particles
C. only a positive electric charge D. only a negative electric charge
88. We can have problems when touching electrical devices with wet hands because
A. the eater itself is a good conductor of electricity
B. the water dissolves the salt on our skin and becomes more conducttve
C. the water contains too many neutral molecules
D. the water containing no charged particles makes it more conductive
89. Which of the following is NOT true according to the passage?
A. Pure water is much more conductive than most water we encounter every day.
B. Graphite is a common solid substance that can conduct electricity.
C. Salt can conduct electricity when it is molten or dissolved.
D. Some materials are more conductive than others.
90. Which of the following could best serve as the title of the passage?
A. Electrical Energy B. Electrical Devices
C. Electrical Insulators D. Electrical Conductivity
PART 2: WRITING (2 pts)
Task 1: SENTENCE TRANSFORMATION (1.0 pt - 0.2/ each) gewrite the following sentences in such
a way that their meanings remain unchanged, using the words given.
91. He is very sorry he didn’t come to the meeting yesterday.
He apologized ………………………………………………………………………………………….
92. We couldn’t relax until all the guests had gone home.
Only ………………………………………………………………………………………………
93. House prices have risen sharply this year.
There has……………………………………………………………………………………………
94. The police said Jim had stolen the money. accused
The police ………………………………………………………………………………………
95. Because of his illness he could not work effectively. impossible
His illness ………………………………………………………………………………………
Part 2:
From 120 to 150 words, write a paragraph about why is it important to protect our environment? What
should we do to fulfill this task?
………………………………………………………………………………………………………
………………………………………………………………………………………………………
………………………………………………………………………………………………………
………………………………………………………………………………………………………
………………………………………………………………………………………………………

ĐỀ SỐ 25
(Đề thi vào lớp 10 THPT Chuyên Bắc Giang)
Thời gian làm bài: 150 phút
LƯU Ý:
- Đề thi gồm 08 trang, thí sinh làm bài trực tiếp vào đề thi
- Thí sinh không được sử dụng bất cứ tài liệu nào.
I. LISTENING - TRACK 7 (20 pts)
Part 1: Listen to the recording and circle the appropriate answer A, B or C to questions 1-4. You will
hear the recording ONCE. (4 pts)
1. The fair will take place at the…………..

115
A. fairgrounds B. park C.school
2. The fair will begin on Friday…………...
A. morning B. afternoon C.evening
3. The fair will begin with a…………...
A. parade B. dance performance C. speech by the major
4. There will be free admission on…………...
A. Friday B. Saturday C. Sunday
Your answers: 1.………….. 2.………….. 3.………….. 4…………..

Part 2: Listen to the recording and complete the table. Write NO MORE THAN TWO WORDS
AND/OR A NUMBER (except phone number) for each answer to questions from 1 to 10. You will hear
the recording ONCE. (10 pts)
Example: Clear-Point Telephone Company
Customer Order Form
Order taken by: Ms. Jones
Name: Harold (1)………………
Address: (2)………………Fulton Avenue, apartment 12
Type of service: (3)………………
Employer: Wrightsville Medical Group
Occupation: (4)………………
Work phone: (5)………………
Time at current job: (6)………………
Special services: (7)………………
(8)………………
Installation Day (9)………………
scheduled for: Time of the day (10)………………

Part 3: Listen to the recording and then indicate whether the statements are True (T) or False (F). You
will hear the recording TWICE. (6 pts)
1………………Rachel came to visit Buenos Aires in 1998.
2………………Tango is a dance that came from Argentina.
3………………The population of Buenos Aires started to increase at the beginning of the twentieth century.
4………………Immigrants to Buenos Aires came from Asia and from Europe.
5………………Rachel decided to learn tango in order to understand Argentina.
6………………Rachel’s teacher of tango was Argentinian.
THE END OF THE LISTENING
II. Choose the correct answer among A, B, C or D to complete the sentences. (12 pts)
1.I had to get up early,…………..I would miss the train.
A. otherwise B. if not C. so that D. but
2. My parents are going to take three days…………..next month to help my brother move his house.
A. at B. over B. over D. out
3. Not a good movie,…………..?
A. did it B. didn’t it C. was it D. wasn’t it
4. Hoi An is…………..for its old, small and tile-roofed houses.
A. well-done B. well-dressed C. well-organized D. well-known
5. John: “Could you tell me how to get to the nearest post office?” Peter: “…………..”
A. Sorry for this inconvenience B. I have no clue
C. Not at all D. Sorry, I’m a new comer here

116
6. Have you ever read anything…………..Earnest Hengmingway?
A. by B. of C. from D. for
7. Either John or his brothers…………..the money.
A. has stolen B. have stolen C. has been stolen D. have been stolen
8. The villagers strongly recommend that a new school…………..immediately.
A. must be built B. is going to be built C. be built D. will be built
9. “.…………..” - “Thank you. We are proud of him.”
A. Your kid is naughty.
B. Can we ask your child to take a photo?
C. Your child is just adorable!
D. I can give your kid a lift to school.
10. I'm sure you'll have no…………..the exam.
A. difficulty to pass B. difficulties to pass
C. difficulties passing D. difficulty passing
11. Wood that has been specially treated is…………..regular wood.
A. as water resistant much more than B. water resistant much more than
C. more than water resistant D. much more water resistant than
12. In 1938, many people listening to the radio heard a report…………..had landed in New Jersey.
A. how beings from Mars B. being from Mars
C. that being from Mars D. that beings from Mars
Your answers:
1………… 2………… 3………… 4………… 5………… 6…………
7………… 8………… 9………… 10………… 11………… 12…………
III. Give the correct form of the words in the brackets to complete the passage - below. (5 pts)
Sports in Japan are a (1. signify)…………….part of Japanese culture. Both traditional sports such as sumo
and martial arts, and Western imports like baseball and association football are popular with both participants
and spectators. Sumo wrestling is considered Japan’s national sport. Baseball was introduced to the country
by (2. visit) …………….Americans in the 19th century. The Nippon Professional Baseball League is Japan's
largest professional sports competition in terms of television and spectators. Martial arts such as judo, karate,
and modern kendo are also widely practiced and enjoyed by spectators in the country. Association football
has gained wide popularity since the founding of the Japan Professional Football League in 1992. Other
popular sports include figure skating, golf, and racing, especially auto racing. There are opportunities to play
(3. vary) …………….sports for all ages, and school plays an important role in the community.
Kindergarten and lower elementary school students can play in a sports club that can be joined (4. private)
…………….for a moderate fee. Most martial arts can be started as young as 4 or 5 years old. When a student
starts 5th grade, the school offers free after-school activities for its students to participate. Middle and high
schools also encourage their students to join school sports clubs. (5. province) …………... and nationwide
contests and tournaments are held every winter and summer for all sports.
Your answers:
1…………….. 4……………..
2…………….. 5……………..
3……………..
IV. Rearrange the statements to make a complete conversation. The first one has been done as an
example. (5 pts)
A. Mum?
B. Oh, Ryan! It’s a really expensive course. I’m not sure we can afford for you to take it again. Things are
difficult enough as they are.
C. You failed it! But wasn’t that an important one?
D. Yes?

117
E. Yeah, I'm really sorry. Well, I’ll probably have to retake the whole course.
F. Oh, yes, I remember. Apart from going out until four in the morning, you mean?
G. You know that Economics exam I had last week?
H. I know, Mum. I’m sorry. I’ll talk to the teacher again and see if I can retake it.
I. The one you didn’t revise for?
J. Well, I have to relax a little, even if I’m revising. Anyway, I didn’t pass it.
K. I did revise for it. I was working on it all weekend. Don’t you remember?
Your answers: 0. A
1…………….. 2……………... 3……………... 4……………... 5……………...
6…………….. 7…………….. 8…………….. 9…………….. 10……………..
V. The passage below contains 10 errors. UNDERLINE and CORRECT them. Write your answers in the blanks provided.
(10 pts)
Line
1 A new ruling which came into effective last week requires some homeowners to purchase new
2 smoke alarms. The ordinance state that there should be a fire alarm installing in every bedroom of
3 the house, and these alarms must be complied with certain safety standards. Some local residents
4 are displease at these new regulations. Gwen Ellis of McKinley says that to meet with the new
5 regulations, she has to buy five new alarms to replace the one she installed just six months ago.
6 With fire alarms cost up to $20 a piece, this is an unwelcome ruling to many people. Fortunately,
7 the McKinley Fire Department is offering grants for homeowners and will provide and install
8 new fire alarms free of charges. If you have small children or are older than 65 years of age, you
9 may be eligible for these. The McKinley Fire Department has 5,000 smoking alarms to give
10 away. To inquire about obtaining a free alarm, or to find out whether you quality, call 692-569-
11 0372.

Your answer:
Line Error Correction
Line……………
Line……………
Line……………
Line……………
Line……………
Line……………
Line……………
Line……………
Line……………
Line……………

VI. Read the passage and do the tasks below. (13 pts)
Nature or Nurture?
A. A few years ago, in one of the most fascinating and disturbing experiments in behavioural psychology,
Stanley Milgram of Yale University tested 40 subjects from all walks of life for their willingness to obey
instructions given by a 'leader' in a situation in which the subjects might feel a personal distaste for the
actions they were called upon to perform, specifically, Milgram told each volunteer 'teacher-subject' that the
experiment was in the noble cause of education, and was designed to test whether or not punishing pupils for
their mistakes would have a positive effect on the pupils' ability to learn.
B. Milgram's experimental set-up involved placing the teacher-subject before a panel of thirty switches with
labels ranging from '15 volts of electricity (slight shock)' to '450 volts (danger - severe shock)' in steps of 15

118
volts each. The teacher-subject was told that whenever the pupil gave the wrong answer to a question, a
shock was to be administered, beginning at the lowest level and increasing in severity with each successive
wrong answer. The supposed 'pupil' was in reality an actor hired by Milgram to simulate receiving the shocks
by emitting a spectrum of groans, screams and writhings together with an assortment of statements and
expletives denouncing both the experiment and the experimenter. Milgram told the teacher-subject to ignore
the reactions of the pupil, and to administer whatever level of shock was called for, as per the rule governing
the experimental situation of the moment.
C. As the experiment unfolded, the pupil would deliberately give the wrong answers to questions posed by
the teacher, thereby bringing on various electrical punishments, even up to the danger level of 300 volts and
beyond. Many of the. teacher-subjects balked at administering the higher levels of punishment, and turned to
Milgram with questioning looks and/or complaints about continuing the experiment. In these situations,
Milgram calmly explained that the teacher-subject was to ignore the pupil's cries for mercy and carry on with
the experiment. If the subject was still reluctant to proceed, Milgram said that it was important for the sake of
the experiment that the procedure be followed through to the end. His final argument was, 'You have no other
choice. You must go on.' What Milgram was trying to discover was the number of teacher-subjects who
would be willing to administer the highest levels of shock, even in the face of strong personal and moral
revulsion against the rules and conditions of the experiment.
D. Prior to carrying out the experiment, Milgram explained his idea to a group of 39 psychiatrists and asked
them to predict the average percentage of people in an ordinary population who would be willing to
administer the highest shock level of 450 volts. The overwhelming consensus was that virtually all the
teacher-subjects would refuse to obey the experimenter. The psychiatrists felt that 'most subjects would not
go beyond 150 volts' and they further anticipated that only four per cent would go up to 300 volts.
Furthermore, they thought that only a lunatic fringe of about one in 1,000 would give the highest shock of
450 volts.
E. What were the actual results? Well, over 60 per cent of the teacher-subjects continued to obey Milgram up
to the 450-volt limit! In repetitions of the experiment in other countries, the percentage of obedient teacher-
subjects was even higher, reaching 85 per cent in one country. How can we possibly account for this vast
discrepancy between what calm, rational, knowledgeable people predict in the comfort of their study and
what pressured, flustered, but cooperative teachers actually do in the laboratory of real life?
F. One's first inclination might be to argue that there must be some sort of built-in animal aggression instinct
that was activated by the experiment, and that Milgram's teacher-subjects were just following a genetic need
to discharge this pent-up primal urge onto the pupil by administering the electrical shock. A modern hard-
core sociobiologist might even go so far as to claim that this aggressive instinct evolved as an advantageous
trait, having been of survival value to our ancestors in their struggle against the hardships of life on the plains
and in the caves, ultimately finding its way into our genetic make-up as a remnant of our ancient animal
ways.
G. An alternative to this notion of genetic programming is to see the teachersubjects' actions as a result of the
social environment under which the experiment was carried out. As Milgram himself pointed out, 'Most
subjects in the experiment see their behaviour in a larger context that is benevolent and useful to society - the
pursuit of scientific truth. The psychological laboratory has a strong claim to legitimacy and evokes trust and
confidence in those who perform there. An action such as shocking a victim, which in isolation appears evil,
acquires a completely different meaning when placed in this setting.'
H. Thus, in this explanation the subject merges his unique personality and personal and moral code with that
of larger institutional structures, surrendering individual properties like loyalty, self-sacrifice and discipline
to the service of malevolent systems of authority.
I. Here we have two radically different explanations for why so many teachersubjects were willing to forgo
their sense of personal responsibility for the sake of an institutional authority figure. The problem for
biologists, psychologists and anthropologists is to sort out which of these two polar explanations is more
plausible. This, in essence, is the problem of modern sociobiology - to discover the degree to which hard-

119
wired genetic programming dictates, or at least strongly biases, the interaction of animals and humans with
their environment, that is, their behaviour. Put another way, sociobiology is concerned with elucidating the
biological basis of all behaviour.
Questions 1-6: This Reading Passage has nine paragraphs, A-I. Which paragraph contains the
following information?
1…………. a biological explanation of the teacher-subjects' behaviour.
2………….the explanation Milgram gave the teacher-subjects for the experiment.
3………….the identity of the pupils.
4………….the expected statistical outcome.
5………….the general aim of sociobiological study.
6………….the way Milgram persuaded the teacher-subjects to continue.
Questions 7-9: Choose the correct letter, A, B, C or D.
7. The teacher-subjects were told that they were testing whether……………
A. a 450-VO It shock was dangerous. B. punishment helps learning.
C. the pupils were honest. D. they were suited to teaching.
8. The teacher-subjects were instructed to……………
A. stop when a pupil asked them to.
B. denounce pupils who made mistakes.
C. reduce the shock level after a correct answer.
D. give punishment according to a rule.
9. Before the experiment took place the psychiatrists……………
A. believed that a shock of 150 volts was too dangerous.
B. failed to agree on how the teacher-subjects would respond to instructions.
C. underestimated the teacher-subjects' willingness to comply with experimental procedure.
D. thought that many of the teacher-subjects would administer a shock of 450 volts.
Your answers: 7.…………… 8……………. 9…………….
Questions 10-13: Decide whether the following statements are True (T), False (F) or Not given (NG)
according to the passage.
10. Several of the subjects were psychology students at Yale University.
11. Some people may believe that the teacher-subjects' behaviour could be explained as a positive survival
mechanism.
12. In a sociological explanation, personal values are more powerful than authority.
13. Milgram's experiment solves an important question in sociobiology.
Your answers: 10.…………… 11.…………… 12.…………… 13.……………
VII. The list of New Book Releases on the following passage has nine book descriptions from A to I.
Choose the correct title for each book from the list of book titles below. One has been done as an
example. Put a CROSS (X) to the titles that don’t match any books. (10 pts)

New Book Releases


A This book describes the creativity of Aboriginal people living in the driest parts of Australia. Stunning
reproductions of paintings, beautiful photography and informative text.
B Pocket-sized maps and illustrations with detailed information on the nesting sites and migration patterns of
Australia. This is a classic booklet suitable for both beginner and expert.
C Packed full of information for the avid hiker, this book is a must. Photographs, maps and practical advice
will guide your journeys on foot through the forests of the southern continent.
D More than-an atlas - this book contains maps, photographs and an abundance of information on the land
and climate of countries from around the globe.
E Australia's premier mountain biking guidebook - taking you through a host of national parks and state
forests.

120
F Here's the A-Z of Australian native animals - take an in-depth look at their lives and characteristics,
through fantastic photographs and informative text.
G Graphic artists have worked with researchers and scientists to illustrate how these prehistoric animals lived
and died on the Australian continent.
H A definitive handbook on outdoor safety - with a specific focus on equipment, nutrition, first aid, special
clothing and bush skills.
I Detailed guides to 15 scenic car tours that will take you onto fascinating wilderness tracks and along routes that you
could otherwise have missed.
List of Book Titles Answers
0. Field Guide to Native Birds of Australia Book….B….
1. The Bush on Two Wheels: 100 Top Rides Book……………..
2. Bush Foods of Australian Aborigines Book……………..
3. A Pictorial History of the Dinosaur in Australia Book……………..
4. Bushwalking in Australia Book……………..
5. World Geographica Book……………..
6. Driving Adventures for 4-wheel-drive Vehicles Book……………..
7. Survival Techniques in the Wild Book……………..
8. Encyclopedia of Australian Wildlife Book……………..
9. Guide to the Art of the Australian Desert Book……………..
10. Field Guide to Animals of the World Book……………..

VII. Choose the correct answer among A, B, C or D that best fits each of the blank spaces. (5 pts)
The popular image of student life is of young people with few responsibilities enjoying themselves
and (1)…………….very little work. This is often not true. Many older people now study at college or
university, sometimes (2) …………….a part-time basis while having a job and looking after a family. These
students are often (3)……………motivated and work very hard.
Younger students are often thought to be lazy and careless about money but this (4) …………….is
changing. In Britain reduced government support for higher education means that students can no longer rely
on having their expenses (5) …………….for them. Formerly, students received a grant towards their living
expenses. Now most can only get a loan (6) …………….has to be paid back. Since 1999 they have paid over
£1 000 towards tuition (7) …………….and this amount will increase up to a maximum of £3 000. In the US
students already (8) …………….pay for tuition and room and board. Many get a financial aid package which
may (9) …………….grants, scholarships and loans. The fear of having large debts places (10) …………….
pressure on students and many take part-time jobs during the term and work full-time in the vacations.
1. A. producing B. carrying C. doing D. making
2. A. for B. with C. on D. at
3. A. highly B. mainly C. absolutely D. adequately
4. A. position B. state C. situation D.
condition
5. A. paying B. paid C. pay D. to pay
6. A. whether B. what C. which D. who
7. A. money B. fees C. allowances D. charge
8. A. had better B. should C. may D. have to
9. A. include B. consist C. compose D. belong
10. A. large B. generous C. considerate D. considerable
Your answers:
1……………. 2.……………. 3.……………. 4.……………. 5.…………….
6.……………. 7.……………. 8.……………. 9.……………. 10.…………….

121
IX. Rewrite the sentences, beginning with the words given so that the meanings stay the same as the
first ones. (5 pts)
1. Though man is wise, he may make mistakes.
→However………………………………………………………………………………………………..
2. It was her lack of confidence that surprised me.
→What I found……………………………………………………………………………………………
3. It was wrong of you to allow a four-year-old child to walk home alone.
→You should not………………………………………………………………………………………….
4. The mother is proud of her son’s contribution to the play last night.
→The mother is proud of what……………………………………………………………………………
5. Only two out of the five rooms we have booked have air conditioning.
→We have booked five rooms, only ………………………………………………………………………
X. Complete the second sentence so that it has similar meaning to the first sentence, using the word
given. DO NOT change the word given. You must use between THREE and FIVE words, including the
word given. (5 pts)
1. Jenny didn’t feel like going to the party. (mood)
→Jenny……………………………………………………………………………to go to the party.
2. I felt that it had been a big mistake to give up that job. (regretted)
→I ……………………………………………………………………………………….up that job.
3. He joined the army as soon as he had finished school. (sooner)
→No………………………………………………………………………school than he joined the army.
4. He won’t let anyone else touch his records. (objects)
→ He………………………………………………………………………………his records.
5. The number of people out of work has been going down little by little. (gradual)
→ There has been a ………………………………………………… the number of people out of work.
XI. Write a paragraph. (10 pts)
Write a paragraph from 160-180 words, beginning with "One day, my best friend did not go to school."
……………………………………………………………………………………………………………
……………………………………………………………………………………………………………
……………………………………………………………………………………………………………
……………………………………………………………………………………………………………
……………………………………………………………………………………………………………
……………………………………………………………………………………………………………
……………………………………………………………………………………………………………
……………………………………………………………………………………………………………

122
Phần 2: ĐÁP ÁN VÀ HƯỚNG DẢN GIẢI
A. NĂM HỌC 2014 - 2015
ĐỀ SỐ 01
A. PHONETICS
I. (2.0 points): 1.0 point for each correct answer
1. B 2.D
II. (3.0 points): 1.0 point for each correct answer
3. A 4. A 5.C
B: GRAMMAR - VOCABULARY - LANGUAGE FUNCTIONS
I. (20 points): 1.0 point for each correct answer
6.C 7.C 8. B 9. A
10.D 11.B 12.D 13.C
14.C 15.B 16. D 17. D
18.A 19.B 20. B 21.D
22.A 23.B 24. B 25. D
II. (5.0 points): 1.0 point for each correct answer
26. qualified 27. beautify 28. miserably
29. enrichment 30. awake
III. (5.0 points): 1.0 point for each correct answer
31.D 32. B 33. B 34. A 35. C
D. READING
I. (5.0 points): 1.0 point for each correct answer
36. D 37. E 38. F 39. A 40. B
II. (10 points): 1.0 point for each correct answer
41.B 42. A 43. D 44. B
45. A 46. C 47. D 48. A
49. B 50. C
III. (10 points): 2.0 points for each correct answer
51. A 52. B 53. A 54. D 55. D
E. WRITING
I (5.0 points): 1.0 point for each correct answer
56. Nam is not used to / accustomed to / familiar with driving on the left.
57. The mother accused Bob of having damaged / damaging her car.
58. However short the journey is, passengers always get something to eat on this airline.
59. The cake was not fresh enough for the boy to eat.
60. Although it snowed heavily, the explorers managed to walk to the village.
or, Although the snow was heavy, the explorers managed to walk to the village.
II. (10 points):
61. Everyone is becoming / has become aware (of the fact) that the environment is a serious issue (1 point).
62. However, we do not do / have not done much to deal with the / this problem because we seem to wait for
the government to take actions (2 points).
63. In my opinion, individuals can do many / a lot of things to help solve the problem (1 point).
64. To begin with, we can be (more) responsible in the way we dispose of waste (1 point).
65. We should not throw rubbish into lakes and rivers (1 point).

123
66. Moreover, we also need to save the water we use because fresh water is running out in many parts of the
world (2 points).
67. Finally, the air we breathe is polluted with exhaust fumes from cars in the cities (1 point).
68. I think, if we use public transport more, we will I can reduce air pollution (1 point).
III. (10 points):
1. Form:
- Correct form of an essay (1.0 point)
2. Task fulfillment:
- Task completed with relevant information (2.0 points)
- Well-organized (2.0 points)
3. Language:
- Accurate grammar (2.0 points)
- Appropriate vocabulary (2.0 points)
- Correct spelling and punctuation (1.0 point).

ĐỀ SỐ 02
PART A. PHONETICS (1.0 point)
I. Pronunciation. (0.5p) →0.1 point for each correct answer
1B 2C 3B 4C 5C
II. Stress. (0.5p) → 0.1 point for each correct answer
1C 2D 3D 4C 5C
PART B. GRAMMAR AND VOCABULARY (3.0 points)
I. MCQ. (1.0 p) → 0.1 point for each correct answer
1B 2B 3B 4D 5B 6C 7A 8A 9A 10B
II. Word form. (1.0p) → 0.1 point for each correct answer
1. variety 2. reduction 3. ambitious 4. celebrity 5. geographical
6. overconfident 7. unpredictable 8. unexpectedly
9.critical 10. politicians
III. Phrasal verbs. (1.0 p) → 0.1 point for each correct answer
1. call in 2. drawn up 3. look down on 4. slow down 5. made out
6. try it on 7. ring off 8. put it aside 9. sat up 10. settled on
PART C. READING (3.0 points)
I. Reading comprehension. (1.2 p) → 0.2 point for each correct answer
1. (It/ The training camp was held) at the Girls’ Brigade Campsite (at Sembawang).
2. (It means) fear of heights.
3. (It is) fastened/ The word “fastened”/ Fastened does/has the same meaning as “secured tightly”.
4. Because she showed/had the potential/ability to become/be a good leader.
5. She didn’t want the Girls’ Brigade to think that she was not a good leader (because/and she had no
courage).
6. The sentence “Then I heard cheers and claps from below”.
(Học sinh có thể diễn đạt theo cách khác nhưng phải đúng, đủ ý và đúng ngữ pháp chính tả mới cho điểm tối
đa)
II. Matching. (1.0 p) → 0.2 point for each correct answer
1C 2E 3D 4F 5A
III. Gap fill. (0.8 p) → 0.1 point for each correct answer
1. contact/touch 2. passengers 3. to 4. based 5. for
6. concentrated/focused 7. black 8. were

124
PART D. WRITING (3.0 points)
I. Sentence transformation. (2.0 p) → 0.2 point for each correct answer
1. more amusing novel than this/ novel as amusing as this
2. you a ring/call the moment
3. was being badly run by/ was being run badly by
4. you had followed my instructions
5. keep/ get in touch with him
6. to avoid upsetting
7. made arrangements to go
8. as if/though nothing matters
9. was meant to set/ move/have set/ have moved
10. the problem at length
11. Topic writing. (1.0 p)
Marking scheme
The impression mark is based on the following scheme:
- Format. (0.1 point): the writing has 3 parts: the Introduction, the body (01 or more para) and the conclusion.
- Content. (0.5 point): a provision of main ideas and details as appropriate to support the ideas.
- Language. (0.2 point): a variety of vocabulary and structures appropriate to the level of secondary gifted
students
Presentation. (0.2 point): coherence, cohesion, and style appropriate to the level of secondary gifted students.

ĐỀ SỐ 03
SECTION A: USE OF ENGLISH
I. Choose the best answer from A, B, C or D
1.A 2. B 3. D 4. A 5. C 6. A 7. D 8. D 9. B 10.C
II. Choose the best answer from A, B, C or D to complete the passage
1.B 2. A 3.D 4. B 5. D 6.C 7. A 8. B 9. D 10. B
III. Fill in each numbered space with a suitable word.
1. both 6. sure
2. something 7. careful / sure
3. advance 8. with
4. Prepare/ Bring 9. may/ might/ could
5. where 10. animal
IV. Complete the passage below using the correct form of the words in brackets
1. theft 6. advice
2. painful 7. insurance
3. owner 8. sensible
4. valuables 9. suggestions
5. accidentally 10. reliable
SECTION B: READING
I. Read the passage below and choose the best answer.
1.C 2. D 3. B 4. A 5.C 6. D 7.B
II. Read the following magazine article about ways of
1.D 2.A 3.C 4.B 5.A 6.B 7.C 8.B 9.A 10.D
SECTION C: WRITING
I. Complete the second sentence so that it has the same meaning as the first one...
1………………under the weather today.

125
2………………is some furniture ………………/………………are some pieces of furniture
3………………very traditionally brought up………………
4………………shouldn’t have allowed………………
5………………you work, the more successful………………
II. Essay.
Length: 25 %
Grammar and vocabulary: 25 %
ideas: 25 %
coherence: 25 %

ĐỀ SỐ 04
I. READING
PART 1(1 point, 0.2 each)
1. B 2. C 3. B 4.C 5. A
PART 2(1 point, 0.25 each)
1. C 2. A 3. D 4. E
II. USE OF LANGUAGE
PART 1 (0.5 point, 0.05 each)
1. A 2. B 3.C 4. D 5.B
6. C 7. A 8. D 9. A 10. D
PART 2 (1 point, 0.1 each)
1.B 2.C 3.B 4.A 5.D
6.D 7.C 8.C 9.D 10.A
PART 3 (1 point, 0.1 each)
1. born 2. His
3. music 4. by
5. an 6. of
17. in 8. than
9. as 10. most
PART 4 (1 point, 0.1 each)
1. happily 2. expensive
3. wealthy 4. savings
5. departure 6. luxurious
7.disappointment 8. boredom
9. freedom 10. disastrous
PART 5 (0.5 point, 0.05 each)
1. There was no point in asking the manager for the day off.
2. We are going to have our house painted by a local firm.
3. George made as many mistakes as Peter.
4. Mr Smith accused Tom of breaking/ having broken the window.
5. I would never have / would have never/ never would have learnt to drive if you hadn't taught me.
6. Both five and seven are odd numbers.
7. It takes three hours to climb to the top of the hill.
8. I think they should abolish/ do away with this law.
9. He has never seen a skyscraper before.
10. The sun, (which is) one of millions of stars in the universe, provides us with heat and light.
III. WRITING (2 points)

126
1. Task respond: How well the student answers the question.
- Physical appearance (description + comparison) (0.25 point)
- Personality (description + comparison) (0.25 point)
- Hobbies (description + comparison) (0.25 point)
- Introducing the topic + Summarizing/ giving suggestions/ opinions (0.25 point)
2. Coherence and cohesion: How well the student’s writing links together. (0.4 point)
3. Lexical resources: How good the student’s vocabulary is. (0.3 point)
4. Grammatical range and accuracy: How good the student’s grammar is. (0.3 point)

B. NĂM 2015 - 2016


ĐỀ SỐ 05
PART 1: GRAMMAR AND VOCABULARY (5 POINTS)
I. Supply the correct tense of the verbs in parentheses. (2 points) (0.2pt/each)
1. had spent 6. had told
2. are moving/are going to move 7. woke
3. announced 8. was dripping
4. have been trying 9. have spent
5. was mixing 10. haven’t / have not found
II. Fill in each blank with an appropriate preposition. (2 points) (0.2pt/each)
1.of 2.at 3.for 4.on 5.in
6.for 7.to 8.to 9.of 10.on
III. Use the correct form of the words given in brackets. (1 point) (0.2pt/each)
1. unpolluted 4. qualifications
2. anxiety 5. excitedly
3. prove
PART 2: READING COMPREHENSION (6 POINTS)
I. Read the following passage and choose the best answer to each question. (2 points) (0.4pt/each)
1C 2A 3D 4D 5B
II. Read the text below and fill in each of the blanks with ONE suitable word.(4 points) (0.4pt/each)
1. director 2. played 3. they 4. for 5. found
6. Hear 7. reason 8. If 9. lots 10. then
PART 3: WRITING (5 POINTS)
I. Sentence transformation
Complete the second sentence so that it has a similar meaning to the first sentence, using the word given and
do not change it. Students must use between two and five words, including the word given. (1 point)
(0.2pt/each)
1. were you, I wouldn’t
2. you mind not using
3. as soon as we arrived
4. are having the van repaired
5. have not changed since
II. Writing
Describe dinner time in your family. (4 points)
Answers vary
- 4 points: All content elements covered appropriately.
Message clearly communicated to reader.
- 3 points: All content elements adequately dealt with.

127
Message communicated successfully, on the whole.
- 2 points: All content elements attempted.
Message requires some effort by the reader.
- 1 point: Little relevant content and/or message requires excessive effort by reader
- 0 point: Totally irrelevant or totally incomprehensible or too short (under 10 words)

ĐỀ SỐ 06
Question 1: A. laughter Question 2: C. exacerbate Question 3: C. picnics
Question 4: D. committee Question 5: B. conversation Question 6: C. would travel
Question 7: B. Might Question 8: B. thick and thin Question 9: D. Having been awarded
Question 10: A. neatly Question 11: A. tightly Question 12: C. Even
Question 13: A. tunnel Question 14: C. be seen to be taking Question 15: D. sacrificed
Question 16: A. stroke Question 17: D. distinguish Question 18: B. quality
Question 19: A. worldclass Question 20: C. spiritual Question 21: B. accessible
Question 22: B. Larger classes are expected to lead to poorer results but they do not.
Question 23: A. do better on standardised tests
Question 24: C. more effective lesson planning
Question 25: A. Class size does not affect student performance.
Question 26: D. is more common in Korea than in Britain
Question 27: D. [1]
Question 28: A. don’t allow themselves much time to relax and have fun
Question 29: A. good discipline and a hard work ethic
Question 30: C. The Asian system is obviously better.
Question 31: A. means [by no means = not at all]
Question 32: C. little
Question 33: B. came
Question 34: B. consisted
Question 36: B. valuable
Question 35: D. off
Question 37: D. transformation
Question 38: B. nonetheless
Question 39: c. expanding
Question 40: A. greater
41. teaching 42. learning 43. liked 44. doing 45. depended
46. learn 47. will never forget 48. have ever had 49. had not taken
50. would not have been
51. MEANINGFUL 52. SINGULAR 53. SCHOLARLY 54. REPLACE 55. LITERARY
56. OBSESSION 57. RESPONSE 58. MATERIALISTIC
59. SATISFACTORY 60. SPECTATOR
61. ready 62. and 63. Children 64. up 65. lay
66. save 67. aware 68. opportunities 69. Who 70. Themselves
Question 71. put Carla up Question 72. coming up Question 73. set up
Question 74. take it upQuestion 75. make up for Question
Question 76. calls for 77. coming off Question 78.dropped out of it
Question 79. will go up Question 80. grew out of
Question 81. He is an authority on [the history of] primitive life.
Question 82. The story about her achievement was beyond belief.

128
Question 83. The interference on the radio made it impossible[for me] to make sense of the message.
Question 84. They arrived at the station in the nick of time.
Question 85. David congratulated her on a good venue for the party.
Question 86. But forJack 's being so affluent, she would not be dating with him.
Question 87. They probably forgot about the extra class.
Question 88. I’m looking for a fairly long, green and woolen scarf
Question 89. Under no circumstances should you open this door when the building is open to the public.
Question 90. Death is more likely to be caused by a bee sting than by a snake bite these days.
Write a paragraph of about 150 words about the benefits of studying at a gifted school.
A Suggested Writing Sample
A gifted school is really good environment to study and culivate the mind. We all can see that studying in a
gifted school bring us many benefits.
Firstly, if you ’re a student at a gifted school, you will have good conditions to develop abilities in your
favorite subjects, help you to build dream and orient your future career.
Secondly, most of the teacher in the gifted school are good and enthusiatic. They always create comfortable
atmosphere in studying for students to receive knowledge easily.
Besides, there are many valuable scholarships for good students, especially, for students that have difficult
circumstances.
Finally, the gifted school always has many different facilities for students to relax after studying hard.
There are many advantages of studying in a gifted school. Therefore, we need to make the best use of these
benefits to have the most best rusults [143 words]

ĐỀ SỐ 07
(Tổng điểm: 100 điểm - phần nghe 20 điểm)
I. (5 pts) (Mỗi câu đúng 0,5 điểm)
1. B. Would you mind making some coffee 6. D. to whom I look up
2. A. seat 7. C. terms
3. C. because 8. A. in for
4. D. made 9. D. small red French
5. C. taken 10. B. went off
II. (5 pts) (Mỗi động từ chia đúng 0,5 điểm)
1. would stay 6. should have informed
2. is always forgetting 7. known
3. has just arrived 8. has been named
4. did Tom visit 9. (should) be repaired
5. complaining/ to help
III. (5 pts) (Mỗi giới từ đúng 0,5 điểm)
1. of / about 4. in / for
2. over 5. into / of
3. up / with 6. down
IV. (5 pts) (Mỗi câu đúng 0,5 điểm)
1. preparations 6. massive
2. unfavorable 7. outnumber
3. surprisingly/ weaken 8. unpolluted
4. independent 9. impressionable
5. prosperity
V. (5 pts) (Mỗi câu đúng 0,5 điểm)
l .B 2. D 3.H 4. G 5. J

129
6. C 7. E 8. A 9.I 10. F
VI. (10 pts) (Mỗi lỗi tìm được được 0,5 điểm, sửa đúng được 0,5 điểm. Các lỗi học sinh tìm được có thể
không theo thứ tự dưới đây)
1. Line 2: much → many
2. Line 3: taught →were taught
3. Line 4: are → is
4. Line 5: natural → naturally
5. Line 7: Person → People
6. Line 8: for → with
7. Line 9: with → without
8. Line 10: putting → to put
9. Line 11: where → which
10. Line 12: suitable → unsuitable
VII. (10 pts) (Mỗi từ điền đúng 1 điểm)
1. killed 6. destroyed/ ruined
2. lasted 7. first
3. of 8. the
4. natural 9. than
5. homeless 10. earthquakes/ quakes
VIII. (10 pts) (Mỗi câu đúng 1 điểm)
1. C. sure 6. B. other
2. A. ancient 7. B. keen
3. A. changed 8. C. sight
4. D. whereas 9. D. agree
5. A. associated 10. C. easily
IX. (5 pts) (Mỗi câu đúng 1 điểm)
l. F 2. A 3.C 4. E 5. D
X. (5 pts) (Mỗi câu đúng 1 điểm)
1. The news (which/ that)/ to which I listened/I listened /to last night was very informative.
2. Becauseit/the weather is (very) hot, a lot of children and old people have to go to hospitals.
3. His brother plays tennis professionally, doesn’t he?
4. Hardly a day goes by/passes without his listening to the radio.
Or: Hardly a day goes by/passes when he doesn’t listen to the radio.
5. Only when they had their second son/ their second son was born did they decide to move to a bigger
house.
XI. (5 pts) (Mỗi câu đúng 1 điểm)
1. The firemen succeeded in putting the fireout after five hours.
2. It is a five-hour drive to the nearest hotel.
3. Have you got used to his teaching style yet?
4. David made a good impression on his new boss by settling down to work.
5. “Is there any chance of Peter changing his mind?” Rob asked.
XII.(10pts)
Các tiêu chí chấm bài viết đoạn văn.
Tiêu chí Mô tả chi tiết các tiêu chí Điểm
Cấu trúc Đúng cấu trúc của đoạn văn, phù họp với chủ đề: Có câu chủ đề (Topic 3
sentence); các ý minh họa (Supporting sentences/ ideas, từ 2 supporting
sentences/ ideas trở lên); và câu kết luận (Concluding sentence).
Nội dung - Viết đúng chủ đề, các ý minh họa rõ ràng, có tính thuyết phục. 2
- Viết logic, hợp lí; mạch lạc, rõ ràng 2

130
Ngôn ngữ - Viết đúng cấu trúc ngữ pháp; sử dụng các liên từ hợp lý để liên kết các ý 2
trong đoạn
- Sử dụng từ vựng phù hợp, phong phú, hạn chế lặp lại từ vựng
Trình bày - Viết đủ số từ theo quy định 1
- Không xuống dòng
Tổng điểm 10/100
Cách tính lỗi (trừ điểm)
- Mỗi lỗi ngữ pháp, cấu trúc câu,.. 0,2
- Mỗi lỗi từ vựng 0,2
- Số lượng từ (+ - 30% số từ theo quy định) 1

ĐỀ SỐ 08
I. Câu 01 - 05: Chọn từ (ứng với A, B, C hoặc D) có phần gạch dưới được phát âm khác với những từ
còn lại trong mỗi câu.
01. B 02. C 03. C 04. D 05. A
II. Câu 06 - 10: Chọn từ (ứng vói A, B, C hoặc D) có trọng âm chính nhân vào âm tiết có vị trí khác với
những từ còn lại trong mỗi câu.
06. A 07. C 08. C 09. B 10. C
III. Câu 11 - 20: Đọc đoạn văn sau và chọn phương án đúng nhất (ứng với A, B, C hoặc D) cho mỗi câu
hỏi.
11. D. destroyed by fire
12. B. the rough streets and alleys
13. A. the configuration
14. B. streets
15. B. speech
16. B. disclosed
17. C. They were followed and London was rebuilt at once.
18. C. the burned-down houses and shops
19. C. He feels that someone rather than Wren should have been chosen to plan the rebuilding.
20. D. "The London That Might Have Been"
IV. Câu 21 - 30: Đọc đoạn văn sau và chọn phương án đúng nhất (ứng với A, B, C hoặc D) cho mỗi chỗ
trống.
21. C. style 22. C. expands 23. C. meets 24. A. average
25. A. build 26. A. exactly 27. B. performance 28. A. purpose
29. A. function 30. C. respond
V. Câu 31 - 40: Chọn từ/ cụm từ thích hợp nhất (ứng với A, B, C hoặc D) để hoàn thành mỗi câu sau.
31. A. go through 32. B. beyond 33. C. sought 34. A. inclined
35. D. nothing as ambitious as 36. B. doubt it 37. B. sympathetic
38. B. I told you of 39. A. pick up 40. A. causing many people to die
VI. Câu 41 - 50: Sử dụng từ trong ngoặc ở dạng thích hợp nhất để điền vào các chỗ trống.
41. appreciated 42. appeal 43. craftiness 44. unseen
45. harmoniously 46. essentials 47. intensity 48. combination
49. planning 50. precision
VII. Câu 51 - 60: Tìm từ thích họp nhất để điền vào chỗ trống trong đoạn văn. Mỗi chỗ trống chỉ điền
MỘT từ.
51. discovered 52. public 53. that / who 54. heart /centre

131
55. so 56. Nowhere 57. outside 58. with
59. current / blowing 60. more
VIII Câu 61 - 70: Sử dụng từ cho sẵn trong ngoặc và các từ khác để hoàn thành câu thứ hai sao cho ý
nghĩa của nó tương tự như câu thứ nhất. Chỉ được viết từ 2 đến 5 từ vào Phiếu trả lời và không được
thay đổi dạng thức của từ cho sẵn.
61. Mary broke down as soon as she heard she'd been rejected for the job.
62. It never occurred to me that I would win the lottery.
63. I failed put it across that I didnt want to see him anymore.
64. Please reply at your convenience
65. Sorry, I didn’t follow the track of time.
66. Everyone who spoke to the victim is under suspicion
67. She stands a chance of being chosen for the beauty contest.
68. She asked John to account for what had happened the day before.
69. There are hardly any differences between those two makes of car.
70. Meeting you tomorrow is out of the question
IX. Câu 71 - 80: Viết lại các câu theo gợi ý (kể cả những từ bắt đầu câu cho sẵn) vào Phiếu trả lời sao
cho ý nghĩa ban đầu của câu không thay đổi.
71. It’s sad, but the crime rate is unlikely to go down this year.
Sad as it is, the crime rate is unlikely to go down this year.
72. You must concentrate on your study more.
You must apply yourself with your study.
73. It has been nearly one year since they stopped subscribing to that magazine.
They cancelled subscribing to that magazine nearly a year ago.
74. I don't feel like going to the party.
I am not in the mood for going to the party.
75. "Please don't run so fast!" Suzy begged her friend.
Suzy pleaded with her friend not to run so fast.
76. You should not only balance your diet but also do sports.
In addition to balancing your diet, you should do sports.
77. Immediately after their arrival, the meeting was delayed.
Barely had they arrived when the meeting was delayed.
78. If you don’t pay on time, your booking will be cancelled.
Failure to pay on time will cancel your booking.
79. David is proud of the fact that he is never late.
David prides himself on being never late.
80. It’s nobody's fault that the meeting was cancelled.
Nobody is to blame for the meeting being cancelled / for the cancel of the meeting.
X. Câu 81: Viết một ĐOẠN VĂN khoảng 120 - 150 từ, sử dụng lập luận và dẫn chứng cụ thể để bình
luận về chủ đề sau.
To be admitted to university, using the result of study during high school years is better than that of the
university entrance examination. Do you agree or disagree?

ĐỀ SỐ 09
I. CHOOSE THE CORRECT ANSWER TO FILL IN THE BLANK (2 PTS)
1.D 2. D 3.C 4. C 5.D 6. B 7.D 8. B
9.D 10. D 11.C 12. B 13. B 14. D 15.C 16. B
17.C 18. A 19. D 20. B 21. D 22. A 13. B 24. A

132
25. B 26. D 27. C 28. D 29. B 30.C 31. A 32. A
33. B 34. C 35.A 36. A 37. D 38.C 39. B 40. C

CHOOSE THE WORD OR PHRASE THAT BEST FITS EACH BLANK IN THE FOLLOWING
PASSAGE. (1.5PTS)
l.C 2. B 3. A 4. D 5. C 6. B 7. D 8.C
9. A 10. D 11.C 12. A 13. D 14. D 15. D
IDENTIFY THE MISTAKE IN EACH SENTENCE BELOW. (0.5 PTS)
1. C. which 2. A. of 3. B. that 3. C. necessity
5. D. holidays 6. A. among 7. C. explains 8. B. considered
9. A. their 10. A. have forming
IV. READ THE PASSAGE AND CHOOSE THE BEST ANSWERS TO THE QUESTIONS THAT
FOLLOW. (1PTS)
l.A 2. C 3.C 4. B 5. D
6.B 7. A 8. B 9. A 10. A
V. SUPPLY THE APPROPRIATE FORMS OF THE WORDS IN THE BRACKETS. (1.5 PTS)
1. Joining this project is a no-brainer. Just do it.
2. There are a lot of newsworthy articles in this newspaper. Why not read it?
3. My cousin is a fashion-conscious person. He is aware of all the latest fashions and wanting to follow
them.
4. The burglar gained entry to the building after disabling the alarm.
5. Whatever happens, don't let this failure dishearten you.
6. Your carelessness may do incalculable harm to people.
7. The contraindications listed for the pills meant that she couldn’t take them because she may be allergic to
some of the chemicals in them.
8. We were defeated because we were outnumbered.
9. It is theoretically summer, but it's rather autumnal today.
10. This type of shatter-proof screen enables drivers to have a clear view even when it is smashed.
VI. SUPPLY EACH BLANK WITH ONE SUITABLE WORD. (1.5PTS)
The human race is not the only one species of beings in the living (1) world. Many other species exist
on this planet. However, human beings have a great influence on the (2) rest of the world. They are changing
the environment by building cities and villages where forests (3) once stood. They are affecting the water
supply by using water for industry and agriculture. They are changing (4) weather conditions by cutting
down trees in the forests and are destroying the air by adding pollutants to it.
It can be said that human beings are changing the environment in all respects (5) through their
actions and their habits. This has (6) resulted in two serious consequences. The first is that many species of
animals are killed. The second is that the environment (7) where these animals are living is (8)
badly/heavily destroyed. As a result, the number of rare animals is decreasing so rapidly that they are in
danger of becoming (9) extinct.
In order to make sure that these rare animals do not disappear, (10) efforts have been made to protect
endangered nature. Scientists have made (11) lists of these species and suggested ways to save them. Many
organizations have been set up and funds have been (12) raised. Thousands of national parks all over the
world have been established to protect endangered animals. (13) Laws have been introduced to prohibit
killing endangered animals in the natural habitats where animals are living.
If people's interference with the environment decreases, more species will survive and (14) produce
offspring. The earth will be a happy planet where human beings, animals and plants peacefully (15) coexist.
VII. REWRITE THE FOLLOWING SENTENCES WITHOUT CHANGING THE MEANING.
(2PTS)
1. Hurtful as/though the truth is/may be, you have to come to terms with it.

133
2. Such was the strength of the wind that we could hardly move.
3. The moment this drug takes effect, you will start to feel better.
4. My dad has gone abroad and the office is under my charge.
5. Had he not won that scholarship, he wouldn't have been over the moon.
6. No sooner had he got round to replying to her than he received another letter from her.
7. I'd rather you hadn’t taken me for a ride yesterday.
8. Don’t make a mountain out of a mole hill.
9. To our amazement, he told that/an untruth.
10. Hardly a day goes by without television viewers witnessing some sort of voilence or crime on their
screens.

ĐỀ SỐ 10
Part A: Phonetics (0,5 điểm) Mỗi từ xác định đúng được 0,1 điểm.
1.C. benefit 2. A. casual 3. D. interesting 4. A. purpose 5. D. situation
* Lưu ý: Thí sinh chỉ cần viết đáp án A, B, C hoặc D
Part B: Vocabulary and grammar (3,0 điểm)
I. (2,0 điểm) Mỗi đáp án đúng được 0,1 điểm.
1. B. which 2. B. so that 3. A. are going 4. C. to apply 5. A. Due to
6. D. otherwise 7. B. Few 8. A. That giant pandas are 9. A. advertised
10. D. Not having spoken 11. B. carry out 12. A. try out 13. A. means 14. C. culture
15. A. practice 16. A. Competitors 17. B. stressful 18. B. blamed
19. D. That’s very kind. Thank you. 20. B. Not at all
* Lưu ý: Thí sinh chỉ cần viết đáp án A, B, C hoặc D
II. (1,0 điểm) Mỗi từ viết đúng được 0,1 điểm
1. unemployment 2. interview 3. applicants 4. reference 5. training
6. successfully 7. promotion 8. unlucky 9. pension 10. retired
Part C: Reading (2,5 điểm)
I. (1,0 điểm) Mỗi đáp án đúng được 0,1 điểm
1. A. carried 2. C. contrary 3. A. mind 4.B. or 5.C. said
6. A. go 7. B. real 8. D. out 9. A. unless 10. D. wake up
* Lưu ý: Thí sinh chỉ cần viết đáp án A, B, C hoặc D
II. (0,5 điểm) Mỗi đáp án đúng được 0,1 điểm.
1. A. housework 2. B. less than 3. A. four- fifths
4. D. preparing food for children going to school
5. C. The work of women
* Lưu ý: Thí sinh chỉ cần viết đáp án A, B, C hoặc D
III. (1,0 điểm) Mỗi đáp án đúng được 0,1 điểm.
l.for 2. so 3. in 4. which 5. where
6. is 7. built 8. have 9. house 10. what
Part D: Writing (4,0 điểm)
I. (1,0 điểm) Mỗi đáp án đúng được 0,1 điểm.
l.C 2.C 3.B 4. C 5. D
6. B 7. D 8. D 9. C 10. C
* Lưu ý: Thí sinh chỉ cần viết đáp án A, B, C hoặc D
II. (1,0 điểm) Mỗi câu viết đúng được 0,2 điểm.
1. They are putting PRESSURE ON ME (ME UNDER PRESSURE) TO MAKE a decision soon.
2. His trip to India MADE A STRONG IMPRESSION ON her.

134
3. It isn’t NECESSARY FOR US TO GET a visa for Singapore.
4. He SHOULD HA VE CALLED the police earlier.
5. Keeping calm IS THE SECRET OF PASSING your driving test.
Nếu có sai sót, tuỳ theo mức độ Giám khảo có thể trừ 1/2 số điểm thành phần.
III. (2,0 điểm) Mỗi câu viết đúng được 0,2 điểm.
1. Nam can describe people far better than me.
→ Nam is much better at describing people than me.
2. The boys prefer playing soccer to staying indoors.
→ The boys would rather play soccer than stay indoors.
3. People say that he sold his house to pay the debt.
→He is said to have sold his house to pay the debt.
4. I have never seen such a mess in my life.
→ Never in my life have I seen such a mess.
5. You read a lot. You know much about the world.
→ The more you read, the more you know about the world.
6. Do you have a good relationship with your boss?
→ Are you getting along (getting on well) with your boss?
7. It’s only when you’re ill that you start to appreciate good health.
→ It is not until you are ill that you start to appreciate good health.
8. It was her lack of confidence that surprised me.
→ What I found surprising was her lack of confidence.
9. We haven't decided where to go this weekend yet.
→ We haven't made up our mind where to go this weekend yet.
→ We haven't made a decision about (on) where to go this weekend yet.
10. The heavy rain prevented us from going camping in the mountain.
→ But for the heavy rain, we would (could) have gone camping in the mountain.
* Nếu có sai sót, tuỳ theo mức độ Giám khảo có thể trừ 1/2 số điểm thành phần.

C. NĂM 2016
ĐỀ SỐ 11
A. PHONETICS (3.0 points): 1.0 point for each correct answer
1.B 2.A 3. B
B: GRAMMAR - VOCABULARY - LANGUAGE FUNCTIONS
I. (15 points): 1.0 point for each correct answer
3. C 5. B 6. B 7. D 8. A 9. D
10. D 11. D 12. C 13. C 14. B 15. C
16. A 17. B 18. C 19. D 20. B
II. (5.0 points): 1.0 point for each correct answer
21. inspiration 22. Specialise / specialize
23. atientively 24. unpredictable
25. ugliness
III. (5.0 points): 1.0 point for each correct answer
26. C 27. D 28. B 29. B 30. D
C. READING
I. (5.0 points): 1.0 point for each correct answer
31. D 32. G 33. A 34. B 35. F

135
II. (10 points): 1.0 point for each correct answer
36. B 37. A 38. D 39. D 40. B
41. B 42. B 43. D 44. D 45. C
III. (10 points): 1.0 point for each correct answer
46. A 47. D 48. D 49. D 50. C
51 B 52. C 53. A 54. D 55. C
D. WRITING
I. (5.0 points): 1.0 point for each correct answer
56. So complicated was the problem that nobody could solve it.
57. If Mr. Jacob’s son had not wanted a pet, he would not have bought the parrot.
Or. If his son had not wanted a pet, Mr. Jacob would not have bought the parrot.
58. It came as no surprise (to everybody) to hear that Caroline had not passed her driving test.
59. Attractive as the salary was, my sister refused to get the job.
60. The heavy snow made it impossible for the students to go to the mountain.
II. (10 points):
Dear Mai,
61. I am writing / write to tell you about my holiday (0.5 p).
62. I have been in Kuala Lumpur for two weeks now (0.5 p) and I am having / have had a good time in a
beautiful country (0.5 p).
63. I spent my / the first few days visiting the Twin Towers and other tourist attractions (0.5 p).
64. I must say (that) Malaysian people are (very) friendly (0.5 p) and (they are) always ready to help (0.5 p).
65. Most places in / of Kuala Lumpur are crowded with tourists (0.5 p) (who come / are) from all over the
world (0.5 ,p);
66. Yesterday I went shopping (0.5 p) and bought a few souvenirs for my parents and friends (0.5 p).
67. I lost my way back to the / my hotel (0.5 p) but it did not matter (0.5 p), because I discovered a
fascinating market with lots of little stalls (0.5 p) which / that sell / selling almost everything from needles to
elephants (0.5 p).
68. Today, I have (just) been to a couple of art galleries (0.5 p) and met many / some / a lot of / lots of / a few
interesting people (0.5 p).
69. I guess I have spent a lot of money so far (0.5 p), but Malaysia is a great country to visit (0.5 p).
70. I have taken lots of / a lot of / many / some / a few photos (0.5 p) and I will show you when I get home on
June 10th (0.5 p) .
See you again.
Trang
III. (10 points):
Mô tả tiêu chí đánh giá Điểm tối đa
1. Bố cục 2.0
♦ - Câu đề dẫn chủ đề mạch lạc
♦ - Bố cục hợp lí rõ ràng phù hợp yêu cầu của đề bài
♦ - Bố cục uyển chuyển từ mở bài đến kết luận
2. Phát triển ý 1.5
♦ - Phát triển ý có trình tự lọgic
* - Có dẫn chứng, ví dụ …đủ để bảo vệ ý kiến của mình
3. Sử dụng ngôn ngữ 2.5
♦ - Sử dụng ngôn từ phù hợp nội dung
♦ - Sử dụng ngôn từ đúng văn phong/ thể loại
♦ - Sử dụng từ nối các ý cho bài viết uyển chuyển

136
4. Nội dung 2.5
- Đủ thuyết phục nguời đọc
- - Đủ dẫn chứng, ví dụ, lập luận
- Độ dài: Số từ không nhiều hơn hoặc ít hơn so với quy định-5%
5. Ngữ pháp, dấu câu và chính tả 1.5
- Sử dụng đúng dấu câu
- Chính tả: Viết đúng chính tả
- Lỗi chính tả gây hiểu nhầm / sai lệch ý sẽ bị tính một lỗi (trừ 1%
điểm của bài viết)
- Cùng một lỗi chính tả lặp lại chỉ tính là một lỗi
- Sử dụng đúng thời, thể; cấu trúc câu đúng ngữ pháp. (Lỗi ngữ
pháp gây hiểu nhầm/ sai lệch ý sẽ bị trừ 1% điểm bài viết.)
Tổng 10.0

ĐỀ SỐ 12
PART A: LISTENING - TRACK 1 (2,0 điểm)
Part 1. You are going to listen to a conversation between a university admission officer and a prospective
student. (1,0 điểm: 0,1 điểm cho mỗi đáp án đúng)
1.Hawberry/ HAWBERRY 2. May 22nd
3.26 4.01903714721
5. No (mobile phone) 6. Secretary
7. Business 8. It is full/ Get full/ It’s very popular
9. Intemational Marketing (couse) 10. Admissions (department)
Part 2. Listen to the recording twice and complete the following sentences using ONE word and/or number
for each gap. (1,0 điểm: 0,1 điểm cho mỗi đáp án đúng)
1. family 2. Mexico 3.blues 4. interval 5. 1999
6. 4/four 7. photo 8. Rain 9. bikes 10. excitement
PART B: LEXICO – GRAMMAR (2,5 điểm)
I.Write the letter A, B, C or D on your answer sheet to indicate the correct answer to each of the
following questions. (1,0 điểm: 0,05 điểm cho mỗi đáp án đúng)
1.C 2.A 3.B 4.B 5.D 6.B 7.A 8.D 9.A 10.A
11.C 12.A 13.B 14.C 15.D 16.A 17.B 18.C 19.A 20.D
II. Write the letter A,B,C, or D on your answer sheet to indicate the underlined part that needs
correcting. (0,5 điểm: 0,1 điểm cho mỗi đáp án đúng)
1.D 2.D 3.C 4.B 5.A
III. Complete the following passage with the correct form of the word given in CAPITALS io the right
of each line. (1,0 điểm: 0,1 điểm cho mỗi đáp án đúng)
1. recently 2. discovery 3. interest 4. traditionnallv 5. burial
6. unknown 7. scientists 8. descendant 9. originally 10. criminal
PART C: READING (3,0 điểm)
I. Read the following passage and write the letter A, B, C or D on your answer sheet to indicate the
correct answer to each of the following questions. (1,0 điểm: 0,1 điểm cho mỗi đáp án đúng)
1. B 2.C 3. A 4. D 5. D
6. B 7. B 8. D 9. B 10.B
II. Read the following passage and write the letter A, B, C or D on your answer sheet to indicate the
correct word or phrase that best fits each of the numbered blanks.
(1,0) điểm: 0,1 điểm cho mỗi đáp án đúng)

137
1. B 2. A 3. A 4. D 5. D
6. A 7. C 8.B 9. C 10. A
III. Complete the passage by filling in each blank with ONE suitable word.
(1,0 điểm: 0,1 điểm cho mỗi đáp án đúng)
1. that/which 3. with 5. like 7. named/called 9. at
2. over/on/across 4. was 6. for 8. to 10. made
PART D: LEXICO - GRAMMAR (2,5 điểm)
I. Complete each of the following sentences in such a way that it means exactly the same as the one
printed before it. The word provided (if any) must not be altered in any way.
(0,5 điểm: 0,4 điểm cho mỗi đáp án đúng)
1. Celine has always been fond of classical music. (TASTE)
→ Celine has always had a taste for classical music.
2. They cannot watch this film because they are not adults yet. (UNDER)
→ Since they are under age, they cannot watch this film.
3. It's a pity you didn’t ask us to spend more time with you.
→ If only you had asked us to spend more time with you.
4. She hasn’t spoken much since the day she met the President.
→ Little has she spoken since the day she met the President.
5. People think that someone started the fire deliberately.
→ The fire is thought to have started deliberately.
II. Essay writing (2,0 điểm)
Vietnamese educators are divided on whether the current system of high schools for the gifted should be
maintained. Whilst some argue that the system may promote unhealthy competition or unnecessary stress
among students, and therefore should be abolished, many others have praised the merits of those schools and
voiced strong approval of their existence.
What advantages can schools for the gifted offer to their students and to the society?
Write 250-300 words. RK
Essay marking scheme:
Candidates' essays are evaluated based on the following criteria:
Content (0,8 điểm): Relevant content with adequately supported arguments (with main ideas, details,
examples etc.).
Language (0,6 điểm): Demonstrate a diverse range of lexical items and grammatical structures.
Presentation (0,6 diem): Write with suitable style, cohesion, coherence; appropriate word count.

ĐỀ SỐ 13
SECTION I: LISTENING - TRACK 2 (2.0 đ).
Question 1: 1.0 đ. (Mọi câu làm đúng đạt 0.2 đ.)
1. 2. 3. 4. 5.
your pet(s) northcountiesrescue@gmai . 12th gift certificate calendar
com September/Sept
(4 pm)

Question 2: 1.0 đ. (Mỗi câu làm đúng đạt 0.2 điểm)


6. 7. 8. 9. 10.
C C B B C

138
SECTION II: USE OF ENGLISH (3,0 đ)
Question 1: 1.0 đ. (Mỗi câu làm đúng đạt 0.1 đ.)
11. 12. 13. 14. 15. 16. 17. 18. 19. 20.
D D A A C D C B C A

Question 2: 0.5 đ. (Mỗi câu làm đúng đạt 0.1 đ.)


21. 22. 23. 24. 25.
showing off cut down on gone down with called off dozed off

Question 3: 1.0 đ. (Mỗi câu làm đúng đạt 0.1 đ.)


26. 27. 28. 29. 30. 31. 32. 33. 34. 35.
being for her √ to ourselves that √ of √

Question 4: 0.5 đ. (Mỗi câu làm đúng đạt 0.1 đ.)


36. 37. 38. 39. 40.
troublesome numerous gratitude disobedient speechless

SECTION III: READING (3.0 đ)


Question 1: 1.0 đ. (Mỗi câu làm đúng đạt 0.1 đ.)
41. 42. 43. 44. 45. 46. 47. 48. 49. 50.
B C B A C D D A B B
Question 2: 1,0 đ. (Mỗi câu làm đúng đạt 0,1 đ)
51. 52. 53. 54. 55.
result (a)round/through fact/practice Both should/must
56. 57. 58. 59. 60.
which If/when too/also times therefore/then
Question 3: 1,0 đ (Mỗi câu làm đúng đạt 0,1 đ)
61. 62. 63. 64. 65. 66. 67. 68. 69. 70.
A C B B C C A D C D

SECTION IV: WRITING (2.0 đ)


Question 1: 0.5 đ. (Mỗi câu làm đúng đạt 0.1 đ.)
71. Barack Obama is believed to have deeply studied Vietnamese history and culture before the visit to
our country.
72. I prefer walking to taking a taxi.
73. The party was so lovely that all of them didn’t want to leave until midnight.
74. Except (for) Linh, my daughter has no real friends/doesn’t have any real friends
75. It was not until Susan met her lawyer that she said something to the police.
Question 2: 0.5 đ. (Mỗi câu làm đúng đạt 0.1 đ.)
76. had no intention of going I didn 9t have any intention of
77. there was no point (in)
78. accused Jonathan of having robbed.
79. are not supposed to drive
80. doesn’t matter what/ is not a matter of what
Question 3: 1.0 đ.
Tổ chấm thống nhất điểm thành phần của bài viết theo những điểm đề nghị sau:

139
Mô tả tiêu chí đánh giá Điểm tối đa
1. Bố cục 2.0
♦ - Câu đề dẫn chủ đề mạch lạc
♦ - Bố cục hợp lí rõ ràng phù hợp yêu cầu của đề bài
♦ - Bố cục uyển chuyển từ mở bài đến kết luận
2. Phát triển ý 1.5
♦ - Phát triển ý có trình tự logic
* - Có dẫn chứng, ví dụ …đủ để bảo vệ ý kiến của mình
3. Sử dụng ngôn ngữ 2.5
♦ - Sử dụng ngôn từ phù hợp nội dung
♦ - Sử dụng ngôn từ đúng văn phong/ thể loại
♦ - Sử dụng từ nối các ý cho bài viết uyển chuyển
4. Nội dung 2.5
- Đủ thuyết phục nguời đọc
- - Đủ dẫn chứng, ví dụ, lập luận
- Độ dài: Số từ không nhiều hơn hoặc ít hơn so với quy định-5%
5. Ngữ pháp, dấu câu và chính tả 1.5
- Sử dụng đúng dấu câu
- Chính tả: Viết đúng chính tả
- Lỗi chính tả gây hiểu nhầm / sai lệch ý sẽ bị tính một lỗi (trừ 1%
điểm của bài viết)
- Cùng một lỗi chính tả lặp lại chỉ tính là một lỗi
- Sử dụng đúng thời, thể; cấu trúc câu đúng ngữ pháp. (Lỗi ngữ
pháp gây hiểu nhầm/ sai lệch ý sẽ bị trừ 1% điểm bài viết.)
Tổng 10.0

ĐỀ SỐ 14
A. HƯỚNG DẪN CHUNG
Phần điểm thập phân của toàn bài thi được quy đổi như sau:
Phần điểm thập 0,2 0,4 0,6 0,8
phân của bài thi
Điểm quy đổi 0,25 0,5 0,75
B. ĐÁP ÁN, BIỂU ĐIỂM
Part I. Phonetics
(1 point/ 5 questions; 0.2 point for each correct answer)
1.A 2.B 3.C 4.C 5.D
Part II. Choose the word, phrase or expression which best completes each sentence
(2 points/10 questions; 0.2 point for each correct answer)
1.B 2.A 3.C 4.B 5.C
6.C 7.A 8.D 9.B 10.A
Part III. Give the correct tense or form of the verbs in brackets
(1 point/ 5 questions; 0.2 point for each correct answer)
1. Have ….(ever) talked…..? 2. had 3. was watching
4. to take 5. repainted
Part IV. Identify the underlined word or phrase that must be changed to make the sentence correct

140
(1 point/ 5 questions; 0.2 point for each correct answer)
1.A 2.B 3.D 4.C 5.D
Part V. Read the passage and then choose the correct answer each question
(1 point/ 5 questions; 0.2 point for each correct answer)
1.C 2.D 3.A 4.D 5.C
Part VI. Read the passage and choose the best word to fill in each blank
(2 point/ 10 questions; 0.2 point for each correct answer)
1. pollutes 6. should
2. rubbish 7. who
3. amount 8. and
4. because 9. of
5. burning 10. green

Part VII. Finish each of the following sentences in such a way that it means exactly the same as the
sentence printed before it
(1 point/5 questions; 0.2 point for each correct answer)
1. Nga said that she was going to visit her grandmother the following/next weekend.
2. The doctor whom you met yesterday is working at Bach Mai hospital.
3. The picture has been stolen.
4. Although Mary was sick, she (Mary) insisted on going to work.
5. Scarcely had he put the phone down when the doorbell rang.
Part VIII. Complete each of the following sentences, using the words given
(1 point/5 questions; 0.2 point for each correct answer)
1. My brother enjoys playing football.
2. I have (I’ve) known him since we were primary students.
3. If today were Sunday, I would go fishing with my brother.
4. It took me three hours to get my / the/ our house tidied yesterday.
5. You should apologize to her for not attending her/the wedding.

ĐỀ SỐ 15
(Đề thi vào lớp 10 THPT chuyên Lê Hồng phong – Nam Định)
Thời gian làmbài: 120 phút
PART A. LISTENING – TRACK 3 (2.0 POINT)
I. PART 1: (0.5 p) → 0,1 for each correct answer
1C 2A 3B 4B 5C
II. PART 2: (0.5 p) →0,1 for each correct answer
1. 6 p.m/ at 6p.m
2. professional (equipment)
3. 3 days
4. John Robert(s)
5. (on) Tuesday(s)
III. PART 3: (1.0 p) → 0.1 p for each correct answer
1. performance, value
2. contamination
3. resist
4. 3.74
5. airconditioner(s)

141
6. (the) sandwich bag
7. (the) bread industry
8. 1969
9. Plastic grocery bag(s)
10. 15
PART B. GRAMMAR AND VOCABULARY (2.0 POINTS)
I. PART 1: (1.0 p) → 0.1 p for each correct answer
1.A 2.B 3.A 4.D 5.C 6.D 7.A 8.C 9.B 10.B
II. PART 2: (0.5p) → 0.1 p for each correct answer
1. did → made 2. exciting → excited 3. like → as/ because
4. their → its 5. gave to → gave
III. PART 3: (0.5p) → 0.1 p for each correct answer
1. valuable/ invaluable 2. childhood 3. impolitely 4. injuries 5. encouraged
PART C. READING (3.0 POINTS)
I. PART 1: (0.8p) → 0.1p for each correct answer
1A 2.D 3A 4C
5. North America and Europe / They are North America and Europe
6. (They should buy) simply-wrapped things/ products/ ones and /,/; high-quality products/ ones (in any
order)
7. They/ The garbage dumps have relatively little glass and plastic (from throw-away bottles)
/There are relatively little glass and plastic (from throw- away bottles)
8.Aluminum cans and /; /, spent motor oil.
II. PART 2: (0.8 p) →0.1p for each correct answer
l.D 2.C 3.A 4.D 5.C 6.D 7.B 8.A
III. PART 3: (0.8 p) → 0. 1p for each correct answer
1. ii 2. iv 3. v 4. vii 5. viii 6. True/ T 7. False/ F 8. False/ F
IV. PART 4: (0.6p) → 0.1p for each correct answer
1.is 2. from 3. discharge 4. that/which 5. amount 6. down
PART D. WRITING (3.0 pts)
I. PART 1: (1.5p) → 0.1p for each correct answer
1. Hardly had we finished dinner when the thunderstorm broke.
2. He is thought to have been staying in London at the time of the assault.
3. It came as a surprise to President Obama when he saw / to see the hospitability of Vietnam government
and people.
4. If he didn't have his nose in a book, he would/ might pay attention to what I say
5. Martin's poor health /can’t/ doesn't stop (prevent) him (from) enjoying life.
6. .. .on the point of leaving...
7. .. .led to the cancellation/cancelling/ canceling.
8. .. .computer games are concerned...
9. ...made a great/big/deep impression on...
10... .any chance of Peter/ Peter’s...
11. I am not to blame for this mix-up.
12. My brother is (feeling) (a bit) under the weather these days
13.I am very thankful (to you) for everything you’ve done for me
14. There is no question of my/ me lending you $500.
15 We were not taken in by his smooth manner.

II. PART 2: (1.5p)

142
Mô tả tiêu chí đánh giá
1. Bố cục: 0.3p
- Bố cục hợp lí rõ ràng phù hợp yêu cầu của đề bài, có đầy đủ ba phần: mở bài, thân bài, kết luận
- Mở bài/ câu chủ đề mạch lạc, trả lời đúng nội dung câu hỏi
- Câu kết luận rõ ràng, không trùng lặp mở bài/ câu chủ đề.
2. Phát triển ý: 0.6p
- Các ý được phát triển có trình tự logic, có giải thích, dẫn chứng, lập luận thuyết phụ người đọc
* - Có 2,3 ý rõ ràng, không bị trùng lặp, trả lời đúng nội dung câu hỏi.
3. Sử dụng ngôn ngữ
♦ - Sử dụng ngôn từ, cấu trúc đa dạng phù hợp nội dung, phù hợp trình độ học sinh khá giỏi.
♦ - Sử dụng từ nối các ý cho bài viết uyển chuyển

5. Ngữ pháp, dấu câu và chính tả: 0.3p


- Sử dụng đúng dấu câu
- Chính tả: Viết đúng chính tả
- 2→3 lỗi chính tả sẽ bị trừ 0.1 điểm bài viết( Cùng một lỗi chính tả lặp lại chỉ tính là một lỗi)
- Sử dụng đúng thời, thể; cấu trúc câu đúng ngữ pháp.
- mỗi lỗi ngữ pháp, cấu trúc sẽ bị trừ 0,1 điểm bài viết.)
Nếu sai quá 3 lỗi chấm câu, chính tả, ngữ pháp sẽ bị trừ hết 0.3 điểm
Tổng
Điểm của bài thi là tổng điểm của các câu cộng lại, không làm tròn điểm.

D. NĂM 2017-2018
ĐỀ SỐ 16
I. LISTENING - TRACK 4 (20 points)
The listening test has TWO parts. You will hear each part TWICE. At the beginning of each part, you
will have time to read the questions and at the end of each part, you will have time to complete your
answers.
Part 1: (10 points)
Questions 1-4
Listen to the recording of three students talking to their tutor about the presentation they are
planning. Circle the best letter among A, B, or C.
1.B 2. C 3. A 4. B
Questions 5-7
Listen to the next part of the recording and complete the information for each slide. Write ONE
WORD ONLY.
5. Advantages 6. Accidents 7. Suggestions
Questions 8-10
Listen to the last part of the recording and complete the sentences. Write NO MORE THAN TWO
WORDS AND/OR A NUMBER.
8. seven / 7 minutes 9. two / 2 minutes 10. same style
Part 2: (10 points)
You will hear a woman called Yvonne on a TV programme giving her opinion about children being
punished at school. For questions 1 - 10, complete the sentences with NO MORE THAN THREE
WORDS.

143
1. hitting 2. telling off 3.(the) wrong shoes 4.(very) cruel 5. (young) children
6. 7. 8. 9. 10.
the government lazy how lucky changed the law teenage crime
II. VOCABULARY AND GRAMMAR (10 points)
for each of the following questions, circle the best answer (A, B, C or D) to fill the blank.
2. B 3. B 4. C 5. B 6. A 7. A 8. A 9. D 10. C
12. D 13. A 14. D 15. A 16. B 17. A 18. D 19. D 20. C
III. READING (25 points)
Part 1: Read the following passage, choose and circle the best answer (A, B, C or D) to fill in each of
the blanks. (10 points)
2. B 3. B 4. C 5. D 6. B 7. A 8. C 9. D 10. A
Part 2: Read the text below and think of the word which best fits each space.
Use only ONE word in each space. (10 points)
1. mean 2. little 3. more 4. all 5. what
6. than 7. in 8. much 9. another 10. tell
Part 3: You are going to read a newspaper article about the problems caused by acid rain. Six
sentences have been removed from the article. Choose from the sentences A—G the one which fits each
gap (0-5). Number (0) has been done for you. There is one extra sentence which you do not need to use.
(5 points)
1.F 2. A 3. B 4. G 5. D
IV. WRITING (25 points)
Part 1: For each question, complete the second sentence in such a way that it is as similar in meaning
as possible to the one printed before it. (5 points)
1. There has been a(n) [huge/dramatic/big/enormous/considerable] increase in the number of road accidents
in recent months.
2. The smoking of cigarettes is no longer as [popular/ widespread] as it used to be/ The smoking of cigarettes
used to be more popular/widespread.
3. I have no intention of apologizing to either of them.
4. Against [all/everybody's/everyone's] expectations, she lost.
5 It’s not the job I'm interested in; it's the people.
Part 2: For each question, complete the second sentence in such a way that it has the closest meaning to
the original one, using the WORD given. DO NOT change this word. You must use between three and
five words, including the word given. (5 points)
1. had trouble (in) following
2. couldn’t put up with
3. had better leave soon
4. expected things would be much
5. in silence because/as/since they were
Part 3: Within from 200 to 250 words, write a short composition on the following topic. (15 points)
Should teenagers spend more time on such cultural activities as enjoying music or theatre, etc and less time
on sports? Why or why not?
1. Content (7 points)
- The essay shows students’ persuasive ideas on the topic.
- Communicative goals are achieved.
2. Organization (4 points)
Clear organization of ideas, with suitable paragraphing and linking

144
3. Grammar and vocabulary (4 points)
Using good grammar in standard English with a large variety of vocabulary.
TT Tiêu chí chấm kỹ năng nói Điểm Ghi chú
1 Pronunciation and Intonation - 4 điểm
2 Grammatical Range and Accuracy - 4 điểm
3 Lexical Resource — 4 diem
4 Fluency and Coherence - 4 điểm
5 Content - 4 điểm
Tổng cộng /20 điểm

ĐỀ SỐ 17
PART A. LEXICO-GRAMMAR (2,5 điểm)
I. Write the letter A, B, C or D on your answer sheet to indicate the correct answer to each of the
following questions. (1,0 điểm: 0,05 điểm cho mỗi đáp án đúng)
1. B 5. D 9. B 13. A 17. A
2. B 6. D 10. D 14. B 18. C
3. C 7. B 11.D 15. A 19. B
4. C 8. D 12. D 16. A 20. A
II. Write the letter A, B, C, or D on your answer sheet to indicate the underlined part that needs
correcting. (0,5 điểm: 0,1 điểm cho mỗi đáp án đúng)
l.C 2. C 3. B 4. D 5. D
III. Complete the following passage with the correct form of the word given in CAPITALS. (1,0 điểm:
0,1 điểm cho mỗi đáp án đúng. Thí sinh dùng chữ viết thường hay in hoa đều được chấp nhận)
1. appealing 3. violent 5. Towering 7. sheltered 9. unpleasant
2. grandeur 4. eruption 6. strength 8. atmospheric 10. inescapable
PART B. READING (3,0 điểm)
I. Read the following passage and write the letter A, B, C or D on your answer sheet to indicate the
correct answer to each of the following questions. (1,0 điểm: 0,1 điểm cho mỗi đáp án đúng)
1. C 3. A 5. A 7. D 9. D
2. B 4. D 6. C 8. A 10. B
II. You are going to read an article in which four crime writers talk about other authors they like. For
questions 1-10, choose from the crime writers (A-D). The writers may be chosen more than once. (1,0
điểm: 0,1 điểm cho mỗi đáp án đúng)
1. D 3. A 5.C 7. A 9. B
2. A 4. D 6. D 8. B 10. C
III. Read the article below about the illegal international trade of whale meat. Six sentences have been
removed from the article. Choose from the sentences A - G the one which fits each gap 1-5. There is
one extra sentence which you do not need to use.
(1,0 điểm: 0,2 điểm cho mỗi đáp án đúng)
1. D 2. B 3. E 4. A 5. C
PART C. WRITING (2,5 điểm)
I. Complete the second sentence so that it has a similar meaning to the first sentence, using the word

145
given. Do not change the word given. You must use between two and five words, including the word
given. (0,5 điểm: 0,1 điểm cho mỗi đáp án đúng)
1. They offered her a job but she refused. (TURNED)
→ She turned down the offer of a job.
2. I find driving on the left in England very strange. (ACCUSTOMED)
→ I am not (yet) accustomed to driving on the left in England.
3. I expect you were very tired by the end of the week. (MUST)
→ By the end of the week you must have been exhausted.
4. Being late is inexcusable. (EXCUSE)
→ There is no excuse for being late.
5. Matthew decided not to do his homework and went to play football. (INSTEAD)
→ Matthew went to play football instead of doing his homework.
II. Essay writing (2,0 điểm)
Every year increasing number of students are doing their higher education abroad.
Do you think the benefits of this development outweigh the problems associated with it?
Write 250-300 words.
Essay marking scheme:
Candidates’ essays are evaluated based on the following criteria:
Content (0,8 điểm): Relevant content with adequately supported arguments (with main ideas, details,
examples etc.).
Language (0,6 điểm): Demonstrate a diverse range of lexical items and grammatical structures.
Presentation (0,6 điểm): Write with suitable style, cohesion, coherence; appropriate word count.

ĐỀ SỐ 18
PART A. GRAMMAR AND VOCABULARY (2.0 POINTS)
I. (1.0 p) → 0.1p for each correct answer
1. C 2. A 3. B 4. A 5. B
6. A 7. B 8. A 9. B 10. D
II. (0.5 p)→0.1p for each correct answer
1. was - were 2. neither - either 3. All of-Of all 4. generating - 5. contact with -
generate contact

III. (0.5 p) → 0.1p for each correct answer


1. satisfaction 2. rebuilt 3. unpredictable 4. imprisoned 5. assistance
PART B. READING (3.0 POINTS)
I. (1.0 p)→0. 1p for each correct answer
1. A 2. D 3. C 4. A 5. B
6. C 7. A 8. B 9. C 10. D
II. (1.0 p)→0.2p for each correct answer
1. maximum 2. a 3. speaking 4.test 5. class
6. beginners 7.situations/contexts 9. up
8.learnt/studied 10.range/variety
PART C. WRITING (3.0 POINTS)
I. (0.5 p)→0.1p for each correct answer
1. not going/ not having gone to his birthday party.

146
2. you not missed/ you attended the class yesterday, you could do this exercise now.
3. Such was her physical attractiveness that every boy ran after her.
4. have taken the money by accident.
5. has been a dramatic fall/ decline/decrease in the number of people out of work this year.
II. (1.0 p) → 0.1p for each correct answer
1. have no intention of taking a part-time job this summer/
- don’t have any intention of taking a part-time job this summer.
2. only I hadn’t told John about my plan.
3. is bound to fail the coming driving test, (not “in” before ‘the coming driving test”)
4. didn’t have enough courage to tell me what he really thought.
5. climate has (a) strong influence on people’s personalities?
6. it hadn’t been for the heavy rain yesterday, I wouldn’t have been ill.
7. was not until he was 40 years old that he got married.
8. gave a detailed description of the events of that day.
9. had I put the phone down when it rang again.
10. any recollection of (his) phoning/ (his) having phoned me.
III. (1.5p)
Mô tả tiêu chí đánh giá Cụ thể
1. Bố cục: 0.3 p
- Bố cục hợp lí rõ ràng phù hợp yêu cầu của đề bài, có đầy đủ ba phần: mở bài, 0.1
thân bài, kết luận
- Mở bài/ câu chủ đề mạch lạc, trả lời đúng nội dung câu hỏi 0.1
- Câu kết luận rõ ràng, không trùng lặp mở bài/ câu chủ đề 0.1
2. Phát triển ý: 0.5 p
- Có 2-4 ý rõ ràng, không bị trùng lặp, trả lời đúng nội dung câu hỏi 0.3
- Các ý được phát triển có trình tự logic, có giải thích, dẫn chứng, lập luận 0.3
thuyết phục người đọc.
3. Sử dụng ngôn ngừ: 0.4p
Sử dụng ngôn từ, cấu trúc đa dạng phù hợp nội dung, phù hợp trình độ học sinh 0.2
khá giỏi. 0.1
Sử dụng từ nối các ý cho bài viết uyển chuyển
4. Ngữ pháp, dấu câu và chính tả: 0.3p
- Sử dụng đúng dấu câu -
- Chính tả: Viết đúng chính tả
Hai lỗi chính tả sẽ bị trừ 0.1 điểm bài viết. (Cùng một lỗ chính tả lặp lại chỉ
tính là một lỗi)
- Sử dụng đúng thời, thể, cấu trúc câu đúng ngữ pháp.
- Hai lỗi ngữ pháp, cấu trúc sẽ bị trừ 0.1 điểm bài viết.
Nếu sai quá 6 lỗi chấm câu, chính tả, ngữ pháp sẽ bị trừ hết 0.3 điểm.
Tổng 1.5

ĐỀ SỐ 19
SECTION A. LISTENING - TRACK 5 (2 points)
Part 1. Listen and fill in the missing information. (1 point)
Your answers:
1. comedy 4. costumes
2. March 12 th
5. 07955240063

147
3. community
Part 2. You will hear a news reporter called Angela Bond, talking on the radio about her job.
Listen and choose the best answer. (1 point)
Your answers:
6 7 8 9 10
B B C A B
SECTION B. LANGUAGE USE (2 points)
Part I. Choose the word or phrase which best complete each sentence. (1 point)
Your answers:
11 12 13 14 15 16 17 18 19 20
C A B B C D C B A C
Part 2. Choose the option A, B, C or D to indicate the word(s) SIMILAR in meaning to the underlined
word(s) in each of the following sentences. (0.5 point)
Your answers:
21 22 23 24 25
A D C B C
Part 3. Give the correct form of the words in capital letters. (0,5 point)
Your answers;
26. satisfaction 29. Amusing
27. rebuilt 30. Unfortunately
28. unpredictable
SECTION C. READING (3 points)
Part 1. Fill in each numbered blank with ONE suitable word. (1 point)
Your answers:
31. to / or
32. train
33. decisions
34. on
35. well / effectively / best
36. Simply
37. Not
38. The
39. demanding / challenging
40. when
Part 2. Read the article on “Spyware ” and write questions A - E in the correct place 41 - 45. (1 point)
A. What is Spy ware D. How does it get on to my computer?
B. How do I get rid of it? E. Is Spyware a common problem?
C. What can it do?
Your answers:
41 42 43 44 45
A D C E B
Part 3. Read the passage carefully and choose the correct answer. (1 point)
Your answers:
46 47 48 49 50
A D C C A
SECTION D. WRITING (3 points)
Part 1. Complete the second sentence, using the word given in bold. Do not change the word given.

148
You must use BETWEEN THREE AND SIX WORDS, including the word given. (1 point)
51. We take pride in our work.
52. Five people took part in the discussion.
53. To everyone’s surprise, John left the meeting early.
54. Peter was on the point of leaving the office when his boss asked him to type up a report.
55. The task is not simple (easy) enough for students in the class to understand.
Part 2. Finish the second sentence in each pair in such a way that it means the same as the sentence before
it. (0.5 point)
56. They have recycled waste paper into newsprint.
Waste paper has been recycled into newsprint.
57. It’s a two-hour flight from Can Tho to Ha Noi.
It takes two hours to fly from Can Tho to Hanoi.
58. “You’d better do more morning exercise, Minh,” I said.
I advised Minh to do more morning exercise.
59. The boys prefer playing soccer to staying indoors.
The boys would rather play soccer than stay indoors.
60. Their dog was so fierce that nobody would visit them.
They had such a fierce dig that nobody would visit them.
Part 3. Write a paragraph of about 120 words about the benefits of studying at a high school for the gifted.
(1.5 points)

ĐỀ SỐ 20
I
1. B 2. A 3. H 4. C 5. D
II.
6. B 7.C 8. D 9. A
III.
10. A 11.C 12. C 13. D 14. A 15. A 16. B 17. B
18. B 19. D 20. D 21. B 22. C 23. C 24. A 25. D
IV.
26. D 31.H 36. A 41. H 46. B
27. B 32. F 37. D 42. C 47. D
28. A 33.1 38. A 43. E 48. A
29. D 34. A 39. C 44. B 49. D
30. C 35. C 40. B 45. D 50. B
V.
51. Money isn’t so important as health.
52. They suggest building a library in their village.
53. She hasn’t ridden a bike for three years.
54. Michael spent half an hour taking his dog for a walk.
55. Lindsey wishes she had not told them her secret.
WRITING
A Suggested Writing
If I were asked whether I would like to live in the city or in the countryside, my answer would
definitely be in the countryside. Living in the countryside brings me a lot of benefits. First, in the
countryside, there is lots of fresh air, so people can enjoy a happy life without clean and fresh air. Second, in
the countryside, there is not so much traffic as in the city, so people will not suffer from too much noise and
they can enjoy a peaceful, quiet and safe life. Last but not least, people in the countryside are very friendly

149
and easy-going. They always smile and give cheerful greetings when they meet their acquaintances and even
visitors. [118 words]

ĐỀ SỐ 21
1. D 2. A 3. C 4. C 5. B 6. B 7. B 8. D 9. B 10. D
11. A 12. A 13. D 14. A 15. A 16. B 17. B 18. B 19. C 20. C
21. B 22. C 23. A 24. D 25. C 26. D 27. D 28. C 29. A 30. C
31. E 32. D 33. C 34. B 35. D 36. C 37. B 38. E 39. A 40. E
41. D 42. B
43. urbanites 44. intimately 45. social 46. promoting 47. fertility
48. pronounced 49. effectively 50. diversity 51. equality 52. excluding
53. without 54. they 55. goes 56. light 57. when
58. in 59. also 60. aware 61. against 62. just
63. out 64. down 65. up 66. Around 67. for
68. mind 69. held 70. rise
71. my amazement, nothing went...
72. no circumstances will Jane ever forgive
73. come up with
74. chances are you won’t
75. burst into flames
76. is thought to be caused by heavy traffic
77. is the popular girl that everyone voted for her
78. But for the smoke alarms going off, we wouldn’t have got out of the building safely.
79. I wish Jane didn’t get angry so often with other staff
80. Given that Evan is younger that others in his class, he is doing very well at school.
81. line 1: hoping → hopes
82. line 2: especial → specially
83. line 4: into → over
84. line 5: reading → readings
85. line 10: latter → later
A Suggested Writing
Nowadays, I am in living in the hi-tech world with modern devices and equipment, so to be a
successful 21st-century learner I should acquire essential skills. In my opinion, the first skill is how to use the
computer well. As I can see that today everything is programmed by computers and operated under people 's
control. Therefore, if I am good at using the computer and take full advantages of them, I can have a brighter
future. The next skill I should have is survival skills (such as cooking, swimming, struggling and self
protecting) which are the basic ones I have to know to help me survive the most difficult situations. Last but
not least, I also need to have some practical skills. They are the supportive factors to help me get on better
with other people and adapt myself more easily to a new living or working environment. [147 words]

E. ĐỀ THI NĂM 2018 - 2019


ĐỀ SỐ 22
I. USE OF ENGLISH.
PART A. CHOOSE THE CORRECT ANSWER TO FILL IN THE BLANKS
1. A 2. B 3. D 4. C 5. D 6.B 7. B 8. A 9.C 10. B
11. B 12. C 13. B 14. D 15. B 16. D 17. D 18. A 19. C 20. D

150
21. B 22. A 23. A 24. C 25. B 26. B 27. B 28. C 29. A 30. D

PART B. CHOOSE THE WORD OR PHRASE THAT FITS EACH SPACE IN THE FOLLOING
PASSAGE.
1. D 2. C 3. C 4. D 5. D 6.C 7. A 8. B
9. A 10. B 11.C 12. D 13. A 14. C 15. A
II. READING.
PART A. READ THE PASSAGE AND CHOOSE THE BEST ANSWERS TO THE QUESTIONS.
1. B 2. C 3. B 4. D 5. A 6. C 7. D 8. B 9. D 10. C
PART B: REARRANGE THE FOLLOWING SENTENCES so THAT THEY MAKE A
MEANINGFUL REVIEW OF A COMPUTER GAME.
1. C. 2. E. 3. B. 4. D. 5. A.
PART C. SUPPLY EACH BLANK WITH ONE SUITABLE WORD.
PASSGE 1.
1. fewer 2. number 3. cheaper 4. account 5. Another
6. spent 7. so 8. for 9. use 10. present
PASSGE 2.
1. granted 2. lives 3. what 4. happens 5. button
6. thought 7. technology 8. level 9. immunity 10. ourselves
I. ERROR CORRECTION
IDENTIFY THE FIVE (5) MISTAKES IN THE FOLLOWING PASSAGE AND CORRECT THEM.
1. interact → interacting
2. nutritions → nutrition
3. other → another
4. careful → carefully
5. economical → economic
II. WORD FORMS
1. disapointedly 2. traffic-free 3. booke-marked 4. devotees
5. fast-paced 6. life-threatening 7. aplogetically 8. multitasking
9. underscore 10. On-the-spot
WRITING.
1. I would prefer it if you got / had the computer fixed.
2. Such was Jack’s nervousness that he could not think straight.
3. The management insisted that we wear / on us wearing / on our wearing dark suits to the meeting all at
once.
4. In a nutshell, they cannot do away with this school regulation.
5. Trying to bring someone around / round to your (point of) view can be a formidable task.
6. After the death of her father, Anna came into a fortune.
Or After the death of Anna’s father, she came into a fortune.
7. If it hadn’t been for the lift you gave us, we wouldn’t have gone to school on / in time.
8. Only with difficulty did I succeed in finishing the task.
9. Much as she disagreed / didn’t agree with the management’s decision, Chloe had no choice / was left with
no choice but / other than to accept it.
10. Fred waited for Sophie to finish her homework before telling her his news.

ĐỀ SỐ 23
151
SECTION A. PHONETICS (10 points)
Part I. Choose the word whose underlined part is pronounced differently from that of the rest.
(5x1=5 points)
1. B 2. B 3. A 4. C 5. D
Part II. Choose the word that has a different stress pattern from the others in the group.
(5 X 1=5 points)
l.A 2.B 3.D 4.A 5.D
SECTION B. LEXICO AND GRAMMAR (25 points)
Part I. Choose the word, phrase or expression which best completes each sentence.
(10 0.5 = 5 points)
1. B 2. C 3.D 4. B 5. D
6. D 7. A 8. B 9.C 10. D
Part II. Give the correct form of the words (10 X 1 = 10 points)
1 loss 6.recommendation
2.improvement 7.uncomfortable
3.approximately 8.gradually
4.Experienced 9.distance
5.check-up 10.possible
Part III. There are ten mistakes in the following passage. Find, underline and correct them. (10 X 0.5 =
5 points)
People are destroying the Earth. The seas and rivers are (1) so dirty to 1. too
swim in. There is so much smoke in the air that it is (2) healthy to live in 2. unhealthy
many of the world’s cities. In one well-known city, for examples. (3) 3. poisonous
poison gases from cars pollute the air so much that (4) policeman have to 4. policemen
wear oxygen masks. We have cut (5) on so many trees that there are now 5. down
vast areas of wasteland all over the world. As a result, so many farmers in 6. grow enough
Darts of Africacan donot (6) enough grow to eat. In some 7. Asia
countries in (7)Asian there is so little rice. Moreover, we do not take 8. quickly
enough care of the countryside wild animals are (8) quick diappearing. For 9. many
instance, tigers are rare in India now because we have killed too (9) much 10. the
for them to survive. However, it isn’t so simple to talk about (10) a
problem. We must act now before it is too late to do anything about it. Join
us now!
Part IV. Give the correct tense or form of the verbs in brackets. (5x1=5 points)
points)
1. has not/hasn't been seen 4. have been running
2. to take 5. had visited
3. was doing
SECTION C. READING COMPREHENSION (25 points)
Part I. Fill in each gap with ONE suitable word to complete the following paragraph.(10 0,5 = 5 points)
1. almost 2. be 3. own 4. which/that 5. a
6. daytime 7. large 8. costs 9. expensive 10. friendly
Part II. Choose the best word A, B, C or D to fill in spaces in the following passage. (10x1 = 10 points)
l.B 2. D 3. A 4. C 5. A
6. B 7. B 8.C 9. D 10. D
Part III. Choose the most suitable heading from the list A - F for each paragraph from 1 - 5 of the
article. (5 X 1= 5points )

152
1.C 2.E 3.B 4.F 5.A
Part IV. Read the passage below, then choose the correct answer A, B, C or D to each of the following
questions (5x1=5 points)
Your answers:
1. A 2. C 3. D 4. A 5. A
SECTION D. WRITING (20 points):
Part 1. Finish each of the following sentences in such a way that It means exactly the same as the
sentence printed before it. (5x1=5 points)
1. It’s thought that he is staying in London at the, time of the accident.
He is thought to be staying in London at the time of the accident.
2. The train left before he gpt to the station.
By the time he got to the station, the train had left.
3. If Cathy didn’t help me, I wouldn’t finish my work on time
But for Cathy’s help, I wouldn’t finish my work on time.
4. Linda was sorry that she didn’t say goodbye to her relatives at the airport.
Linda regretted not saying/not having said goodbye to her relatives at the airport.
5. He never suspected that the money had been stolen.
At no time did he suspect that the money had been stolen.
Part II. Rewrite each of the following sentences so that It means exactly the same as the given one.
Using the given word in CAPITAL letters. Do not change the word. (5x1=5 points)
1. I can’t believe that he passed the exam. FIND
I find it hard/ impossible to believe that he passed the exam.
2. This traditional craft tillage rs often transferred from generation to generation. DOWN
This traditional craft village is often passed down from generation to generation.
3. His coming to the party last night was unexpected. BLUE
His coming to the party last night was put of the blue.
4. Could you guard my handbag while I go to the shop? EYE
Could you keep an eye on my handbag while I go to the toilet?
5. He studies hard, as the result, he gets good marks. THE
The harder he studies, the better marks he gets.
Part III. Writing (10 points)
Recently, there have been some changes in the roles of teachers and students.
Write a paragraph of about 180 words about the changes and their effects.
a. Form : 2 points
- A paragraph with appropriate form, language styles and length (about 180 words )
- handwriting: clean and clear layout, easy to read
b. Contents: 4 point
- successful fulfillment of the task with appropriate content
c. Use of language (4 points )
- Appropriate vocabulary
- Correct grammatical structures and spelling
- Right use of connectors and punctuation.

ĐỀ SỐ 24
HƯỚNG DẪN PHẦN THI NGHE HIỂU
- Trước khi bắt đầu mỗi phần thí sinh có 20 giây để đọc đề.
- Sau mỗi phần thí sinh có 1 phút để kiểm tra đáp án,
- Bài nghe gồm 2 phần, mỗi phần nghe 2 lần

153
A. LISTENING - TRACK 6 (1 pt)
Part 1: You will hear an interview with a teenager called Amy Manero, who is talking about her
interest in music, particularly jazz piano. For each question, choose the correct answer A, B or C.
1.B 2.C 3. B 4. B 5.C
Part 2. You will hear a boy called Adam telling his class about a hot air balloon flight he went on. For
each question, fill in the missing information in the numbered space.
6. an hour 7. mountains 8. bridge 9. stand 10. golf course
I. TRẮC NGHIỆM NGÔN NGỮ (3,5 điểm)
PART 1: PHONETICS (0.5 pts - 0.1/ each)
Task 1: Choose the word (A, B, C or D) whose underlined part is pronounced differently from that of
the rest in each 0 f the following questions.
11. B. valentine 12. D. question
Task 2: Choose the word (A, B, C or D) whose main stress pattern is different from that 0 f the rest in
each of the following questions.
13. C. necessary 14. A. delicious 15. B. imagination
PART 2: LANGUAGE FUNCTION (0.5 pts - 0.1/ each)
Choose the word or phrase (A, B, C or D) that best completes each of the following exchanges.
16. What an attractive hair style you have got, Mary! - Đây là lời khen
=> Đáp án là C. Thank you for your compliment! - Cám ơn lời khen của bạn
Các đáp án khác không hợp ngữ cảnh:
A. Cám ơn bạn rất nhiều, Tôi e sợ.
B. Bạn đang nói dối.
D. Tôi không thích lời nói của bạn.
17. Đây là cuộc trò chuyện qua điện thoại.
Dick: Rất tiếc, Brian không có ở đây
=> Đáp án là D. Tôi có thể để lại lời nhắn không?
Câu "Can I leave a message?" khi người gọi muốn để lại lời nhắn cho người không có mặt ở đó.
Còn , "Can I take a message?" là ý chỉ rằng "Tôi có thể ghi lại lời nhắn giùm được hay không?"
- thường là câu nói của người nghe, khi người trung gian không có mặt ở đó.
18. Câu này hỏi về kỹ năng giao tiếp.
Tom says he doesn’t like you! - Tom nói là anh ta không thích cậu.
Đáp án là D. I don’t care.- Tớ không quan tâm.
19. Chọn D.
Communicative speaking
- Sentence: "Tôi có thể hút thuốc được không?" - "cứ tự nhiên thôi"
20. Laura: “What a lovely house you have!” - Maria: “…………”.
Câu này hỏi về kỹ năng giao tiếp.
What a lovely house you have! - Nhà của bạn thật đẹp! => Đây là lời khen
Chọn đáp án B. Thank you. Hope you will drop in - Cảm ơn. Hy vọng bạn sẽ ghé thăm.
PART 3: SYNONYM AND ANTONYM (0.5 pts - 0.1/ each)
Task 1: Choose the word or phrase (A, B, C or D) that is CLOSEST in meaning to the underlined part
in each of the following sentences.
21. A. busy 22. A. skillful 23. B. trying
Task 2: Choose the word or phrase (A, B, C or D) that is OPPOSITE in meaning to the underlined
part in each of the following sentences.
24. B. unsure 25. A. happiness
PART 4: LEXICO - GRAMMAR (2.0 pts - 0.1/ each)
Choose the word or phrase (A, B, C or D) that best completes each of the following sentences.
26. D 27. A 28. A 29. C 30. D 31. B 32. D 33. A 34. B 35. D

154
36. D 37. C 38. A 39. B 40. D 41. B 42. C 43. A 44. A 45. A
II. TỰ LUẬN KIẾN THỨC NGÔN NGỮ (1,5 điểm)
PART 1: WORD FORM (0.5 pts - 0.05/ each)
Give the correct form of the words in brackets to complete the following sentences.
46. In the exam we had to write one descriptive essay. DESCRIBE
47. The reports are treated as strictly confidential. CONFIDENCE
48. They knew that a vaccine for the virus was theoretically possible. THEORY
48. The streets were illuminated with strings of coloured lights. LUMINOUS
49. Children normally feel a lot of anxiety about their first day at school. ANXIOUS
50. Safety tests on old cars have been standardised throughout Europe. STANDARD
51. We eventually began the laborious task of sorting through his papers. LABOUR
52. The engineering sector achieved significant growth last year. GROW
53. There is (an) supposedly simple explanation of what happened. SUPPOSE
54. The magazine offers tips on cutting your house maintenance costs. MAINTAIN
55. Over 30,000 competitors will run in the New York marathon. COMPETE
PART 2: ERROR CORRECTION (0.5 pts - 0.1/ each)
There are 05 errors in the following passage. Identify the errors, write the line number and correct
them. Write your answers in the numbered spaces below. Number (00.) is done as an example.
Line Error Correction
1 00. health → healthy
2 56. nicely → nice
4 57. poisonous → poison
58. fortunately → Unfortunately
10 59. wood → wooden
14 60. it → them

PART 3: PREPOSITIONS AND PHRASAL VERBS (0.5 pts n 0.05/ each)


Supply each space with a suitable preposition or adverbial particle to complete each of the following
sentences.
51. The certificates can be exchanged for goods in any of our stores.
52. Their stated aim was to free women from domestic slavery.
63. I think they've gone to the airport to see their boss off.
64. Her injuries are consistent with having fallen from the building.
65. He made a good impression on his first day at work.
66. Why should you be displeased at / with the fun of the children?
be displeased al (with) something: không hài lòng về cái gì, bực mình về cái gì
67. Be temperate in speech as well as in eating and drinking.
68. Rewards should be proportionate to merit.
69. Where is Anna? She should be here by now.
70.Without heat from the sun, there would be no life on earth.
KỸ NĂNG NGÔN NGỮ
PART 1: READING (2 pts - 0.1/ each)
Task I: GUIDED CLOZE TEST
Choose the word or phrase (A, B, C or D) that best fits each blank in the following passage.

71. C 72. D 73. C 74. C 75. B 76. A 77. C 78. B 79. D 80. B
Task 2: READING COMPREHENSION

155
Read the passage and choose the best answer for each question below.
81. A 82. C 83. C 84. D 85. D 86. D 87. B 88. B |89. A 90. D

PART 2: WRITING (2 pts)


Task 1: SENTENCE TRANSFORMATION (1.0 pt - 0.2/ each)
Rewrite the following sentences in such a way that their meanings remain unchanged, using the words
given.
91. He apologized for not coming / having come to the meeting yesterday.
92. Only after/when all the guests had gone (home) could we relax / were we able to relax
93. There has been a sharp increase/ rise in (the) house prices this year
94. The police accused Jim of having stolen the money.
95. His illness made him impossible to work effectively.
Part 2:
From 120 to 150 words, write a paragraph about why is it important to protect our environment? What
should we do to fulfill this task?
We live in the environment that is polluted, we need a healthy environment in order to survive. We need to
protect the environment now to help prevent health problems, and to preserve the Earth for our children. We
all know that pollution from factories and cars can cause huge damage to the atmosphere. It makes the air
dirty. Breathing this air causes lung diseases, particularly for children and the elderly. We need to control the
amounts of pollution we produce in order to prevent health problems. Furthermore, we also need to pay
attention to the ecosystem. Plant life, animal life, and people all depend on each other. Polluted environment
disturbs our ecosystem. For example, pollution might cause certain kinds of plants to die. If these plants are
food for some certain kind of animals, they will die too. If people use these animals as food source, there
could be a big problem. To avoid the big problems, factories should pollute as little as possible. If we do not
protect our environment, it will continue to get worse. That leads our children to suffer the consequences.
The air and water will be dirtier, natural resources will become scarcer, and more plants and animals will die.
Even worse, their well-being will be threatened. In short, with contaminated air to breathe, an unhealthy
ecosystem, that’s the bad future for our generation, the human race will not survive. That is why protecting
our environment is crucial.
ĐỀ SỐ 25
I. LISTENING - TRACK 7 (20 pts)
Part 1: (4pts): Mỗi đáp án đúng được 1 điểm
l.B 2. C 3. B 4.A
Part 2: (10 pts): Mỗi đáp án đúng được 1 điểm
1. Kramer 6. nine years
2. 58/ fifty eight 7. long distance
3. Residential 8. Internet
4. office manager 9. (next) Friday
5. 6375559014 10. morning

Part 3: (6 pts): Mỗi đáp án đúng được 1 điểm


1. F 2. F 3.T 4. F 5. T 6. F

THE END OF THE LISTENING


II. (12 pts): Mỗi đáp án đúng được 1 điểm
1. A. otherwise 7. B. have stolen
2. C. off 8. C. be built

156
3. C. was it 9. C. Your child is just adorable!
4. D. well-known 10. D. difficulty passing
5. D. Sorry, I’m a new comer here than 11. D. much more water resistant
6. A. by 12. D. that beings from Mars
III. (5pts): Mỗi đáp án đúng được 1 điểm
1. significant 2. visiting 3. various
1. privately 5. Provincial
IV. (5 pts): Mỗi đáp án đúng được 0,5 điểm
0.A. l. D. 2. G. 3.I. 4. K. 5. F. 6.J. 7. C. 8. E. 9.B. 10. H.
II. (10 pts): Mỗi đáp án đúng được 1 điểm (gạch chân đúng lỗi sai được 0,5 điểm; sửa đúng được 0,5
điểm mỗi câu)
Line Error Correction
Line 1 effective effect
Line 2 state states
Line 2 installing installed
Line__3__ be complied comply
Line__4__ displease displeased
Line__6__ one ones
Line__6__ cost costing
Line 7 to for
Line 8 charges charge
Line 10 smoking smoke

VI. (13 pts) Mỗi đáp án đúng được 1 điểm


Questions 1-6:
1.F 2. A 3.B 4. D 5.I 6. C
Questions 7-9:
7. B. punishment helps learning.
8. D. give punishment according to a rule.
9. C. underestimated the teacher-subjects’ willingness to comply with experimental procedure.
Questions 10-13: 10. NG 11. T 12. F 13. F
VII. (10 pts): Mỗi đáp án đúng được 1 điểm.
List of Book Titles Answers
0. Field Guide to Native Birds of Australia Book B
1. The Bush on Two Wheels: 100 Top Rides Book E
2. Bush Foods of Australian Aborigines X
3. A Pictorial History of the Dinosaur in Australia Book G
4. Bushwalking in Australia Book C
5. World Geographica Book D
6 Driving Adventures for 4-wheel-drive Vehicles Book I
7. Survival Techniques in the Wild Book H
8. Encyclopedia of Australian Wildlife Book F
9. Guide to the Art of the Australian Desert Book A
10. Field Guide to Animals of the World X

157
VIII (5 pts): Mỗi đáp án đúng được 0,5 điểm
1. C. doing 2. C.on 3. A. highly 4. C. situation 5. B. paid
6. C. which 7. B. fees 8. D.have to 9. A. include 10. D. considerable
IX. (5 pts): Mỗi câu viết đúng được 1 điểm
1. However wise man is, he may make mistakes.
2. What I found surprising was her lack of confidence.
3. You should not have allowed a four-year-old child to walk home alone.
4. The inother is proud of what her son contributed to the play last night.
5. We have booked five rooms, only two out of which have air conditioning.
X. (5 pts): Mỗi câu viết đúng được 1 điểm
1. Jenny wasn’t in the mood to go to the party?
2. I regretted (ever) having giyen/giving up that job.
3. No sooner had he finished school than he joined the army.
4. He object to anyone else/anyone else’s touching his records.
5. There has been a gradual decrease/decline/fall/reduction in the number of people out of work.
XI. Write a paragraph. (10 pts)
Write a paragraph from 160-180 words, beginning with "One day, my best friend did not go to school."
Các tiêu chí chấm bài viết đoạn văn.
Nếu lạc đề, bài viết sẽ bị điểm không (0)

Tiêu chí Mô tả tiêu chí đánh giá Điểm tối đa

Đúng cấu trúc, bố cục của đoạn văn:


√ Có câu chủ đề (topic sentence) mạch lạc, đúng chủ đề
0,5
√ Các câu văn chứng minh, diễn giải (supporting sentences) rõ ràng,
Bố cục 2,0
hợp lí (có ít nhất 2 câu)
0,5
√Câu kết (concluding sentence) rõ ràng, tóm tắt được chủ đề và các ý
đã triển khai
√ Viết đúng chủ đề, mạch lạc, nêu được các lập luận 1,0
Nội dung, √Các ý minh họa logic, có tính thuyết phục 1,0
phát triển ý √ Liên kết các ý chặt chẽ (Sử dụng từ nối, liên từ phù hợp) 1,0
√ Đảm bảo số lượng từ theo yêu cầu 1,0
√ Sử dụng ngôn từ phù hợp với chủ đề, phù hợp với văn viết 1,0
Ngôn ngữ, √ Đa dạng về cấu trúc và từ vựng, hạn chế lặp lại từ vựng 1,0
√Sử dụng đúng dấu câu, cấu trúc, thời thể đúng ngữ pháp 1,0
trình bày

Tổng điểm 10
Cách tính lỗi (trừ điểm)
- Xuống dòng -1,0
- Số lượng từ (+ - 20% số từ theo quy định) -1,0
- Mỗi lỗi ngữ pháp, cấu trúc câu, dấu câu, chính tả, -0,5
- Mỗi lỗi từ vựng -0,5

158

You might also like